You are on page 1of 189

Canadian Exam Answers 2007

1) Patient is brought to the hospital after smoke inhalation. The patient has a
headache, dizziness, and is confused. What is the percentage of
carboxyhemoglobin:

a) 15%
b) 25%
c) 35%
d) 45%
Answer A
Carbon Monoxide Toxicity
When significant smoke-induced lung injury occurs, must suspect CO toxicity, by
measurement of blood carboxyhemoglobin (COHb). A high FIO2 not only improves
oxygenation but promotes elimination of CO; an FIO2 of 1.0 decreases the blood half-life
of COHb from the 4 hours seen in room air to <1 hour.
CO interfere with mitochondrial cytochrome function, reducing ATP production and
causing met acidosis it also produces tissue hypoxia primarily by its 200-fold greater
affinity for hemoglobin than oxygen and by its ability to shift the hemoglobin dissociation
curve to the left.
Table 48-10. SYMPTOMS OF CO TOXICITY AS A FUNCTION OF THE BLOOD
CARBOXYHEMOGLOBIN (COHb) LEVEL
Blood COHb Level (%) Symptoms
<15–20 Headache, dizziness, and occasional confusion
20–40 Nausea, vomiting, disorientation, andvisual impairment
40–60 Agitation, combativeness, hallucinations, coma, and shock
>60 Death

2) What is the most important determinant of subarachnoid block height?

a) Patient height
b) Level of injection
c) Barbotage
d) Baricity
Answer D

3) In a patient going through alcohol withdrawal, which is the most likely


electrolyte abnormality post-operatively:

a) Hypomagnesemia
b) Hyponatremia
c) Hypokalemia
d) Hyperkalemia
Answer A

1
4) Maternal Administration of which of the following drugs is associated with a
decrease in FHR variability?

a) Atropine
b) Glycopyrolate
c) Propanalol
d) Metoprolol
Answer A
Chest nut 125-126
Coexisting 680
Causes of decreased FHR varriability
1) fetal hypoxia
2) fetal sleep state
3) fetal neurologic abnormality (ie anencephaly)
4) decreased CNS activity that results from exposure to drugs such as opioids
5) fetal arrhythmia
6) vagal blockade (ie- atropine, which readily crosses the bld brain barrier-
Chectnut p111).
7) intrathecal opioids .
8) magnesium.
9) Prematurity

Chestnut p114
Maternal administration of propranolol has little effect on FHR variability
The decompensation of cerebral blood flow and O2 delivery results in loss of FHR
variability.
FHR is mainly due to the influence of the Parasympathetic system

Chestnut p 421
Glycopyrolate does not cross the placenta, while atropine can cross the placenta
and cause increased FHR and Decreased beat to beat variability.

Chestnut p60
F/M ratios of Beta blockers:
Chronic propranolol >1
Acute propranolol 0.26
Metoprolol 0.94
Labetalol 0.38
Esmolol 0.2

For opioids
Morphine 0.61 ( if given intrathecally 0.92)
Meperidine >1 after 2-3 hours
Fentanyl 0.57
Sufenta 0.81
Remi 0.88
Alfenta 0.3

5) Which of the following is associated with an increase in IOP

a) STP 5mg/kg IV

2
b) Administration of N2O after intravitreal sulfahexafloride
c) Hyperventilate to a pCO2 of 30
d) Neuromuscular blockade with Rocuronium
Answer B
From Barash 5ed p976-991
 IOP is normal between 10 and 21.7mmHg, and is abnormal above 22mmHg.
 Straining , coughing and vomiting can raise IOP as much as 40 mmHg
 When using Sux , an average peak of IOP of 8mmHg occurs between 1 -4 min
after an IV dose, and IOP returns to baseline within 7 min
 CVP , BP, Hypoxemia and CO2 all effect IOP
 Avoid N2O for 5 days with air bubbles, 10 days w/ SF6, >30 days with
perfluoropropane
Barash 4ed
Table 34-5. CONCERNS WITH VARIOUSOCULAR PROCEDURES
Procedure Concerns

Strabismus repair Forced duction testing


Oculocardiac reflex
Oculogastric reflex
Malignant hyperthermia
Intraocular surgery Proper control of IOP
Akinesia
Drug interactions
Associated systemic disease
Retinal detachment Oculocardiac reflex
surgery Proper control of IOP
Nitrous oxide interaction with air, sulfur hexafluoride, or
perfluoro-carbons

6) Given that Pressure Gauge on an E cylinder of O2 says 950 mmHg and you are
delivering a 10 L/min flow rate, how long until the cylinder is empty?

a) 24 min
b) 32 min
c) 44 min
d) 60 min
Answer B
PV=PV and assuming 950 mmHg is gauge pressure and not absolute press
(950+760)* 5 = 760 * V
V will equal 11L ie the answer will be 1 min!!
But if it is 950psi, then

(950+14.7)*5 = 14.7 * V
V = 328L
If divided by 10, the answer is 32.8 min

3
I found this from this web page http://www.manuelsweb.com/O2remaining.htm

This applies to the E cylinder


It is easier to look at this formula :
0.35 x psi on gauge
L/min to be delivered

The 0.35 comes from dividing 660 by 1900.


(service capacity for an e-cylinder carrying oxygen is 1900 psi)
(volume of oxygen in an e-cylinder is 660 liters)

In the example above:


0.35 x 500 psi
= 28 min
6 L/min

Plus note:
1KPa = 1000Pa
1KPa = 7.5 mmHg = 10.2 cmH2O
1atm = 100KPa = 1 bar = 760mmHg = 14.7 psi
To convert from mmHg to psi , must divide by 51.71

7) An 80 kg patient is scheduled for surgery at 10:00 and she has fasted since
midnight, what is her fluid deficit?

a) 600 cc
b) 800 cc
c) 1200 cc
d) 1600 cc
Answer C

8) What is the immediate precursor of norepinephrine?

a) Dopamine
b) Epinephrine
c) Phenylalanine
d) Tyramine
Answer A
Miller

4
9) Which of the following medications accumulates the most in renal failure?

a) Esmolol
b) Digoxin
c) Propanalol
d) Nifedipine
e) Diltiazem
Answer B

Therapeutic plasma concentrations of digoxin can be achieved rapidly with IV


administration of the drug (up to 10 µg/kg over approximately 30 minutes), Clearance of

5
digoxin from the plasma is primarily by the kidneys, with approximately 35% of the drug
excreted daily. In the presence of renal dysfunction, the elimination half-time of digoxin is
depressed in proportion to the decrease in creatinine clearance. For example, the
elimination half-time of digoxin is 31 to 33 hours in the presence of normal renal function
and up to 4.4 days in the absence of renal function. The principal inactive tissue
reservoir site for digoxin is the skeletal muscles. A decrease in the size of this reservoir,
as in elderly patients, results in increased plasma and myocardial levels of digoxin.
Minimal amounts of digoxin accumulate in fat. Approximately 25% of digoxin is bound to
protein. Occasionally, patients form antibodies to digoxin, which prevents a therapeutic
effect. Metabolism of digoxin is minimal, with a few patients forming the inactive
metabolite dihydrodigoxin.

In addition to positive inotropic effects, cardiac glycosides enhance parasympathetic


nervous system activity, leading to delayed conduction of cardiac impulses through the
atrio-ventricular node and decreases in heart rate. The magnitude of this negative
dromotropic and chronotropic effect depends on the preexisting activity of the autonomic
nervous system.

The electrophysiologic effects of therapeutic plasma concentrations of cardiac


glycosides manifest on the ECG as (a) prolonged P-R intervals due to delayed
conduction of cardiac impulses through the atrioventricular node, (b) shortened Q-T
intervals because of more rapid ventricular repolarization, (c) ST segment depression
(scaphoid or scooped-out) due to a decreased slope of phase 3 depolarization of cardiac
action potentials, and (d) diminished amplitude or inversion of T waves. ST segment and
T wave changes on the ECG may suggest myocardial ischemia. When digitalis is
discontinued, the changes on the ECG disappear in approximately 20 days.

Dig toxicity

Causes

Hypokalemia probably increases myocardial binding of cardiac glycosides, resulting in


an excess drug effect. Other electrolyte abnormalities that contribute to digitalis toxicity
include hypercalcemia and hypomagnesemia. An increase in sympathetic nervous
system activity as produced by arterial hypoxemia increases the likelihood of digitalis
toxicity. Elderly patients with decreased skeletal muscle mass and decreased renal
function are vulnerable to the development of digitalis toxicity if the usual doses of
digoxin are administered. Impaired renal function and electrolyte changes (hypokalemia,
hypomagnesemia) that may accompany cardiopulmonary bypass could predispose the
patient to the development of digitalis toxicity.

Sx

Anorexia, nausea, and vomiting are early manifestations of digitalis toxicity. These
symptoms, when present preoperatively in patients receiving cardiac glycosides, should
arouse suspicion of digitalis toxicity. Excitation of the chemoreceptor trigger zone is the
principal mechanism responsible for vomiting. Transitory amblyopia and scotomata have
been observed. Pain simulating trigeminal neuralgia may be an early sign of digitalis
toxicity. The extremities may also be a site of discomfort.

6
Electrocardiogram

There are no unequivocal features on the ECG that confirm the presence of digitalis
toxicity. Nevertheless, toxic plasma concentrations of digitalis typically cause atrial or
ventricular cardiac dysrhythmias (increased automaticity) and delayed conduction of
cardiac impulses through the atrioventricular node (prolonged P-R interval on the ECG),
culminating in incomplete to complete heart block. Atrial tachycardia with block is the
most common cardiac dysrhythmia attributed to digitalis toxicity. Activity of the sinoatrial
node may also be directly inhibited by high doses of cardiac glycosides. Conduction of
cardiac impulses through specialized conducting tissues of the ventricles is not altered,
as evidenced by the failure of even toxic plasma concentrations of digoxin to alter the
duration of the QRS complex on the ECG. Ventricular fibrillation is the most frequent
cause of death from digitalis toxicity.

Treatment of digitalis toxicity includes (a) correction of predisposing causes


(hypokalemia, hypomagnesemia, arte-rial hypoxemia), (b) administration of drugs
(phenytoin, lidocaine, atropine) to treat cardiac dysrhythmias, and (c) insertion of a
temporary artificial transvenous cardiac pacemaker if complete heart block is present.
Supplemental potassium decreases the binding of digitalis to cardiac muscle and thus
directly antagonizes the cardiotoxic effects of cardiac glycosides. Life-threatening
digitalis toxicity can be treated by administering antibodies (Fab fragments) to the drug,
thus decreasing the plasma concentration of cardiac glycosides available to attach to
cell membranes. The Fab-digitalis complex is eliminated by the kidneys.

Propranolol is metabolized by the liver

Calcium Channel blockers

Diltiazem
The drug is 70% to 80% bound to proteins and is excreted as inactive metabolites
principally in bile (about 60%) and, to a lesser extent, in urine (about 35%). The
elimination half-time is 3 to 5 hours.

Nifedipine
Protein binding approaches 90%. Hepatic metabolism is nearly complete, with
elimination of inactive metabolites principally in urine (about 80%) and, to a lesser
extent, in bile. The elimination half-time is 2 to 5 hours.

Verapamil

The elimination half-time of verapamil is 6 to 12 hours, and this may be prolonged in


patients with liver disease. Like nifedipine, verapamil is highly protein bound (90%), and
the presence of other drugs (lidocaine, diazepam, propranolol) can increase the
pharmacologically active, unbound portion of the drug. Chronic oral administration of
vera-pamil or the presence of renal dysfunction leads to the accumulation of
norverapamil, which does have some antidysrhythmic properties

7
TABLE 18-2. Comparative pharmacologic effects of calcium channel blockers
Verapamil Nifedipine Nicardipine Diltiazem
Systemic blood Decrease Decrease Decrease Decrease
pressure
Heart rate Decrease Increase to no Increase to no Decrease
change change
Myocardial depression Moderate Moderate Slight Moderate

Sinoatrial node Moderate None None Slight

8
depression
Atrioventricular node Marked None None Moderate
conduction depression
Coronary artery Moderate Marked Greatest Moderate
dilation
Peripheral artery Moderate Marked Marked Moderate
dilation

10) A 55 y/o male presents with new onset hypertension with a hypokalemic
metabolic alkalosis, and a decreased plasma renin activity, what is the most
likely diagnosis?

a) Cushing's
b) Pheo
c) RAS simulation (UWO version is carcinoid)
d) Primary hyporaldosteronism (UWO version is Primary
hyporaldosteronism)
ANSWER D
COEX P429
Primary Hyperaldosteronism (Conn Syndrome)
In Primary Hyperaldosteronism, there is excess secretion of anldosterone from a fxnal
tumer , independent of a physiologic stimulus. Plasma rennin is suppressed in Pts w/
untreated Primary hyperaldosteronism. Secondary Hyperaldos. Is present when an
increased levels of rennin stimulate the release of aldosterone.
Sx & Sg & Dx features
 Can be assymtomatic
 Headache ( from the hypertension, which is due to Na reabsorbtion and
increased Circulating volume)
 Hypokalemia (polyuria –which is due to hypokalemic nephropathy causing a
defect in urine concentrating ability—, skeletalmuscle cramps , skeletal muscle
weekness)
 Hypokalemic metabolic alkalosis
 Can also have HypoMg and abnormal glucose tolerence

Tx
Supplemental K
Considerations
Aldosterone antagonist such as Spironolactone  Hypokalemia
Antihypertensive drugs,  Hypervolemia & HTN or
Hypovolemia
Management of Anesthesia  Alkalosis
 Correct preop K  Periop steroid coverage
 Assoc HypoMg

9
 Avoid Hyperventilation, as K changes by 0.5 mmol/l w/ each 10mmHg
change in CO2
 Hypokalemia effect non depolarizing blockers
 Hyper or Hypovolemic depending on preop Tx ( well duiresed or not)
 Bil adrenalectomy requires steroid supplementation (100mg IV over 24 hrs)

Hypoaldosteronism
Hyperkalemia in the presence of N renal fxn suggests hypoaldosteronism.
The HyperK can cause Heart block. HypoNa may be present. These pts can also
have Hyperchloremic metabolic acidosis.
This can be congenital or due to Hyporeninemia or ACE inh that lead to loss of
angiotensin stimulation . It is Tx w/ liberalNa intake and fludrocortisone

Renin released into the circulation interacts with angiotensinogen to cleave off the
decapeptide angiotensin I; through converting enzymatic activity, this becomes the
active octapeptide angiotensin II. Angiotensin II is a potent vasopressor and has a
direct effect on the adrenals to stimulate aldosterone. By its vasoconstrictive action,
angiotensin II increases salt retention.

10
CUSHING SYNDROME
Summary of Anesthetic Considerations:
1. Cause: ↑ICP and pituitary function (DI/Thyroid)
2. Hypertension
3. Hyperglycemia
4. Hypervolemia → possible CHF (normalize with spironolactone)
5. Hypokalemic alkalosis
6. Obesity (access, intubation)
7. Psychiatric issues (including frank psychosis)
8. Weakness:
a. Potential respiratory muscle weakness → hypoventilation
b. Decrease doses of muscle relaxants

11
11) All of the following are signs and symptoms for NMS EXCEPT
a) Diaphoresis
b) Rigidity
c) Hypertension
d) Urinary retention
Answer D
Barash
The symptoms and signs of the neuroleptic malignant syndrome include fever,
rhabdomyolysis, tachycardia, hypertension, agitation, muscle rigidity, and acidosis.44 The
mortality rate is unknown, but may be as high as 20%.
P&P
Neuroleptic Malignant Syndrome
Neuroleptic malignant syndrome occurs in 0.5% to 1.0% of all patients treated with
antipsychotic drugs. Risks factors for the development of this syndrome may include
dehydration and intercurrent illness.
The syndrome typically develops over 24 to 72 hours in young men and is characterized
by :
(a) hyperthermia;
(b) generalized hypertonicity of skeletal muscles;
(c) instability of the autonomic nervous system manifesting as alterations in systemic
blood pressure, tachycardia, and cardiac dysrhythmias; and
(d) fluctuating levels of consciousness.

Autonomic nervous system dysfunction may precede the onset of other symptoms.
Increased skeletal muscle tone may so decrease chest wall expansion that it becomes
necessary to provide mechanical support of ventilation. Skeletal muscle rigidity may be
severe enough to cause myonecrosis leading to increased creatine phosphokinase
levels, myoglobinuria, and renal failure. Liver transaminase enzymes are likely to be
increased. Mortality is 20% to 30%, with common causes of death being ventilatory
failure, cardiac failure and/or dysrhythmias, renal failure, and thromboembolism.
The cause of neuroleptic malignant syndrome is not known and, as a result, treatment is
empirical and includes supportive measures and the administration of the direct-acting
muscle relaxant dantrolene and the dopamine agonists bromocriptine or amantadine.
The reported efficacy of dopamine agonists in the treatment of skeletal muscle rigidity as
well as the prevention of the onset of the syndrome with abrupt withdrawal of levodopa
therapy suggests a role of dopamine receptor blockade in the development of this
syndrome
Malignant hyperthermia associated with anesthesia as well as the central anticholinergic
syndrome may mimic the neuroleptic malignant syndrome. A distinguishing feature is the
ability of nondepolarizing muscle relaxants to produce flaccid paralysis in patients
experiencing the neuroleptic malignant syndrome but not in those experiencing
malignant hyperthermia
CoEx p638
Sux can be used in pts w/ NMS , as there is no evidence of pathophysiologic link
between the two disorders, but pts should be monitored closely during GA

12) All EXCEPT ONE of the following are factors for postoperative apnea in a
former preterm infant. Indicate the exception:
a) Gestational age at delivery
b) Postconceptual age at time of surgery

12
c) General anesthesia
d) Presence of anemia

Answer A (Hani says C )


Anemia (hematocrit <30) is also associated with an increased incidence of apnea in
preterm infants < 60 conceptual weeks.
Some techniques have been tried to eliminate the problem of postoperative apnea in
infants, to allow them to go home on the same day of a procedure. When caffeine 10
mg/kg was given immediately after induction of anesthesia, no preterm infants £44
conceptual weeks developed postoperative apnea. Spinal anesthesia, without the use of
other drugs intraoperatively or postoperatively, also was not associated with apnea(Fig.
46-3). Although preterm infants who have received caffeine or an unsupplemented
spinal anesthetic may be discharged on the day of the procedure, they should probably
be observed overnight in a hospital until more data confirm the safety of this practice.

Postoperative Apnea. Nelson‘ pediatrics p353


The risk of developing postoperative apnea in premature neonates is inversely
proportional to postconceptual age at the time of surgery. This risk is minimal by the
time premature infants have reached the postconceptual age of 60 wk. Apnea is
most common within the first 12hr after surgery; postanesthetic apnea has been
reported in premature infants up to 48hr later. It is generally agreed that general
anesthesia should be avoided, except for emergent surgery, in full-term children younger
than 44 wk postconceptual age. If surgery is required within the first month, overnight
observation and monitoring is indicated. Theophyllines decrease the incidence of
postoperative apnea; however, they do not ablate it and are, therefore, not routinely
used. The safest course is to monitor premature infants younger than 60 wk
postconceptual age and full-term infants younger than 1 mo old for at least 24hr
after anesthesia.

Although the risk of apnea may be less with regional anesthesia, apnea may still
occur and may even be increased if regional anesthesia is combined with sedation
(ketamine, midazolam)

13) All EXCEPT ONE of the following is seen in a patient with a third degree burn
covering 50% body surface area within the first one hour. Indicate the
exception:
a) Hypertension
b) Hypothermia
c) Hyperkalemia from succinylcholine
d) Hyperthermia

Answer C (2001 Question)


A: No clear reference. I think it makes sense given the severe pain and the
catecholamine surge.
Anesthetic Mx
A hypermetabolic state characterized by tachycardia, tachypnea, catecholamine surge,
increased O2 consumption, and augmented catabolism follows the initial few hours of a
burn injury. Anesthesia may be required during all three phases of burn injury: during the

13
resuscitation phase (first 24–48 hours), during convalescence (a few weeks) for
escharotomies, and during reconstruction (6 months). Extensive and repeated
escharotomies may be required during the initial phase of convalescence, usually
between the second day and the second week after injury, often necessitating massive
transfusion, temperature control, and management of fluid, electrolyte, and coagulation
abnormalities
Large-bore iv catheters are essential. Hyperthermia occurs, but hypothermia is more
likely in the operating room and is to be avoided.
Stoelting: Anes & Co-existing Disease, 4th Ed, Ch 32
BURN (THERMAL) INJURIES
Cardiac Output – decreases dramatically in the immediate post burn period. The
initial decrease precedes any measurable loss of intravascular fluid volume and may
reflect the presence of circulating low-molecular-weight myocardial depressant
factors. …. Sudden circulatory decompensation may occur if the depressant effects
of volatile anesthetic are superimposed.
After the initial 24 hours of resuscitation the circulatory system enters a
hyperdynamic state that persists will into the postburn period.

Early CVS response – hypovolemia (burn shock), impaired myocardial contractility

Systemic Hypertension – Approximately 30% of children with extensive thermal


injury become hypertensive during the post-burn period. The onset of systemic
hypertension is usually within the first 2 weeks.

Metabolism and Thermoregulation – The metabolic rate increases in proportion to


the extent of burn injury. It can be more than doubled in individuals with burn injury
involving 50% of the body surface area. Accompanying these hypermetabolic
responses, the metabolic thermostat is reset upward so burn patients tend to
increase skin and core temperatures somewhat above normal, regardless of the
environmental temperatures.

14) The effects of positive pressure ventilation and it resulting in decreased CO


include all EXCEPT

a) RV dysfunction
b) Decreased preload
c) Decreased LV compliance
d) Increased LV afterload

Answer D

Effects of PPV and PEEP


(Miller 6th ed p2820,)

Pulmonary effects
i) redistribution of extravascular water → improved oxygenation → ↑ lung compliance
→ improved V/Q matching.
ii) ↑‘d FRC (alveolar recruitment)→ ↓‘d shunt

Cardiovascular effect
i) ↓ Preload → ↓ CO

14
a) Decreased venous return
- ↑‘d intrathoracic pressure results in ↓‘d pressure gradient for venous flow into
thorax → ↓‘d EDV of both Ventricles
b) Decreased Ventricular Distensibility
- ↑‘d +ve intrathoracic pressure on the outer surface of ventricles → reduced
ventricular filling during diastole
c) RV dysfuction
- PPV → ↑ PVR → ↑ RV afterload
d) Decreased LV Distensibility
- PPV → ↑PVR → ↑RV volumes → septal shift → ↓ LV distensibility → ↓ LVEDV

ii) ↓‘d Afterload


- PPV → ↓‘d transmural pressure across ventricles → ↓ afterload
There is evidence that even in the presence of normal compliance, PEEP of greater than
10 cm H2O makes measurement of pulmonary capillary wedge pressure unreliable.

15) All are true for SIADH, EXCEPT


a) Serum Na and osmolality are decreased
b) It occurs in a substantial number of patients post-op
c) Chlorpropamide is useful for treatment
d) It occurs most often between 3-10 days and lasts 10-15 days

Answer C (2003/2001)

Causes of SIADH
Intracranial tumors
Hypothyroidism
Pulmonary infections
Head injuries
Porphyria and carcinoma of the lung (small cell)
Alleged to occur in most patients following major surgery

• Hyponatremia is due to dilution


• Increased urinary sodium concentrations and osmolarity in the presence of
hyponatremia and decreased serum osmolarity

TREATMENT
• Restricted oral fluid intake (about 500 ml), antagonism of the effects of ADH on the
renal tubules by administration of demeclocycline (and lithium), and IV infusions of
sodium chloride (hypertonic saline if there are acute neurologic symptoms)

• chlorpropramide (a sulfonylurea) is an effective treatment for diabetes insipidus


by sensitizing the renal tubules to ADH therefore is useful for nephrogenic DI (not
central DI)

• The most common endocrine complication after a TBI (traumatic brain injury) is SIADH
• Hyponatremia often is seen in the rehabilitation setting among survivors of either
traumatic or nontraumatic brain injury (eg, hemorrhagic stroke, brain tumors, CNS
infections). This problem is associated most often with SIADH*

16) The following are signs of fat embolism syndrome except:

15
a) Hypertension
b) Hypoxemia
c) Petechiae
d) Mental confusion
Answer A

Table 40-2. CRITERIA FOR DIAGNOSIS OF FAT EMBOLUS SYNDROME


MAJOR
Axillary/subconjunctival petechiae
Hypoxemia (PaO2 < 60 mm Hg; FIO2 < 0.4)
CNS depression (disproportionate to hypoxemia)
Pulmonary edema
MINOR
Tachycardia (>110 beats·min–1)
Hyperthermia
Retinal fat emboli
Urinary fat globules
Fat globules in sputum
Decreased platelets/hematocrit (unexplained)
Increased erythrocyte sedimentation rate
Diagnosis of fat embolus syndrome requires at least one major and four
minor criteria

A certain degree of lung dysfunction occurs in all patients after long bone fractures, but
clinically significant fat embolism syndrome as such develops in only 10-15% of these
patients. Signs include hypoxia, tachycardia, mental status changes, and petechiae on
the conjunctiva, axilla, or upper thorax. Fat globules in the urine are nondiagnostic, but
lung infiltrates seen on CXR confirm the presence of lung injury

The pathophysiology of fat embolism represents capillary endothelial breakdown causing


pericapillary hemorrhagic exudates most apparent in the lungs and brain. Pulmonary
edema and hypoxemia occur as a result of pulmonary exudates. Hypoxia and areas of
cerebral edema may account for the variable neurologic abnormalities seen.

17) Regarding intercerebral aneurysms, all are true EXCEPT:


a) Common post-mortem finding
b) Usually involves anterior cerebral circulation
c) Cigarette smoking increases the risk of rupture
d) Prodromal headache occurs in a small proportion of patients prior to
rupture
Answer D
RPT Q257 2003
• Most saccular aneurysms arise on the circle of Willis
Anterior circulation aneurysms:
• Approximately 85% of all intracranial aneurysms arise on the anterior (carotid)
circulation therefore B is true**

16
• Common locations are the anterior communicating artery (30-35%), the internal carotid
artery (ICA) at the posterior communicating artery origin (30-35%), and the MCA
bifurcation (20%).
Posterior circulation aneurysms:
• About 15% of all intracranial aneurysms arise on the posterior (vertebrobasilar)
circulation
• Common posterior fossa sites include the superior cerebellar artery and the vertebral
artery (VA) at the origin of the posterior inferior cerebellar artery (PICA). Anterior inferior
cerebellar artery (AICA) aneurysms are rare.

• Systemic hypertension and smoking are risk factors for aneurysmal rupture (C is true)
• Family history in a first degree relative is also a risk factor
• Risk of aneurysm rupture depends on the size of the aneurysm

• Intracerebral hemorrhage accounts for 11% of stroke mortality and notably affects
African Americans
Estimated incidence of cerebral aneurysms in North America is 1:50 with an average
incidence of aneurismal rupture of 12: 100 000.
Most aneurysms are asymptomatic remaining undetected throughout life (A is true)*
• The estimated volume of blood and the level of consciousness are the most
reliable predictors of outcome

• Prodromal ―sentinel leaks” or “warning leaks‖ occur in about 50-70% of patients


who suffer a ruptured cerebral aneurysm (NOT a ―small proportion‖ therefore D is false)
• These leaks are frequently missed

Hunt and Hess classification is useful because it serves as an indicator of overall


prognosis as well as of surgical risk (E is true)

Grade Perioperative mortality (%)


0 Unruptured aneurysm (0-2%)
I Mild symptoms (slight headaches) 0.5%
II Severe headache, cranial nerve palsy (2-10%)
III Drowsiness, mild focal neurologic deficit (10-15%)
IV Coma (stupor, severe hemiparesis (60-70%)
V Coma, decerebrate posturing (70-100%)

18) All are risk factors for ventricular dysfunction post CABG except?
a) Advanced age
b) Female gender
c) Diabetes
d) Decreased ejection fraction preoperatively
Answer C
DM is associated with an increased risk of sternal wound infection and long term
complication.
Reference: John Butterworth, Perioperative myocardial failure. IARS 2001 review
course lecture.

17
Increasing age, female sex, decreased LVEF preop. And increased duration of
CBP are all associated with greater likelihood that inotropic drug support will be
administered after CABG.

Comparison of Factors Associated With 30-Day Mortality After Coronary Artery


Bypass Grafting in Patients With Versus Without Diabetes Mellitus
Multivariate logistic regression analysis identified female gender, 3-vessel disease, and
left main disease as independent risk factors for 30-day, post-CABG mortality unique to
diabetic patients. Left ventricular dysfunction was found to effect a greater risk among
diabetic patients, whereas chronic renal failure was associated with greater risk among
nondiabetics.

Anesth Analg. 1998 Mar;86(3):461-7.


FACTORS THAT PREDICT THE USE OF POSITIVE INOTROPIC DRUG SUPPORT AFTER CARDIAC
VALVE SURGERY.
Butterworth JF 4th, Legault C, Royster RL, Hammon JW Jr.

IMPLICATIONS: We evaluated factors related to use of positive inotropic drugs after


cardiac valve surgery. The likelihood of a patient receiving these drugs increases with
advancing age and with more severe preoperative left ventricular dysfunction, but was
not influenced by the specific diseased valve or the duration of cardiopulmonary bypass.

19) All are anatomical structures participating in ascending pathway for pain
perception except?
a) Ventral posterolateral nucleus of thalamus
b) Lateral cervical nucleus
c) Postganglionic sympathetic fibers
d) Dorsolateral funiculus
Answer C
Barash 5 ed, Ch 56 p 1447
Ascending pathway:
1- spinothalamic tract
a. origin lamina I and V, Majority of fibres cross near their level of origin
b. 2 fxnal divisions
i. (Lateral) Neospinothalamic: discriminative function (intensity ,
location) . Project to post thalamic nuclei (eg ventral
posterolateral nucleus)
ii. Medial Paleospinaothalamic: autonomic and emotionally
unpleasant aspect of pain , project to medial thalamic nuclei. Imp
in pain assoc w/ denervation dysesthesia.
2- spinoreticular tract:
a. origin unknown
b. produce pain related arousal contributes to neural activity underlying
motivational, affective, and autonomic responses to pain.
3- spinomesecephalic tract
a. projects to the midbrain reticular formation.
b. evokes nondiscriminative painful sensations and may be important in the
activation of descending antinociceptive pathways.
4- spinocervical tract

18
a. It is located in the dorsolateral funiculus (dorsal part of spinal cord).
Fibres ascend uncrossed to lateral cervical nucleus which serves as a
relay, sending fibers to the contralateral thalamus.

5- Spinohypothalamic & Spinotelencephalic tracts activates the hypothalamus and


evoke emotional behaviour
6- some fibers in the dorsal columns are responsive to noxious stimuli.

20) All of the following are clinical manifestations of sarcoidosis, EXCEPT ONE.
Identify the exception.
a) Pulmonary hypertension
b) Hilar lymphadenopathy
c) Hypocalcemia
d) Cardiomyopathy
Answer C
Sarciodosis
Summary of Anesthetic Considerations:
1. Airway:
a. Laryngeal sarcoid = narrowed or obstruced larynx (look for voice
changes, SOB, nasal obstruction)
b. Extrinsic airway compression (thoracic lymph nodes)
2. Systemic manifestations:
a. Resp: pulmonary fibrosis, restrictive lung disease, extrinsic airway
compression (thoracic lymph nodes), cysts and/or bullae (stage IV
disease), bronchoreactivity
b. CVS: cor pulmonale, heart block, arrhythmia, restrictive cardiomyopathy,
valvular dysfunction, pericardial effusion
c. CNS: cranial nerve palsy, neuromuscular sarcoid may result in peripheral
neuropathy or myopathy
d. GI: Hepatic dysfunction (uncommon)
e. Renal: renal dysfunction from nephrolithiasis (hypercalcemia)
f. Haem: leukopenia or thrombocytopenia due to splenic sequestration
(anemia, thrombocytopenia may also occur due to bone-marrow
involvement)
g. Endocrine: hypothyroidism, hypopituitarism (diabetes insipidus)
h. Metabolic: hypercalcemia
3. Effects of treatment:
a. Perioperative steroid coverage
b. Immunosuppression (strict asceptic technique)

21) The secretion of all of the following hormones, EXCEPT ONE, is increased
during stress. Identify the exception

a) Insulin
b) Glucagon
c) ADH
d) Epinephrine
Answer A

22) All EXCEPT one of the following drugs are appropriate in the management of a
hemodynamically stable patient with paroxysmal supraventricular tachycardia

19
and impaired left ventricular function according to ACLS guidelines. Indicate
the EXCEPTION:
a) Diltiazem
b) Metoprolol
c) Digoxin
d) Amiodarone
Answer C Refs\Brady & Tachy Circulation 2005.pdf As Digoxin, is not even in the
guidelines but, B blockers can be used w/ caution.
Vagal maneuvers and adenosine are the preferred initial therapeutic choices for the
termination of stable reentry SVT (Box 7). Vagal maneuvers alone (Valsalva maneuver
or carotid sinus massage) will terminate about 20% to 25% of reentry SVT14; adenosine
treatment is required for the remainder.
Adenosine. If reentry SVT does not respond to vagal maneuvers, give 6 mg of IV
adenosine as a rapid IV push (Class I). Although 2 randomized clinical trials
documented a similar SVT conversion rate between adenosine and calcium channel
blockers, adenosine was more rapid with fewer severe side effects than verapamil.
Adenosine, however, does have several important drug interactions. Larger doses may
be required for patients with a significant blood level of theophylline, caffeine, or
theobromine. The initial dose should be reduced to 3 mg in patients taking dipyridamole
or carbamazepine, those with transplanted hearts, or if given by central venous access.
Side effects with adenosine are common but transient; flushing, dyspnea, and chest pain
are the most frequently observed. If the rhythm does convert (Box 9), it was probably
reentry SVT. Monitor the patient for recurrence and treat any recurrence with adenosine
or control the rate with a longeracting AV nodal blocking agent (eg, diltiazem or B
blocker). Amiodarone can achieve nearly 100% efficacy in the inhibition of induced
sustained reentrant SVT (LOE 6).22 Adenosine is safe and effective in pregnancy.

Calcium Channel Blockers and B Blockers. If adenosine fails to convert reentry SVT
(Box 10), attempt rate control with a nondihydropyridine calcium channel blocker (ie,
verapamil or diltiazem) or B blocker as a second-line agent (Class IIa).25–27 These
drugs act primarily on nodal tissue either to slow the ventricular response to atrial
arrhythmias by blocking conduction through the AV node or to terminate the reentry SVT
that depends on conduction through the AV node.
Verapamil and, to a lesser extent, diltiazem may decrease myocardial contractility and
critically reduce cardiac output in patients with severe left ventricular dysfunction.
Calcium channel blockers that affect the AV node (including verapamil and diltiazem) are
considered harmful when given to patients with atrial fibrillation or atrial flutter associated
with known pre-excitation (Wolff-Parkinson-White [WPW]) syndrome. B Blockers
should be used with caution in patients with pulmonary disease or congestive
heart failure. For verapamil, give a 2.5 to 5 mg IV bolus over 2 minutes (over 3 minutes
in older patients). If there is no therapeutic response and no drug-induced adverse
event, repeated doses of 5 to 10 mg may be administered every 15 to 30 minutes to a
total dose of 20 mg. An alternative dosing regimen is to give a 5-mg bolus every 15
minutes to a total dose of 30 mg. Verapamil should be given only to patients with
narrowcomplex reentry SVT or arrhythmias known with certainty to be of
supraventricular origin. It should not be given to patients with impaired ventricular
function or heart failure.
For diltiazem, give a dose of 15 to 20 mg (0.25 mg/kg) IV over 2 minutes; if needed, in
15 minutes give an IV dose of20 to 25 mg (0.35 mg/kg). The maintenance infusion dose
is 5 to 15 mg/h, titrated to heart rate.

20
21
23) All EXCEPT one of the following are true about Nd-YAG laser. Indicate the
EXCEPTION:

a) Can ignite laser resistant tubes


b) Transmit close to infrared
c) Only penetrates superficially in tissue
d) Reflects from fiberoptics
Answer C

The Laser Flex tube (Mallinckrodt Inc., St. Louis, Mo) is an airtight stainless steel spiral
with two distal, saline-inflatable PVC cuffs (redundant in case of puncture of the proximal
cuff). This tube is resistant to CO2 and KTP laser energy, but not to the Nd-YAG laser.

Miller
Long infrared wavelengths are absorbed with great efficiency by water, the main
constituent of tissue. Long-wavelength laser light, such as that from CO2 (operating at
10,600 nm), is therefore completely absorbed by water in the first few layers of cells.
With powerful, focused CO2 beams, this results in explosive vaporization of the surface
tissue of the target with surprisingly little damage to underlying cells. Excimer lasers
have an extremely precise effect because they produce ultraviolet light that is absorbed
more intensely by water and other molecules. These lasers are of great value in office-
based refractive surgery. Near-infrared light from an Nd:YAG laser at 1064 nm is
less absorbed by water, and the beam diffuses through several millimeters,
scattering through a volume of tissue perhaps 100 to 1000 times the volume
through which a CO2 beam diffuses. Consequently, the energy of an Nd:YAG
beam is more widely disseminated ( Fig. 67-5 ), producing less vaporization and
more thermal coagulation (i.e., a cooking effect). Some of the effects of this bulk
coagulation may not be apparent for hours or days after exposure. The red light
produced by a ruby laser (694 nm) is poorly absorbed, except by cells containing dark
pigment. The green and blue light produced by argon (514,488 nm) or krypton
(476,521,568 nm) gas lasers is transmitted by water but is intensely absorbed by
hemoglobin, providing the ability to penetrate skin or ocular structures and selectively
coagulate vascular or pigmented regions. Although lasers that produce infrared or visible
light have exclusively thermal effects on tissue, the photons produced by ultraviolet
excimer lasers are energetic enough to disrupt chemical bonds directly and cause
ionization, which can result in mutation and carcinogenesis.

22
Table 67-2 -- Laser media wavelengths
Wavelength
Laser Medium Color Typical Application
(nm)
CO2 Far infrared 10,600 General, cutting
Erbium-YAG Infrared 2,930 Dental, arthroscopy
Holmium-YAG Infrared 2,060 Angioplasty
Near General, coagulation, with
Neodymium-YAG 1,064
infrared fiberoptics
Ruby Red 694 Tattoos, nevi
Krypton Yellow 568 Retina
Argon Green 514 Vascular, pigmented lesions
Xenon chloride Ultraviolet 308 Cornea, angioplasty
Krypton chloride Ultraviolet 222 Cornea, angioplasty
KTP, potassium-titanium phosphate; YAG, yttrium-aluminum-garnet.

23
Fiberoptic bundles provide a convenient, flexible conduit for visible and near-
infrared wavelengths. Wavelengths outside this range, such as the long infrared
from a CO2 laser.

24) All are true in transfusion reaction, EXCEPT


a) Hyperthermia
b) Rigidity
c) Bleeding
d) Hypotension
Answer B
Miller
Table 47-8 -- Frequency and signs and symptoms of hemolytic transfusion reactions in
40 patients
Sign or Symptom No. of Patients
Fever 19
Fever and chills 16
Chest pain 6
Hypotension 6
Nausea 2
Flushing 2
Dyspnea 2
Hemoglobinuria 1
The classic signs and symptoms ( Table 47-8 ) of a hemolytic transfusion reaction—
chills, fever, chest and flank pain, and nausea—are masked by anesthesia. Under
general anesthesia, the only signs may be hemoglobinuria, bleeding diathesis, or
hypotension.

Treatment

If a hemolytic reaction is suspected, blood and urine samples should be sent to the
laboratory for examination. The blood bank should check all paperwork to ensure that
the correct blood component was transfused to the patient. Laboratory tests should be
performed to determine the presence of hemoglobinemia: a direct antiglobulin test,
repeat compatibility testing, repeat other serologic tests (i.e., ABO and Rh) and analysis
of urine for hemoglobinuria.

The systems most effected:


 Renal system
the most common hypothesis is that hemoglobin in the form of acid hematin
precipitates in the distal tubule and causes mechanical tubular blockage. The
magnitude of the precipitation probably is inversely related to the volume of urine
flow and its pH.
 Coagulation
DIC commonly occurs with hemolytic transfusion reactions, probably because RBC
stroma is severed, releasing erythrocytin, which activates the intrinsic system of
coagulation. This activated coagulation leads to fibrin formation. Subsequently,
platelets and factors I, II, V, and VII are consumed. As soon as a hemolytic

24
transfusion reaction is recognized, platelet count, prothrombin time, and partial
thromboplastin time should be determined to provide baseline values with which
subsequent laboratory values can be compared. Hypotension during a hemolytic
transfusion reaction may result from activation of the kallikrein system.[103] After a
series of reactions, plasma kininogen is converted to bradykinin, a potent vasodilator
that can cause hypotension.

Table 47-9 -- Steps for the treatment of a hemolytic transfusion reaction


1. STOP THE TRANSFUSION.
2. Maintain the urine output at a minimum of 75 to 100 mL/hour by the following
methods:
a. Generously administer fluids intravenously and possibly mannitol (12.5 to 50 g,
given over 5 to 15 minutes).
b. If intravenously administered fluids and mannitol are ineffective, administer
furosemide (20 to 40 mg) intravenously.
3. Alkalinize the urine; because bicarbonate is preferentially excreted in the urine, only
40 to 70 mEq of sodium bicarbonate per 70 kg of body weight is usually required to raise
the urine pH to 8, whereupon repeat urine pH determinations indicate the need for
additional bicarbonate. (Alkalinization of the urine to prevent precipitation of acid hematin in the
distal tubules is of questionable value but is easy and therefore recommended)
4. Assay urine and plasma hemoglobin concentrations.
5. Determine platelet count, partial thromboplastin time, and serum fibrinogen level.
6. Return unused blood to blood bank for repeat crossmatch.
7. Send patient's blood and urine sample to blood bank for examination.
8. Prevent hypotension to ensure adequate renal blood flow

Delayed Hemolytic Transfusion Reaction (Immune Extravascular Reaction)


In many cases of hemolytic transfusion reaction, the transfused donor cells may survive
well initially, but after a variable delay (2 to 21 days), they are hemolyzed.[105][106] This
type of reaction occurs mainly in recipients sensitized to RBC antigens by previous blood
transfusions or pregnancy.
Sx & Sg (after 2-21 days)
 Decrease in Hct
 Jaundice
 Hemoglobinuria
 impairment in renal function
 Rarely , death

The most common adverse reactions to blood transfusions are the febrile reactions. The
symptoms consist of chills, fever, headache, myalgia, nausea, and nonproductive cough
occurring shortly after blood transfusion. Less frequently, the patient may have
hypotension, chest pain, vomiting, and dyspnea. Even pulmonary infiltrations with x-ray
evidence of prehilar nodule formation and lower lung infiltrates along with overt
pulmonary edema have been reported.[107] Because febrile reactions obviously involve
fever, they can be easily confused with a hemolytic transfusion reaction. A direct
antiglobulin test readily differentiates a hemolytic reaction from a febrile reaction

25
because this test rules out the attachment of an RBC antibody to transfused donor
RBCs.

Most allergic transfusion reactions are mild and are thought to be caused by the
presence of foreign protein in the transfused blood. The most common symptom is
urticaria associated with itching. Occasionally, the patient has facial swelling. Allergic
reactions occur in about 3% of all transfusions.
Infrequently, a more severe form of allergic reaction involving anaphylaxis occurs in
which the patient has dyspnea, hypotension, laryngeal edema, chest pain, and shock.
These are anaphylactic reactions caused by the transfusion of IgA to patients who are
IgA deficient and have formed anti-IgA. This type of reaction does not involve red cell
destruction and occurs very rapidly, usually after the transfusion of only a few milliliters
of blood or plasma. The patients who experience these anaphylactic reactions must be
given transfusions with washed RBCs from which all traces of donor IgA have been
removed or with blood lacking the IgA protein.

25) The risk of HIV transmission after a needlestick injury from a patient with HIV
include all of the following except:

a) Gender
b) Hollow needle
c) Deep injury
d) Advanced HIV
Answer A

26
26) In a patient with severe traumatic brain injury with normal head CT, all are
indications for intracranial pressure monitoring except?
a) Age over 40
b) Systolic blood pressure lower than 90
c) Hemoglobin lower than 80
d) Motor posturing
Answer C
Miller
Guidelines for Intracranial Pressure Monitoring
A patient with a diagnosis of head trauma, Reye's syndrome, or metabolic
encephalopathy is a candidate for ICP monitoring if the following criteria are met: a GCS
score of 8 or less, clinical signs of increased ICP, and evidence of increased ICP on CT.
Coagulopathies must be corrected before any ICP monitor is placed.

Brain Trauma foundation Guidelines:


http://www2.braintrauma.org/guidelines/downloads/btf_guidelines_management.pdf
ICP monitoring is appropriate in Pts w/ severe head injury w/ an abnormal CT
scan. Severe head injury is defined as a GCS score of 3 – 8 after resus. An
abnormal CT head , is one that reveals hematomas , contusion, edema or
compressed basal cisterns.
ICP monitoring is appropriate in Pts w/ severe head injury and w/ a Normal CT
scan, if 2 or more of the following features are noted on admission :
1. Age over 40
2. Unilateral or bilateral motor posturing
3. SBP < 90
ICP monitoring is not indicated in pts w/ mild or mod head injury

27) All of the following statements regarding acute respiratory distress syndrome
(ARDS) are true EXCEPT ONE. Identify the exception.

a) Blood gas analysis in early ARDS may show initial respiratory alkalosis.
b) The radiographic images of the lungs in ARDS can be used as an
accurate prognostic indicator.
c) CT scan in ARDS often shows patchy areas of disease, rather than
uniform spread of disease throughout the lung.
d) Prognosis is affected by the ability of the patient’s lungs to perform gas
exchange at the onset of ARDS.
Answer D
Barash 5ed p1486
Recent evidence suggests that the PaO2: FiO2 ratio is not a risk factor for mortality in
ARDS, and mortality in ARDS and ALI appear to be similer

28) A pregnant woman at term is given 4 mg/kg of thiopentone for induction of


general anesthesia. Blood levels of thiopentone in the fetus remain low after
this maternal bolus, for all of the following reasons, EXCEPT ONE. Identify the
exception.

a) Metabolism of thiopentone by the fetal liver.


b) Dilution of thiopentone in the fetal blood volume.
c) Placental metabolism of thiopentone.

27
d) Inconsistencies in intervillous exchange surfaces at the placenta.
Answer C (can also Q 267 2003)
Barash states that the placenta does not metabolize any commonly used anesthetic
drug. While chestnut says that the placenta does not metabolize amide local
anesthetics, but it does (based on in vitro studies) metabolize ester local anesthetics

Miller

Drugs cross the placenta by three main processes: simple diffusion, active transport, or
pinocytosis. The extent of drug transfer is dependent on numerous factors, including
molecular weight, protein binding, degree of lipid solubility, maternal drug concentration,
and maternal and fetal pH. The Fick principle governs the rate of transfer of a drug

across a membrane: where Q/t = rate of


diffusion, K = diffusion coefficient, A = surface area of membrane available for
exchange, Cm - Cf = concentration gradient between the maternal and fetal
circulations, and D = thickness of the membrane.

Larger molecules are less likely to cross the placenta, but those with weights under 500
daltons will cross easily. Most drugs administered to a parturient in labor have low
molecular weights; therefore, they transfer easily to the fetus. Drugs with high lipid
solubility will also readily cross the placenta. Highly ionized substances with poor lipid
solubility (such as non-depolarizing muscle relaxants) have limited transfer.

Once a drug crosses the placenta, fetal pH and protein binding affect drug disposition.[44]
The degree of ionization greatly influences drug transfer because only non-ionized
portions of the drug can cross the placenta.

A phenomenon known as "ion trapping" may occur in a fetus with acidosis because
lower fetal pH favors ionization of basic local anesthetics (such as lidocaine) and may
explain the accumulation of drugs in a compromised fetus.[45]

Because of its unique characteristics, the fetal circulation can have a major impact on
drug distribution within the fetus. After drugs cross the placenta, they enter the fetal
circulation through the umbilical vein. The liver is the first fetal organ perfused by
umbilical venous blood, and substantial hepatic uptake has been described for many
drugs. However, approximately 40% of the umbilical venous blood bypasses the liver.[46]
Hepatic drug uptake in the fetus may protect the fetus from the deleterious effects of
certain drugs on the central nervous system.[47] Dilution of umbilical venous blood across
the foramen ovale and ductus arteriosus may also modify fetal drug distribution.

29) All explain the decrease in cardiac output with PEEP except:

a) Right ventricular dysfunction


b) Decrease right ventricular preload
c) Increase left ventricular afterload

28
d) Decrease left ventricular distensibility
Answer C
See Q14 2007

30) Concerning SIADH which is false:

a) Decrease plasma Na and osmolarity


b) Often in the post-operative period
c) 3-15 days after a close head injury and last 15-20 days
d) TX= Chlorpropamide
Answer D
See Q 15 2007

31) All are true about anticholinesterase drugs, EXCEPT?

a) Neostigmine bind reversibly and covalently


b) Edrophonium binds electrostatically
c) Edrophonium has a faster onset than Neostigmine
d) Edrophonium is better than Neostigmine to reverse a deep block
Answer D

32) All EXCEPT ONE of the following statements is associated with transient
radicular injury in ambulatory surgery. Indicate the exception:

a) Lidocaine 2%
b) Obesity
c) Spinal needle
d) Lithotomy
Answer C
Can see Q223 2004
BARASH CH 26 Golden paragraph
In a large epidemiologic study of nearly 2000 patients, Freedman et al characterized the
clinical picture of TRI.207 They found that patients receiving lidocaine were significantly
more likely to have TRI than patients receiving spinal tetracaine or bupivacaine, although
TRI did occur with these latter drugs as well. Additional risk factors for TRI included
surgery in the lithotomy position (with lidocaine, but not bupivacaine or tetracaine),
outpatient status, and obesity. Variables shown not to increase the risk of TRI included
lidocaine dose, type of spinal needle, addition of epinephrine to lidocaine, paresthesia,
hypotension, and blood-tinged CSF. In a separate study, Sakura et al has shown that
the addition of phenylephrine is a risk factor for TRI when 0.5% tetracaine is used for
spinal anesthesia.208

Miller chap 13
The risk of transient neurologic symptoms following spinal anesthesia was not
diminished by dilution of lidocaine from 5 to 1 to 2 percent

Barash Chap 26
Transient Radicular Irritation.In addition to cauda equina syndrome, transient radicular
irritation (TRI) has emerged as a concern after central neuraxial blockade. TRI is defined
as pain, dysesthesia, or both in the legs or buttocks after spinal anesthesia, and was first
proposed as a recognizable entity by Schneider et al.199 All local anesthetics have been

29
shown to cause TRI, although the risk appears to be greater with lidocaine than other
local anesthetics.

33) All are complications associated with meperidine EXCEPT ONE:

a) Increased biliary tract pressure


b) Diminished myocardial contractility
c) Tachycardia
d) Increased peripheral resistance
Answer D
P&P
MEPERIDINE:

Synthetic opioid agonist derived from phenylpiperidine.

Structurally, meperidine is similar to atropine, and it possesses a mild atropine like


antispasmodic effect.

There are several analogues of meperidine, including fentanyl, sufentanil, alfentanil and
phenoperidine.

PHARMACOKINETICS:

Meperidine is about one tenth as potent as morphine with 80 to 100 mg IM being


equivalent to about 10 mg of morphine IM.

In equal analgesic doses, meperidine produces as much sedation, euphoria, nausea,


vomiting, and depression of ventilation as does morphine.

Unlike morphine, meperidine is well absorbed from GIT, but nevertheless it is only about
one half as effective orally as when administered intramuscularly.

Meperidine also has well recognized weak local anesthetic properties. Compared to
morphine, fentanyl, and buprenorphine-injected perineurally, only meperidine alters
nerve conduction and produces analgesia. Because of its local anesthetic effects,
however spinal meperidine may also produce sensory and motor blockade as well as
sympatholytic effects, which are not seen with other opioids.

METABOLISM:

Metabolism of meperidine in the liver is extensive, with about 90% of the drug initially
undergoing demethylation to noremeperidine and hydrolysis to meperdinic acid.
Noremeperidine may subsequently be hydrolysed to normeperdic acid.
Urinary excretion of meperidine is PH dependant. For example, if the urinary PH is less
than 5, as much as 25% of the opioid is excreted unchanged.

Noremeperidine has elimination half –time of 15 to 40 hours and can be detected in the
urine for as long as 3 days following administration of meperidine.

This metabolite, in addition to having stimulant effects on the CNS, is about one half as
active as the parent compound as an analgesic.

30
Noremeperidine toxicity manifesting as Myoclonus and seizures is most likely during
prolonged administration of meperidine, especially to patients with impaired renal
function.

Noremeperidine may also be important in meperidine-induced delirium (confusion,


hallucination), which has been observed in patients receiving the drug for more than 3
days, corresponding to the accumulation of this active metabolite.

ELIMINATION HALF- TIME:

It is about 180 – 264 minutes. Since clearance of meperidine primarily depends on


hepatic metabolism, it is possible that large doses of opioid would saturate enzyme
system and result in prolonged elimination half time. Nevertheless, elimination half time
is not altered by doses of meperidine up to 5 mg/kg IV.

SIDE EFFECTS:

- In therapeutic doses, meperidine is associated with orthostatic hypotension. In


fact, hypotension after meperidine injection is more frequent and more profound
than after comparable doses of morphine. Orthostatic hypotension suggests that
meperidine, like morphine, interferes with compensatory sympathetic nervous
system reflexes, meperidine in contrast to morphine, rarely causes bradycardia
but instead may increase heart rate, reflecting its modest atropine-like qualities.
- Large doses of meperidine result in reductions in myocardial contractility that,
among opioids, is unique for this drug.
- Delirium and seizures when they occur presumably reflect accumulation of
noremeperidine, which has stimulating effects on the central nervous system.
- Meperidine readily impairs ventilation and may be even more of a ventilatory
depressant than morphine.
- Meperidine promptly crosses the placenta, and concentrations of meperidine in
umbilical cord blood at birth may exceed maternal plasma concentrations.
Nevertheless, meperidine produces less depression of ventilation in the neonate
than does morphine.
- Meperdine may produce less constipation and urinary retention than morphine.
- After equal analgesic doses, biliary tract spasm is less after meperidine injection
than after morphine injection but greater than that caused by codeine.
- Meperidine does not cause miosis but rather tend to cause mydriasis, reflecting
its modest atropine-like action. A dry mouth and increase in heart rate are further
evidence of the atropine-like effects of meperidine.
- The pattern of withdrawal symptoms after abrupt discontinuation of meperidine
differs from that of morphine in that there are few autonomic nervous system
effects. In addition, symptoms of withdrawal develop more rapidly and are of
shorter duration compared with those of morphine.

34) All are possible causes of pulmonary oedema following obstructive


tonsillectomy EXCEPT ONE:

a) Impaired lymphatic drainage


b) Increased hydrostatic pressure
c) Lost of grunting effect

31
d) Acute right ventricular failure
Answer D, RPT Q312 2004
Reference: Barash, Clinical Anesthesia, 4th Edition, Chapter 35, page 991-993.

Acute postoperative pulmonary edema is an infrequent but potentially life-threatening


complication encountered when airway obstruction is suddenly relieved. One proposed
mechanism is that during inspiration before adenotonsillectomy, the negative intrapleural
pressure that is generated causes an increase in venous return, enhancing pulmonary
blood volume. In the healthy child without airway obstruction, pleural pressure ranges
from –2.5 cm to –10.0 cm H2O during inspiration. Intrapleural pressure generated in the
child with airway obstruction can be as much as –30 cm H2O which, when transmitted to
the interstitial peribronchial and perivascular spaces, causes disruption of the capillary
walls of the pulmonary microvasculature. Concurrent with a negative transpulmonary
gradient is an increase in venous return to the right side of the heart, thus increasing
preload, which, in the setting of ―leaky capillaries,‖ facilitates transudation of fluid into the
alveolar space. To counterbalance this, positive intrapleural and alveolar pressure is
generated during exhalation, which decreases pulmonary venous return and blood
volume. This is similar to an expiratory ―grunt‖ mechanism in which the transpleural
pressures generated are similar to those present during a Valsalva maneuver.

The rapid relief of airway obstruction results in decreased airway pressure, an increase
in venous return, an increase in pulmonary hydrostatic pressure, hyperemia, and
finally pulmonary edema. The all-important counterbalance of the expiratory ―grunt‖
in limiting pulmonary venous return is lost when the obstruction is relieved.
Contributing factors are the increased volume load on both ventricles as well as
the inability of the pulmonary lymphatic system to remove acutely large amounts
of fluid. The anesthesiologist may attempt to prevent this situation during induction of
anesthesia by applying moderate amounts of continuous positive pressure to the airway,
thus allowing time for circulatory adaptation to take place. This physiologic sequence is
similar to that seen in patients with severe acute airway obstruction secondary to
epiglottitis or laryngospasm.

Negative-pressure pulmonary edema is signaled by the appearance of frothy pink fluid in


the endotracheal tube of an intubated patient or the presence of a decreased oxygen
saturation, wheezing, dyspnea, and increased respiratory rate in the immediate
postoperative period in a previously extubated patient. Mild cases may present with
minimal symptoms. The differential diagnosis of negative-pressure pulmonary
edema includes aspiration of gastric contents, adult respiratory distress
syndrome, congestive heart failure, volume overload, sepsis and anaphylaxis. A
chest radiograph illustrating diffuse, usually bilateral interstitial pulmonary infiltrates
combined with an appropriate clinical history will confirm the diagnosis.24

Treatment is usually supportive, with maintenance of a patent airway, oxygen


administration, and diuretic therapy in some cases. Endotracheal intubation and
mechanical ventilation with positive end-expiratory pressure may be necessary in severe
cases. Resolution is usually rapid and may occur within hours of surgery. Most cases
resolve without treatment within 24 hours. There is currently no reliable method for
predicting which children will experience this clinical syndrome after their airway
obstruction has been resolved.

32
It has been postulated that patients with fixed upper airway obstruction in whom the
negative inspiratory forces are largely compensated for by forced exhalation (Valsalva
maneuver) or the development of air trapping, which causes increased end-expiratory
pressures (auto-PEEP), are more likely to develop pulmonary edema after relief of the
obstruction. Laryngeal spasm during emergence from general anesthesia or airway
tumours account for 50% of reported cases in adults. In children younger than 10 years,
croup and epiglottitis are associated with more than half of reported cases.

35) Concerning the bleeding trauma patient. All are true EXCEPT:

a) All intravenous fluids should be heated to 37°C.


b) Tachycardia indicates a greater volume loss than hypotension.
c) Blood losses should be replaced with two to three times their volume of
crystalloids.
d) Most patients arrive at the hospital hypothermic.
Answer B
Barash 4ed
Table 48-3. ADVANCED TRAUMA LIFE SUPPORT CLASSIFICATION OF
HEMORRHAGIC SHOCK*
Class I Class II Class III Class IV
Blood loss (ml) 750 750–1500 1500–2000 2000
Blood loss (% blood 15 15–30 30–40 40
volume)
Pulse rate (per min) <100 >100 >120 140
Blood pressure Normal Normal Decreased Decreased
Pulse pressure Normal or Decreased Decreased Decreased
increased
Respiratory rate 14–20 20–30 30–40 <35
(breaths×min–1)
Urine output (ml×hr– 30 20–30 5–15 Negligible
1
)
Mental status Slightly Mildly Anxious and Confused,
anxious anxious confused lethargic
Fluid replacement Crystalloid Crystalloid Crystalloid + Crystalloid +
(3:1 rule)‡ blood blood
* For a 70-kg male patient; based on initial presentation.

† 3:1 rule is based on empiric observation that most patients require 300 ml
balanced electrolyte solution for each 100 ml blood loss. Without other clinical
and monitoring parameters, this guideline may result in excessive or inadequate

33
fluid resuscitation.

Adapted with permission from American College of Surgeons Committee on


Trauma: Shock. In American College of Surgeons (ed): Advanced Trauma Life
Support Course for Physicians, p 108. Chicago, American College of Surgeons,
1997.

36) Physiologic changes of pregnancy include all EXCEPT:

a) Increase in tidal volume


b) Decrease in minimal alveolar concentration
c) Decrease in fibrinogen
d) Increase in cardiac output
Answer C Miller's Anesthesia, 6th ed
PHYSIOLOGIC CHANGES OF PREGNANCY
Hematology and Coagulation

Change or
Stops
Parameter actual Starts
(postpartum)
measurement
Blood volume ↑ 45% 6 – 9 wks
Plasma volume ↑ 55% 6-12 weeks
RBC volume ↑ 30% Starts
increasing
after 16wks
(prior to this
it falls)
Hemoglobin 116 (15% ↓) 6 wks
Colloid osmotic ↓ by 5 mmHg 6 wks
pressure during
pregnancy
Plasma ↓ 25% During first
anticholinesterase trimester

Coagulation: Increased platelet turnover, increased clotting and increased fibrinolysis


→ pregnancy is a state of compensated intravascular coagulation (returns to to
prepregnancy state by 2 weeks postpartum)
 II and V unchanged
 XIII and XI decreased
 all other factors increased
Platelet aggregation in response to collagen, epinephrine, adenosine, diphosphate and
arachidonic acid is increased. Increased production offsets increased aggregation in most pts.
since plts.< 150 in 7.6%, and plts.< 100 in 1% , and unchanged in remainder.

Increase in most factor concentrations and a decrease in antithrombin III suggest activation of the
clotting system.

34
Increase in concentration of fibrin degradation products and increase in plasminogen
concentrations consistent with enhanced fibrinolysis.

Immune system:
 ↑ PMN level during pregnancy that persists until beyond 6 weeks postpartum BUT
PMN function is impaired during pregnancy

37) The differential diagnosis of seizures in a women in active labour include all
EXCEPT:

35
a) Abruptio placentae
b) Cerebrovascular accident
c) Eclampsia
d) Amniotic fluid embolism
Answer A

Obstetrics & Gynecology 2005;105:402-410

Amniotic fluid embolus

36
Clinical Presentation: AFE may occur in the 1st, 2nd or 3rd trimester, and may even occur
postpartum (usually within 5 minutes of delivery). Vast majority of patients that
experience AFE do so after the rupture of membranes (either spontaneous or artificial)

CNS
 seizure
Cardiovascular
 Hypotension (may have transient hypertension)
 Cardiovascular collapse (right and then left heart failure)

Respiratory
 Pulmonary edema or ARDS
 Hypoxemia and SOB
 Bronchospasm

Hematologic
 Coagulopathy (DIC)

Obstetric
 Fetal distress
 Uterine atony → postpartum hemorrhage

CJA;48:88-98 (2001)- Amniotic fluid embolus:


The presenting symptom in 51% of patients was respiratory distress. In the
remainder, the first indication of a problem was hypotension in 27%, a
coagulopathy in 12% and seizures in 10%. Clark et al, on the other hand, found that,
of those women presenting prior to delivery, 30% had seizures or seizure-like activity
while 27% complained of dyspnea. Fetal bradycardia (17%) and hypotension (13%) were
the next most common presenting features. Of the 13 patients who developed symptoms
after the delivery of the infant, seven (54%) presented with an isolated coagulopathy
manifested by postpartum hemorrhage.

38) With respect to Complex Regional Pain Syndrome involving the lower limb, all
are beneficial treatments EXCEPT

a) Epidural steroids
b) Oral prazosin
c) Lumbar plexus block
d) Physiotherapy
Answer A (There are references for all except A)

e-medicine:

o For cases involving an upper extremity, a stellate ganglion block may be


diagnostic and therapeutic. However, failure to relieve pain does not
eliminate the diagnosis.

o Differential blockade has been performed using Bier blocks with a variety
of agents, including local anesthetics, bretylium, steroids, ketorolac,
reserpine, and guanethidine.

37
o The rationale for selective neural blockade is to interrupt stimulation to the
sympathetic nervous system. Again, this is effective only in those whose
pain is sympathetically dependent.

Once the diagnosis is established, a number of treatment modalities that have been
proven helpful are available. The most effective treatment involves differential neural
blockade.

 Most patients, especially children, can benefit from physical therapy.

 Tricyclic antidepressants have been used to decrease burning. Gabapentin


(Neurontin) and systemic steroids have also been used with varying degrees of
success. Other agents include the alpha-1 adrenoreceptor antagonists terazosin
and phenoxybenzamine; the alpha-2 adrenoreceptor agonist clonidine; and the
NDMA receptor antagonists ketamine, dextromethorphan, and calcitonin. When
treatment reaches a plateau invasive interventions to be considered include
tunneled epidural catheters and neuroaugmentation.

From Miller:
TABLE 70-2 -- History, Signs, and Symptoms of Complex Regional Pain Syndrome
CRPS type 1 (RSD)
 Initial noxious event
 Distal aspect of extremity involvement
 Pain, allodynia, or hyperalgesia
 Pain disproportionate to injury
 Pain not limited to a single nerve
 Associated edema, changes in skin blood flow, abnormal pseudomotor
activity
What is not CRPS type 1 (RSD)
 Pain in an area of decreased sensation
 No cutaneous hyperalgesia or allodynia
 Pain only in a specific nerve distribution
 Proximal symptoms only
CRPS type 2 (causalgia)
 Burning pain
 Allodynia
 Hyperpathia
 Hand and foot involvement
 Sequela of injury to nerve or major branch
 More discrete localization than type 1 CRPS
 Most common nerves: median, sciatic, tibial, and ulnar

38
Management of reflex sympathetic dystrophy (Uptodate)
Medication Other
Topical capsaicin cream

Tricyclic antidepressants: Physical therapy


Amytriptyline 25 to 150 mg or
doxepin 5 to 20 mg Occupational therapy
Stage I
Ketorolac (30 to 60 mg) TENS unit

Guanethidine or phenoxybenzamine Sympathetic ganglion blocks


10 to 30 mg/day or prazosin 1 to 6
mg/day
Ketorolac IM (30 to 60 mg)

Prednisone 80 mg/day, decrease 10


Failure or
mg q 4 days to 20mg/day, then Physical therapy
stage II
taper slowly

Tricyclic antidepressants
Physical therapy
Failure or
Tricyclic antidepressants
stage III
Sympathetic ganglion blocks
Failure Referral to tertiary center, or if patient
has had good response to sympathetic
blocks consider sympathectomy

Miller
Sympathetic Nerve Blocks
Complex Regional Pain Syndrome

This entity involves the somatic, sympathetic, and somato-motor nervous systems, but
the exact explanation of the interaction of these symptoms has not been defined. CRPS
can follow minor trauma that does not result in demonstrable tissue damage (such as
strains, sprains, infections) (type I), or injury to major nerves (type II), viscera, or the
CNS.[112] The intensity of the symptoms do not correlate with the severity of the inciting
trauma and the symptoms seem to spread within the nervous system and to the distal
and/or contralateral limb, thus not being confined to a single dermatome.[113] The concept
that patients with CRPS progress through stages of the disease has been repeatedly
challenged, based on clinical observations. Bruehl and co-workers clarified this issue
somewhat by presenting the view that CRPS has distinct subtypes, for which gradations
of targeted therapy are practical.[114]

Clinical signs and symptoms include burning pain, allodynia or hyperalgesia, edema,
changes in blood flow (vasomotor) and sweating (sudomotor) in the affected limb(s) and
muscle-joint changes, tremors, weakness, and dystonia.[115] Criteria for the diagnosis of
CRPS have been published, but no system has achieved the desired uniformity of
application or the necessary sensitivity and specificity.[116] Also, because specific

39
diagnostic tests are lacking, the diagnosis is made by extracting findings from the history
and physical examination.[115]

Treating the cause of the CRPS does not constitute its management. A common
treatment for CPRS has been to interrupt the alleged somatic-sympathetic interaction by
performing nerve blocks of the SNS (also see Chapter 44 ), although the rationale for
this is challenged by consideration of the contemporary mechanisms of sympathetically
mediated pain.[75][76][117] The therapeutic or diagnostic effect of the specific technique of
sympathetic blockade should not be confounded with sensory or motor block.
Cervicothoracic (i.e., stellate) ganglion block is performed with up to 10 mL of local
anesthetic at the C6 level with an anterior paratracheal insertion site.[75][76] The admitted
unreliability of assessing the adequacy of the subsequent block clinically challenges the
conventional wisdom that only a positive response is necessary for this block to be
considered diagnostic of CRPS.[117][118] Lumbar sympathetic blocks are performed at the
L2, L3, and L4 levels or with a single needle at L2 or L3.[75][76][119] A 22-gauge needle is
advanced, usually under fluoroscopic guidance, to the anterolateral border of the chosen
vertebral body level (a test injection of x-ray dye will confirm proper needle placement),
and 10 to 15 mL of local anesthetic are injected.

In both of these types of sympathetic blockade, the regional anatomy involved in the
technique predicts the potential side effects and complications from errant needle
placement or drug administration. These possibilities are very important if neurolytic
agents (e.g., 6% to 10% aqueous phenol or 50% to 100% absolute alcohol) are used for
lumbar sympathetic block. Genitofemoral neuralgia has been a reported complication in
8% to 10% of patients after neurolytic blockade of the lumbar sympathetic chain.[119][120]

As with the exact utility of SNS blocks in CRPS, the use of chemical sympathectomy for
neuropathic pain has been challenged.[117][118][120] Based upon a Cochrane database
review, chemical sympathectomy was shown to have only a temporary effect, and that
on the cutaneous allodynia component of the patient's clinical presentation.[117]

With serial local anesthetic blocks, a staircase pattern of improvement is sought,


meaning that pain is reduced more and for a longer period of time with each block.
Progress with the all-important rehabilitative physical therapy is enhanced when pain is
reduced and joint range of motion restored. Improved functional recovery is the ultimate
goal of treatment. If benefits are not progressive but definitive with each block (i.e.,
temporary effects are obtained with serial blocks), consideration may be given to
radiofrequency ablation or thoracoscopic/surgical sympathectomy.[76][115]

The most contemporary treatment offered for patients with enduring CRPS is spinal cord
stimulation (see earlier). This popular treatment has not yet passed rigorous scrutiny as
to its utility in such patients, even though it is touted to inhibit A-β fiber-mediated
hyperexcitability of dorsal horn neurons, reduce SNS outflow, and increase peripheral
blood flow.[121] Van Hilten and associates reported that the administration of intrathecal
baclofen to patients with CRPS-related dystonia showed benefit in the upper extremity
but not the lower extremity.[122] Rauck and co-workers have shown that epidural clonidine
infusion may be beneficial in selected patients.[123] Suresh and colleagues reexamined
the use of intravenous regional techniques, their application using lidocaine and
ketorolac proving to be at least moderately effective in children and adolescents.[124]

40
Blockade of the celiac plexus, another technique involving the SNS, is notable for its
continued use in patients with intractable pain due to cancer of the pancreas or other
upper abdominal viscera.[76][80][125][126] Successful block of the celiac plexus denervates
the abdominal organs from the gastroesophageal junction to the splenic flexure of the
large intestine. The exact utility of this technique in patients with chronic, non-cancer
pain has yet to be determined. Its consideration in such patients merits a serious review
of the risks and benefits, especially when the use of neurolytic agents is contemplated.

39) When comparing low molecular weight heparin to regular unfractionated


heparin, LMWH has all the following characteristics EXCEPT

a) Longer half life


b) Decreased incidence of autoimmune thrombocytopenia
c) Little value in checking PTT level
d) Less interaction with anti-platelet agents
D , repeat Q-196 (2005) All the others are true

196. Which is not one of the advantages of low molecular weight heparin compared to
unfractionated ( ? copy error in French version) heparin ?
1. long half life
2. No need to measure PTT
3. Less formation of antiplatelet antibodies
4. Less interaction with antiplatelet agents.

Answer : 4
Ref. : Warkentin TE. HIT and the Anesthesiologist. Can J Anesth 2002. 49(6) : S36.
Weitz JI. Low molecular weight heparins. New Eng J Med 1997. 337(10) :688.
1 . Half life is 2-4 times longer than UFH but in the OR this can be a disadvantage if
rapid normalization of coagulation is desired because LMWH is also not reversible
2. True. The PTT is not likely to be affected as increased PTT with UFH represents
primarily thrombin inhibition whereas LMWH inhibits Xa predominantly. Monitoring for
patients on LMWH is only required for extremes of weight and those with renal failure.
3. True. The incidence of HIT is lower with LMWH than UFH.
4. True?. LMWH interferes less than UFH with platelet function (NEJM article). A
literature search of LMWH and antiplatelet agents turned up primary literature from
cardiology that in acute MI LMWH may be safer than UFH with G2b3a agents.

40) Propofol’s main hemodynamic effect in a healthy person?

a) Decreased HR
b) Decreased SVR
c) Increased cardiac output
d) ?
B
Miller; Chapter 10
Hemodynamic changes after induction of anesthesia with nonbarbiturate
hypnotics

41
Etomidate
Diazepam Droperidol Ketamine Lorazepam Midazolam Propofol
*
HR -9%–13% Unchanged -5%–10% 0–59% Unchanged -14%–12% -10%–10%
MBP 0%–19% 0%–10% 0–17% 0–40% -7%–20% -12%–26% -10%–40%
SVR -22%–13% -5%–15% -10%–14% 0–33% -10%–35% 0–20% -15%–25%
PAP 0%–10% Unchanged -9%–8% 44%–47% — Unchanged 0–10%
PVR 0–19% Unchanged -18%–6% 0–33% Unchanged Unchanged 0–10%
PAO Unchanged 25%–50% Unchanged Unchanged — 0–25% Unchanged
RAP Unchanged Unchanged Unchanged 15%–33% Unchanged Unchanged 0–10%
CI Unchanged Unchanged -20%–14% 0–42% 0–16% 0–25% -10%–30%
SV 0–8% 0–10% 0–20% 0–21% Unchanged 0–18% -10%–25%
LVSWI 0–36% Unchanged 0–33% 0–27% — -28%–42% -10%–20%
dP/dt Unchanged — 0–18% Unchanged — 0–12% Decreased

41) A primigravida in 2nd trimester complains of SOBOE and has a midsystolic


murmur at the sternal border with a fixed widely split S2 heart sound and a
parasternal heave. Her CXR shows fullness of the pulmonary arteries and
vascular redistribution. The most likely cause is:

a) ASD
b) IHSS
c) MVP
d) Pulmonic Stenosis
A , Repeat Q-71 (2002)
Snider: Anesthesia for Obstetrics 4th Ed, Chapter 26, pg.460

ASD - most common congenital heart lesion


- symptoms, if they arise, occur in 4th or 5th decades
- Signs and Symptoms:
- arrhythmias
- PH
- RV failure
- LV failure
- fixed expiratory splitting of second heart sound
- systolic ejection murmur at LSB
- intensity of murmur correlates with degree of L-R shunt
- ECG shows R axis deviation
- CXR shows RV enlargement, pulmonary artery
prominence, increased pulmonary vascular markings

Mitral Insufficiency
- second most common valvular defect in pregnancy
- leads to CHF
- complications occur late in life so most tolerate pregnancy well

42
- 5.5% incidence of pulmonary congestion
- 4.3% incidence of atrial tachycardia
- 2.8% incidence of PE
- 8.5% incidence of infective endocarditis

- Signs and Symptoms: - LV failure


- pansystolic blowing murmur, loudest at apex,referred to L
axilla
- a.fib in 1/3 of patients
- late signs: pulmonary edema, PH, RV enlargement

MVP (mitral valve prolapse)


- most common congenital valvular lesion (5-10% of general population)
- most prevalent in young females
- 85% of people are asymptomatic
- 15% go on to develop mitral regurgitation over 10-15 year period
- pregnant females can have MVP without insufficiency, and tolerate pregnancy
well
(complications only arise if mitral insufficiency is present)

- Signs and Symptoms: - anxiety, palpitations, dyspnea, chest discomfort


- lightheadedness, emotional disturbances
- orthostatic hypotension
- marfanoid features or thoracic skeletal abnormalities
- mid to late systolic click (after upstroke of carotid pulse)
- may have mid to late crescendo systolic murmur
- in severe MVP, click is early and murmur is holosystolic

Aortic Stenosis
- patients become symptomatic in 5th or 6th decade
- when symptoms appear, 50% mortality within 5 years
- asymptomatic pregnant patients are not at higher risk
- symptomatic AS markedly increases maternal and fetal morbidity and mortality

- Signs and Symptoms: - severe AS: CHF, syncope, angina


- systolic ejection murmur loudest at 2nd right intercostals
space
- murmur radiates to neck
- ECG shows LVH and occasionally LBBB
- CXR shows LV enlargement, poststenotic aortic dilatation

Aortic Insufficiency
- 7-10 year latent period after rheumatic fever
- asymptomatic for an additional 7-10 years
- symptoms appear in 4th or 5th decade, so pregnant patients often tolerate
pregnancy well

- Signs and Symptoms: - widened pulse pressure


- decreased DBP

43
- bounding peripheral pulses
- early blowing diastolic murmur along L sternal border
- duration of murmur correlates with severity of disease

42) Which drug should you avoid in a patient with porphyria?

a) Atropine
b) Lidocaine
c) STP
d) Isoflurane
Answer C
Barash Table 20-5. DRUGS KNOWN TO PRECIPITATE PORPHYRIA

Sedatives
Barbiturates
Hypnotics such as chlordiazepoxide, glutethimide, diazepam
Analgesics
Pentazocaine, antipyrine, aminopyridine
Lidocaine
Anticonvulsants
Phenytoin, methsuximide
Antibiotics
Sulfonamides, chloramphenicol
Steroids
Estrogens, progesterones
Hypoglycemic sulfonylureas
Tolbutamide, chlorpropramide
Toxins
Lead, ethanol
Miscellaneous
Ergot preparations
Amphetamines
Methyldopa

Miller,6th ed; Ch 19, P 749

Porphyrias are uncommon disorders of heme synthesis (porphyrin metabolism).


Usually, the disorder remains subclinical until an endogenous or exogenous stress
triggers a porphyric crisis.

Clinical features of acute porphyrias include recurrent, dramatic, and potentially


fatal neurologic reactions. Most patients develop abdominal pain (90%) and dark
urine (80%). Neurotoxicity may result from increased plasma and tissue levels of
porphyrin precursors (particularly ALA and PBG), which have chemical structures
resembling the inhibitory neurotransmitter γ-aminobutyric acid (GABA). The most
common of the acute porphyrias is acute intermittent porphyria (AIP). Its
prevalence is about 1 in 10,000 in the general population and may be as high as 1
in 500 among patients with psychiatric disorders. Women are five times more likely
than men to have AIP. Among the triggers of porphyric crises are sex hormones,
glucocorticoids, cigarette smoking, and medications,including certain anesthetic
agents (e.g., barbiturates, etomidate, enflurane, pentazocine).Barbiturates and
other inducers of cytochrome P-450 (CYP) stimulate the synthesis of cytochrome
protein; incorporating heme into newly forming hemoproteins causes the
intracellular heme concentration to decline.The decrease in heme reduces the
inhibitory influence on ALA synthetase, which is rate-limiting, and thus porphyrin
(heme) synthesis speeds up.

44
Acute attacks are characterized by severe abdominal pain (nausea, vomiting all
secondary to ANS dysfunction), autonomic nervous system instability (tachycardia,
hypertension, or less commonly hypotension), electrolyte disturbances (due to vomiting),
and neuropsychiatric manifestations (skeletal muscle paralysis, upper motor neuron
lesions, seizures, psychiatric disturbances). NOTE: many patients with the disorder
have never had any symptoms.

Anesthetic management consists of avoiding triggering agents, and minimizing


response to stressful stimuli. Glucose containing solutions may be useful in avoiding
attacks. There is no specific prophylactic therapy. Regional anesthesia is
acceptable, remembering that it may confuse neurological assessment in case
of an attack.

An attack is treated by removing causative factors. Adequate hydration and


carbohydrates are necessary. Opioids are used for pain, nausea and vomiting are
treated with anitemetics. Tachycardia and hypertension are treated with b-blockers.
Benzodiazepines are used for seizures. Hematin 3-4 mg/kg IV over 20 minutes is a
specific therapy for acute attacks, presumably by increasing the pool of heme.
Somatostatin decreases the rate of formation of the initial enzyme in the heme
pathway (ALA synthetase), and may be useful in conjunction with plasmapheresis.

43) Patient with an unstable C6 injury without symptoms is at greatest risk for
further injury when:

a) During intubation
b) During positioning
c) During surgical manipulation
d) Upon emergence

UWO version:
Patient coming for posterior fusion of unstable C6 fracture. What time period is highest risk
for neurologic injury:
a. Intubation
b. Positioning
c. Emergence/extubation
d. Surgical manipulation

B ?
Canadian Journal of Anesthesia 49:733-744 (2002) (Airway management after upper
cervical spine injury: what have we learned?)
DURING LARYNGOSCOPE BLADE INSERTION, MINIMAL DISPLACEMENT OF THE SKULL BASE AND
CERVICAL VERTEBRAL BODIES WAS OBSERVED. HOWEVER, ELEVATION OF THE
LARYNGOSCOPE BLADE TO ACHIEVE VISUALIZATION OF THE LARYNX CREATED SUPERIOR
ROTATION OF THE OCCIPUT AND CL IN THE SAGITTAL PLANE, AND MILD INFERIOR ROTATION OF
C3–C5. C2 MAINTAINED NEAR-NEUTRAL POSTURE. THIS PATTERN OF DISPLACEMENT
RESULTED IN EXTENSION AT EACH MOTION SEGMENT, WITH THE MOST SIGNIFICANT MOTION
PRODUCED AT THE ATLANTO-OCCIPITAL AND ATLANTO-AXIAL JOINTS (MEAN 6.8° AND 4.7°,
RESPECTIVELY;).

Miller,6th ed :

45
Cervical spinal surgery may be complicated by injury to the spinal cord during surgery,
leading to postoperative quadriplegia with respiratory impairment.[8] For this reason,
somatosensory evoked potentials (SSEPs) are often used during these procedures to
monitor surgical manipulations.

44) When a drug is given by constant rate IV infusion, the time to reach 90% of
steady-state plasma concentration is dependent mainly on:

a) Plasma concentration
b) Protein binding
c) Rate of elimination
d) Volume of distribution
Answer C,
Q62 (2001)
62. The steady state during a continuous infusion:
a) Is mainly dependent on the rate of clearance and infusion rate
b) loading dose depends on volume or distribution and plasma concentration
c) A large initial dose can cause initial adverse effects
d) it takes 5 half lives to reach steady state concentrations
e) steady state concentration depends mainly on half life

Elimination half-time is the time necessary for the plasma concentration of a drug to
decrease to 50% during the elimination phase. Elimination half-time of a drug is
directly proportional to its Vd and inversely proportional to its clearance. For this
reason, renal or hepatic disease that alters Vd and/or clearance will alter the

46
elimination half-time. Conversely, elimination half-time is independent of the dose of
drug administered.

Elimination half-life, in contrast to elimination half-time, defines the time necessary to


eliminate 50% of the drug from the body after its rapid IV injection. Elimination half-
time and elimination half-life are not equal when the decrease in the drug‘s plasma
concentration does not parallel its elimination from the body. The amount of drug
remaining in the body is related to the number of elimination half-times that have
elapsed. For example, if 50% of a drug is eliminated in 10 minutes, another 10
minutes will be needed for elimination of one-half of the remaining drug. About five
elimination half-times are required for nearly total (96.9%) elimination of drug from
the body. For this reason, drug accumulation is predictable if dosing intervals are
less than this period of time. Drug accumulation continues until the rate of its
elimination equals the rate of its administration. As with drug elimination, the time
necessary for a drug to achieve a steady-state plasma concentration (Cpss) with
intermittent dosing is about five elimination half-times.

45) A 12-year-old male with bladder extrophy is having a repeat repair. 30 minutes
after the start of the anesthetic he develops increased peak inspired airway
pressures, has hypotension and tachycardia. The first management priority is:

a) Salbutamol
b) Methylprednisolone
c) Insert a chest tube
d) Epinephrine
D

46) When is lung compliance (elastic recoil of lung tissue) in health the greatest?

a) 2 year
b) 8 years
c) 18 years
d) 60 years
D

47) From age 40yrs to 80yrs, MAC decreases by:

a) 10%
b) 20%
c) 30%
d) 40%
C,
Barash P1224
The effect of nervous system aging on requirements for general anesthesia is less
controversial. Between young adulthood and the geriatric era, relative minimum
alveolar concentration (MAC) values for the newer inhalational agents decline
progressively by as much as 30%, the same decrement seen with older
anesthetics. The mechanisms producing age-related increases in sensitivity to
anesthetic agents remain unknown, but the consistency of this phenomenon for
anesthetic agents with markedly different chemical characteristics suggests that it is
the result of a fundamental neurophysiologic process such as loss of neuronal mass

47
or cerebral blood flow. The decline in anesthetic requirements also parallels the
reduction in brain neurotransmitter activity that occurs with increasing age, however,
and a direct relationship between brain catecholamine levels and MAC is well
established for patients treated with drugs that deplete or to enhance these
neurotransmitters. Whatever the mechanism of age-related increase in anesthetic
potency, changes in anesthetic requirement may be a clinically relevant indicator of
nervous system functional reserve.

48) Which would increase the alveolar-arterial gradient for CO2

a) Endobronchial intubation
b) Cardiovascular collapse with continued mechanical ventilation
c) Alveolar hypoventilation
d) Malignant hyperthermia
B
Barash 5th ed, P 670
V/Q maldistribution is a commen cause of increased Paco2-PAco2 gradient. Dead space
is the extreme example of V/Q mismatch, where a complete absence perfusion in the
presence of adequate alveolar ventilation occur, common causes include; embolic
phenomena (thrombus, fat, air, amniotic fluid), Hypoperfusion states with reduced
pulmonary blood flow, and COPD.

49) Which drug would decrease the seizure duration in ECT?

a) Etomidate
b) Ketamine
c) Remifentanil
d) Esmolol
D , miller 6th ed ,Ch 69;
Many intravenous anesthetics have been used to induce anesthesia for ECT, including
methohexital, thiopental, propofol, and ketamine. Methohexital (0.75 to 1.0 mg/kg) is the
most commonly used drug for ECT anesthesia and is considered the "gold standard."
Propofol (0.75 mg/kg) was found to reduce seizure duration, which was believed to
decrease the efficacy of ECT. However, more recent studies have demonstrated no
difference in outcome with propofol versus methohexital. Administration of methohexital,
as well as propofol, is associated with pain on injection, which may be poorly tolerated
by psychiatrically fragile patients. The use of thiopental (1.5 to 2.5 mg/kg) avoids pain on
injection, but it is associated with more hypertension and tachycardia than propofol is.
Etomidate may prolong seizures and recovery, but prolongation of the seizure may be
useful in patients in whom seizure duration is deemed too short with other agents.
Benzodiazepines have anticonvulsant activity and should be avoided before ECT.
Ketamine has been demonstrated to not increase seizure length or produce excessive
postprocedural agitation. Given the hemodynamic response expected after ECT,
ketamine would seem to be a less desirable agent.

Prophylactic medications have been advocated to avoid various side effects of ECT.
Transient asystole is rare during ECT, but it may be prevented with anticholinergic
pretreatment. Glycopyrrolate is preferred over atropine because glycopyrrolate has no
central anticholinergic side effects. In addition, glycopyrrolate is an effective
antisialagogue. Both esmolol and labetalol have been successfully used to control
hypertension and tachycardia after ECT. Some evidence has shown that esmolol

48
reduces seizure duration. Routine treatment with esmolol or labetalol is not
recommended because the hypertension and tachycardia are usually self-limited, as
are premature ventricular contractions. Should treatment be necessary, these drugs
can be administered immediately after the stimulus.

CJA, Vol 38, 204-209, 1991 Comparison of two esmolol bolus doses on the
haemodynamic response and seizure duration during electroconvulsive therapy;

Twelve ASA physical status I-III patients were enrolled in a double-blind, prospective,
randomized, three-way, within-patient crossover study designed to determine the effect
of two standard esmolol bolus doses (100 and 200 mg) on the haemodynamic
response and seizure duration during electro-convulsive therapy (ECT). Treatment
with esmolol 200 mg resulted in a significantly shorter mean seizure duration than with
placebo. As the 200 mg dose caused a shorter seizure duration and the
haemodynamic effects of 100 mg and 200 mg doses were similar, it was concluded
that the 100 mg esmolol bolus dose was the better dose for ECT.

50) You correctly place a left DLT for left upper lobectomy. Midway through the
case (with the patient in right lateral decubitus & on one lung ventilation) the
airway pressure rises and the tidal volumes decrease to 100mL. How do you
proceed?

a) Suction the non-dependant lung


b) Pull back the DLT
c) Deflate the bronchial cuff
d) Apply CPAP to non-dependent lung
I would go w/ C
Miller,6th ed,Ch 49 ;

Recommended Combined Conventional and Differential Lung Management of


One-Lung Ventilation

49
If hypoxemia is present after this initial conventional approach, two major causes of
hypoxemia, namely, malposition of the DLT and poor hemodynamic status, must be
ruled out. Proper tube position should be confirmed with fiberoptic bronchoscopy. If the
DLT is correctly positioned and the patient's hemodynamic status is satisfactory, simple
tidal volume and respiratory rate adjustments should be made. For example, if the tidal
ventilation is thought to be too high, it should be decreased, and if the tidal ventilation is
thought to be too low, it should be increased. If these simple maneuvers do not quickly
resolve the problem, the studies of selective nondependent lung CPAP and differential
lung PEEP dictate that the next treatment should be to apply 5 to 10 cm H2O of CPAP to
the nondependent lung. Nondependent lung CPAP should be applied during the
deflation phase of a large-tidal volume breath to overcome critical opening pressures in
the atelectatic lung. If oxygenation does not improve with nondependent lung CPAP (as
it does in the large majority of cases), 5 to 10 cm H2O of PEEP should then be applied to
the ventilated dependent lung. If dependent lung PEEP does not improve oxygenation,
nondependent lung CPAP should be increased to 10 to 15 cm H2O while the dependent
lung is maintained at 5 to 10 cm H2O of PEEP. If arterial oxygenation is still not
satisfactory, the nondependent lung CPAP level should be matched with an equal
amount of dependent lung PEEP. In this way, a differential lung PEEP/CPAP search for
maximal compliance and minimal right-to-left transpulmonary shunt is made in an
attempt to find the optimal end-expiratory pressure for each lung and the patient as a
whole.

Arif (from barash)

Third, the double-lumen tube may not be inserted far enough, leaving the bronchial
lumen opening above the carina. In this position, good breath sounds are heard
bilaterally when ventilating through the bronchial lumen, but no breath sounds are
audible when ventilating through the tracheal lumen because the inflated bronchial cuff
obstructs gas flow arising from the tracheal lumen. The cuff should be deflated and the
double-lumen tube rotated and advanced into the desired mainstem bronchus.

51) Which would be seen with a potassium <2.5mEq/L


a) Shortened QT
b) Increased sensitivity to depolarizing muscle relaxants
c) Decreased dose requirements for Neostigmine for reversing non-
depolarizing muscle relaxants
d) Decreased length of refractory period in Purkinje cells
Answer D most likely
Kelly‘s
Prolongation of the QT interval, largely caused by prolongation of phase 2 of the action
potential, can be recorded in the presence of hypocalcemia, hypertension,
antiarrhythmic drugs, alkalosis, myocardial ischemia, cerebrovascular accident, and
bradycardia, after resuscitation, and with other less common conditions. The common

50
causes of QT-interval shortening include hypercalcemia, hyperkalemia, acidosis, and
digitalis therapy.

Hypokalemia is manifested by depression of the ST segment, decrease in amplitude,


and inversion of the T wave, which merges with a prominent U wave (Fig. 84.15). The
changes are best seen in leads V2 to V4. The duration of the QT interval is normal;
the QU interval is prolonged.

Maybe D
The genesis of which is unclear. There are two prevailing concepts of the generation of
the U wave. One suggests that the U wave represents delayed repolarization of
Purkinje fibers, and the other states that the U wave is secondary to a diastolic
myocardial event.

Not B: Acute hypokalemia increases the resting membrane potential ( makes more
negative ), NDMR are potentiated while depolarizing relaxants are antagonized. The
reverse is true for in Hyperkalemia

Not C: P&P
Antagonism of neuromuscular blockade by anticholinesterase drugs may be inhibited or
even prevented by (a) certain antibiotics, (b) hypothermia, (c) respiratory acidosis
associated with a PaCO2 of >50 mm Hg, or (d) hypokalemia and metabolic acidosis

TABLE 9-3. Recommended doses of neostigmine or edrophonium according to


responses to train-of-four (TOF) stimulation
TOF visible twitches Fade Anticholinesterase drug Dose (mg/kg)
None* — — —
<2 ++++ Neostigmine 0.07
3–4 +++ Neostigmine 0.04
4 ++ Edrophonium 0.5
4 ± Edrophonium 0.25

Long QT can be induced by drugs such as quinidine, procainamide, disopyramide,


and phenothiazines and also by bradycardia, hypokalemia, hypomagnesemia, or
acute ischemia or infarction

The most important effects of hypokalemia (usually at values less than 3 mEq per L) are
on the myocardium. Typical ECG changes result, including ST-segment depression,
decreased amplitude or inversion of the T wave, and increased height of the U wave
(more than 1 mm). With increases in severity, hypokalemia may lead to increased P-
wave amplitude, prolongation of the PR interval, and widening of the QRS complex.
Atrioventricular block and supraventricular and ventricular tachycardia, including
ventricular fibrillation, can also occur, particularly in the setting of acute myocardial
infarction. Hypokalemia also increases the incidence of digitalis toxicity.

51
Hypokalemia may lead to vague malaise, muscular weakness (usually proximal and in
the lower extremities), and muscle cramps. Constipation and even intestinal ileus may
occur from impaired smooth muscle function. Severe hypokalemia (less than 2 mEq per
L) can cause marked weakness and life-threatening paralysis (respiratory arrest).
Rhabdomyolysis may result.

Miller
Cardiovascular manifestations of hypokalemia include autonomic neuropathy, which
results in orthostatic hypotension and decreased sympathetic reserve; impaired
myocardial contractility; and electrical conduction abnormalities, which can result in
sinus tachycardia, atrial and ventricular arrhythmias, and disturbances in intraventricular
conduction that can progress to ventricular fibrillation. No studies demonstrate increased
morbidity or mortality for patients undergoing anesthesia with a potassium level of at
least 2.6 mEq/L.

52) Which would cause PCWP to underestimate LVEDP?


a) Acute AI
b) Mitral stenosis
c) Atrial myxoma
d) PAC in West lung zone I

A
Miller Ch 32
Table 32-12 -- Underestimation of left ventricular end-diastolic pressure
Condition Site of Discrepancy Cause of Discrepancy
Diastolic dysfunction Mean LAP < LVEDP Increased end-diastolic a wave
LAP a wave < Mitral valve closure before end-
Aortic regurgitation
LVEDP diastole
Bidirectional runoff for pulmonary
Pulmonic regurgitation PADP < LVEDP
artery flow
Right bundle branch
PADP < LVEDP Delayed pulmonic valve opening
block
PAWP < LAP or Obstruction of pulmonary blood
Postpneumonectomy
LVEDP flow
Atlas of Cardiovascular Monitoring. New York, Churchill Livingstone, 1998, p 59.

Table 32-13 -- Overestimation of left ventricular end-diastolic pressure


Condition Site of Discrepancy Cause of Discrepancy
Positive end-expiratory Mean PAWP > mean Creation of lung zone 1 or 2 or
pressure LAP pericardial pressure changes
Pulmonary arterial PADP > mean
Increased pulmonary vascular resistance
hypertension PAWP
Pulmonary veno- Mean PAWP > mean Obstruction to flow in large pulmonary
occlusive disease LAP veins

52
Condition Site of Discrepancy Cause of Discrepancy
Obstruction to flow across the mitral
Mitral stenosis Mean LAP > LVEDP
valve
Retrograde systolic v wave raises mean
Mitral regurgitation Mean LAP > LVEDP
atrial pressure
Ventricular septal Antegrade systolic v wave raises mean
Mean LAP > LVEDP
defect atrial pressure
PADP > mean LAP > Short diastole creates pulmonary
Tachycardia
LVEDP vascular and mitral valve gradients

53) When is the period of greatest risk In HELLP syndrome:


a) 0-4 hours postpartum
b) 4-12 hours postpartum
c) 24-48 hours postpartum
d) At delivery
C
http://www.aafp.org/afp/990901ap/829.html
My Notes from Chestnut
Its clinical and pathological manifestations result from an unknown insult that leads to
intravascular platelet activation and microvascular endothelial damage. Haemolysis is
due to microangiopathic haemolytic anaemia
► Pts with HELLP have a higher incidence of maternal complications
 Stroke
 DIC
 Placental abruption
 Acute renal failure
 CNS or hepatic haemorrhage
► Most cases develop preterm, but 20% present postpurtum, up to 6 days post
partum
► Most pts develop HELLP before delivery, it reaches its peak intensity 24 to
48hrs postpartum
► Haemolysis typically ceases within 48hrs of delivery and the platelet count
increases to >100 within 72hrs
► A number of studies noted better maternal outcome with the administration of
dexamethazone 10mg IV q12hrs until disease remission is noted
► Can also platelet counts
► Delivery represents the only definitive Tx

54) Which of the following herbals can impair platelet function

a) St. johns
b) Kava
c) Valerian
d) Ginko Baloba

D Any one starts with G

53
Important
Herbs (Common Preoperative
Pharmacologic Perioperative Concerns
Names) Discontinuation
Effects
Activation of cell-
Echinacea Allergic reactions No data
mediated immunity
Purple coneflower Decreased effectiveness of
root immunosuppressants
Potential for immunosuppression
with long-term use
Increased heart rate
and blood pressure
Risk of myocardial ischemia and
through direct and At least 24 hours
Ephedra (ma huang) stroke from tachycardia and
indirect before surgery
hypertension
sympathomimetic
effects
Ventricular arrhythmias with
halothane
Long-term use depletes
endogenous catecholamines and
may cause intraoperative
hemodynamic instability
Life-threatening interaction with
MAO inhibitors
May increase risk of bleeding,
Inhibition of platelet
especially when combined with At least 7 days
Garlic (ajo) aggregation (may be
other medications that inhibit before surgery
irreversible)
platelet aggregation
Increased fibrinolysis
Equivocal
antihypertensive
activity
May increase risk of bleeding,
Ginkgo (duck-foot
Inhibition of platelet- especially when combined with At least 36 hours
tree, maidenhair tree,
activating factor other medications that inhibit before surgery
silver apricot)
platelet aggregation
Ginseng (American
ginseng, Asian
Lowers blood At least 7 days
ginseng, Chinese Hypoglycemia
glucose before surgery
ginseng, Korean
ginseng)
Inhibition of platelet
aggregation (may be May increase risk of bleeding
irreversible)
Increased PT/PTT in May decrease anticoagulant effect
animals of warfarin

54
Important
Herbs (Common Preoperative
Pharmacologic Perioperative Concerns
Names) Discontinuation
Effects
Many other diverse
effects
Kava (awa,
May increase sedative effect of At least 24 hours
intoxicating pepper, Sedation
anesthetics before surgery
kawa)
Anxiolysis
Ability to increase anesthetic
requirements with long-term use
unstudied
Saw palmetto (dwarf Inhibition of 5-α
May increase risk of bleeding No data
palm, sabal) reductase
Inhibition of
cyclooxygenase
Induction of cytochrome P450
St. John's wort enzymes, affecting cyclosporin,
Inhibition of
(amber, goat weed, warfarin, steroids, and protease At least 5 days
neurotransmitter
hardhay, hypericum, inhibitors and possibly affecting before surgery
reuptake
Klamath weed) benzodiazepines, calcium channel
blockers, and many other drugs
MAO inhibition is
unlikely
Decreased serum digoxin levels
Delayed emergence
Valerian (all heal,
May increase sedative effect of
garden heliotrope, Sedation No data
anesthetics
vandal root)
Benzodiazepine-like acute
withdrawal
May increase anesthetic
requirements with long-term use
MAO, monoamine oxidase; PT, prothrombin time; PTT, partial thromboplastin time.

55) Following inguinal hernia a patient develops chronic ilioinguinal nerve pain.
Which of the following would be seen on physical exam?
a) Pain to light touch in the groin
b) Problems with hip flexion
c) Glossy skin changes in groin
d) Lack of sensation to lateral thigh

A
The ilioinguinal nerve supplies the skin of the perineum and adjoining portion of the inner
thigh.

55
THE MOUNT SINAI JOURNAL OF MEDICINE Vol. 73 No. 4 July 2006:
Ilioinguinal neuralgia (T12 –L1) is characterized by burning pain through the lower
abdomen radiating to the superior medial thigh and to the scrotum or labia majora. A
positive Tinel‘s sign may be present, in which the pain may be reproduced by tapping
the affected area or extending the hip and thigh. Similarly, genitofemoral neuralgia
(L1 –l2) presents with pain in the inguinal region radiating to the superior medial thigh
and genitals. Unlike ilioinguinal neuralgia, a Tinel‘s sign cannot be induced.
Differentiation of the two may prove difficult, as anatomic variability sometimes allows
for communication between the two nerves. Nerve blocks may be performed on either
nerve to see if pain relief results, thus indicating which nerve is involved .

56) The lateral cutaneous nerve of the thigh originates from:


a) L2-3
b) L3-4
c) L4-5
d) L5-S1
A
The upper nerve roots from L2 to L4 form the lumbar plexus, which then ramifies
eventually to form the lateral femoral cutaneous, femoral, and obturator nerves.

Miller ch 43 nerve blocks


The femoral nerve is formed within the psoas major muscle by posterior divisions of L2,3
& 4 nerves. It emerges from the lateral border of the psoas muscle to descend in
the groove between the psoas and iliacus muscles and enters the thigh by
passing beneath the inguinal ligament lateral to the femoral artery. At this point, the
nerve divides into multiple terminal branches, which have been classified as anterior and
posterior. The anterior branches are primarily cutaneous, the deep branches chiefly
motor.
The femoral nerve supplies the anterior compartment muscles of the thigh (quadriceps,
sartorius) and the skin of the anterior thigh from the inguinal ligament to the knee

The obturator nerve is derived primarily from L3,4 nerves with an occasional minor
contribution from L2. The nerve lies deep in the obturator canal, having descended from
the medial border of the psoas muscle. As the nerve leaves the obturator canal, it
divides into anterior and posterior branches. The anterior branch supplies an articular

56
branch to the hip and the anterior adductor muscles and a variable cutaneous branch
to the lower medial thigh. The posterior branch innervates the deep adductor muscles
and may send an articular branch to the knee.

The lateral femoral cutaneous nerve (L2 and L3) emerges at the lateral border of
the psoas muscle immediately caudad to the ilioinguinal nerve. It descends under
the iliac fascia to enter the thigh deep to the inguinal ligament 1 to 2 cm medial to the
anterior superior iliac spine. The nerve emerges from the fascia lata 7 to 10 cm below
the spine and divides into anterior and posterior branches. The skin of the lateral portion
of the thigh from the hip to midthigh is supplied by the posterior branch; the anterior
branch supplies the anterolateral thigh to the knee

57) Hypercapnia upon CO2 insufflation is least likely to be seen with:


a) Female gender
b) ASA 3
c) Extraperitoneal surgery
d) Preop abnormal PFTs
Answer A

Among the 4 available options, A seems to be the answer

Barash, Anesthesia Library (This will also answer a previous question)

The absorption of CO2 from a closed cavity depends on its diffusibility and perfusion of
the walls of that cavity, and not on the rate of insufflation of the gas into the cavity.143
Mullet et al144 hypothesized that the rate of CO2 diffusion into the body depends on the
duration and the site of CO2 insufflation. They examined PETCO2 and pulmonary CO2
elimination during CO2 insufflation under general anesthesia for gynecologic laparoscopy
(intraperitoneal for 43 minutes), laparoscopic cholecystectomy (intraperitoneal for 125
minutes), and pelviscopy (extraperitoneal for 45 minutes). PETCO2 and pulmonary CO2
elimination increased in parallel from the 8th to the 10th minute after the start of CO 2
insufflation. A plateau was reached 10 minutes later in patients having intraperitoneal
insufflation, whereas PETCO2 and pulmonary CO2 elimination continued to increase
slowly throughout CO2 insufflation during pelviscopy.

Wittgen et al145 compared 20 healthy patients undergoing laparoscopic cholecystectomy


with 10 ASA Class II-III patients. Compared with the healthy group, there were
significant decreases in p H and increases in PaCO2 during pneumoperitoneum in the
ASA III group, and this group also had significantly higher minute ventilation and peak
inspiratory pressures than baseline values. Increasing minute ventilation in most cases
maintains PaCO2 within normal limits but inevitably leads to some increase in airway
pressure. However, in ASA Class III or IV patients, PaCO2 may remain elevated despite
adjusting minute ventilation to normalize PETCO2.127 Preoperative evaluation with
pulmonary function tests demonstrating forced expiratory volumes less than 70% of
predicted values and diffusion defects less than 80% of predicted values can identify
patients at risk for development of hypercarbia and respiratory acidosis after
pneumoperitoneum.

57
Extraperitoneal insufflation of CO2 is one of the most common complications of
laparoscopy. The incidence of this complication has been reported to vary from 0.4 to
2%.

Extraperitoneal insufflation has been associated with higher levels of CO2 absorption
than intraperitoneal insufflation and may be the cause of a sudden rise in PETCO2
during the procedure.104

Anesthesiolgy problem oriented patient management, Anesthesia libraray:


It is at this time that the anesthesiologist should be alert to the possibility of vagally
mediated reflexes. Bronchospasm, bradycardia, and even sinus arrest have been
reported, especially in young women.

TABLE 38-4. Comparison of Laparoscopic Cholecystectomy With Open


Cholecystectomy
Smaller abdominal incision
Longer operating time
Requires pneumoperitoneum
Expensive instrumentation
Decreased postoperative pain
Less postoperative diaphragmatic dysfunction
Improved postoperative pulmonary function
Less postoperative ileus
Early ambulation
Economic benefits (shorter hospital stay, early return to work and normal
activities)

58) All potentiate neuromuscular blockade EXCEPT?


a) Quinidine
b) Neomycin
c) Hypercalcemia
d) Local anesthetics

59) What is the GCS of a patient who opens their eyes to pain, answers questions
with mild confused speech and obeys commands?
a) 10
b) 11
c) 12
d) 13

C
2+4+6
Table 28-18. MODIFIED GLASGOW COMA
SCALE

58
EYE OPENING
Spontaneously 4
To verbal command 3
To pain 2
None 1
BEST VERBAL RESPONSE
Oriented, conversing 5
Disoriented, conversing 4
Inappropriate words 3
Incomprehensible sounds 2
No verbal response 1
BEST MOTOR RESPONSE
Obeys verbal commands 6
Localizes to pain 5
Flexion/withdrawal 4
Abnormal flexion (decorticate) 3
Extension (decerebrate) 2
No response (flaccid) 1
Mild head injury = 13–15; moderate = 9–12;
severe = 8.

60) Regarding a patient with Alzheimer’s dementia, all of the following are true,
EXCEPT
a) Avoid centrally acting anticholinesterases
b) Midazolam is useful to optimize patient cooperation
c) Desflurane will allow for rapid return to baseline level of cognitive
function
d) Can use Glycopyrolate for reversal

Answer B
2005 Q 195 All are good strategies at a Alzheimer patient, except
1. Sedation in premedication will increase the collaboration of patient
2. To avoid anticholinergic drugs
3. The desflurane will allow a return to the faster basic function
4. Glycopyrorate can be used
Answer is 1

59
Currently used cholinesterase inhibitors include donepezil, tacrine, velnacrine, and a
sustained-release preparation of physostigmine. Tacrine and velnacrine can produce
direct hepatotoxicity. Therapy has also been directed at increasing neurotransmitter
levels in the noradrenergic and serotoninergic systems as well. Many other drugs,
including cerebrovascular vasodilators, vitamins, and antidepressants, are used as
adjunctive therapy for Alzheimer's disease.

Management of Anesthesia
Sedative drugs, such as those used for preoperative medication, should be
administered rarely because further mental confusion could result. Anesthetics
known to result in rapid postanesthetic recovery, such as propofol, desflurane, and
sevoflurane, may be advantageous because they permit a more rapid return to the
patient's preoperative state. The possibility of coexisting hepatic dysfunction from tacrine
and velnacrine may also influence the choice of a halogenated, volatile anesthetic. If an
anticholinergic is required, glycopyrrolate, which does not cross the blood–brain barrier,
may be preferable to atropine or scopolamine. In theory, an anticholinergic drug that
enters the brain could exacerbate the dementia. The patient's preoperative drug list
should be reviewed for the possibility of interaction with anesthetics.

61) All are true regarding use of the LMA for supraglottic surgery, EXCEPT
a) Decreased blood and debris in the airway
b) No possibility of cranial nerve injury
c) Less likelihood of bronchospasm on emergence
d) Less sore throat

Aspiration of pharyngeal contents has not been shown to be a problem with the LMA,
and its use has been reported to be suitable during ear nose and throat and dental
procedures. When positive pressure ventilation is employed and airway pressures
exceed 20 cm H2O, inflation of the stomach has occurred. Sore throat occurs in 4% to
12% of patients after LMA use, an incidence comparable to use of a face mask and
oropharyngeal airway. The frequency of coughing and laryngospasm is also similar with
the LMA and the oropharyngeal airway. Although not clinically important in most
instances, partial airway obstruction occurs in 25% to 50% of pediatric and 10% of adult
cases (usually resulting from a downfolding of the epiglottis). Herniation or rupture of the
cuff can occur after repeated autoclaving of the device. Other problems include failure to
insert or function properly (incidence of 0.4% to 6%), overt trauma, and complete
displacement of the LMA from the pharynx.

TABLE 23-2. Complications with Use of the Laryngeal Mask Airway


 Gastroesophageal reflux and aspiration
 Laryngospasm
 Coughing
 Bronchospasm
 Sore throat (less than tracheal intubation)
 Transient changes in vocal cord function (possibly related to cuff overinflation during

60
prolonged procedures)
 Nerve injury (recurrent laryngeal, hypoglossal, lingual)
 LMA cuff pressure should not exceed 60 cm H2O
 Nitrous oxide may diffuse into cuff and increase pressure

62) Which of the following is the reason nasal tracheal intubation is more difficult
than oral in neonates:
a) Large occiput
b) Broad epiglottis
c) Cephalad larynx
d) Anterior angulation of vocal cords
Answer D

63) In chronic renal failure which of the following drugs would have the greatest
increase in their half-life?
a) Verapamil
b) Digoxin
c) Esmolol
d) Hydralazine
B

64) Which is LEAST likely to occur with shoulder arthroscopy in the beach chair
position
a) Venous air embolism
b) Airway compression
c) Pneumothorax
d) Brachial plexus injury
Answer C
Miller
The surgical field is clearly higher than the heart, so air embolism is a risk. An
impressive complication of shoulder arthroscopy is rapid, progressive, and complete
airway obstruction, caused by extravasation of the fluid used during the visualization out
of the capsule and into the tissues of the neck

Reconstructive shoulder surgery, including total shoulder arthroplasty and rotator cuff
repair, presents unique management and positioning considerations to the
anesthesiologist. These surgical procedures are typically performed with the patient in
the ―beach chair‖ position. The patient is flexed at the hips and knees and placed in a
10–20-degree reverse Trendelenburg position to promote venous return. The patient is
shifted laterally to the edge of the operating table to allow unrestricted surgical access to
the upper extremity. The patient's head, neck, and hips must be secured to prevent
additional lateral movement. The head and neck must remain firmly supported by the
operating table and secured in a neutral position; excessive rotation or flexion of the
head away from the operative side results in stretch injury to the brachial plexus. Care
also must be taken to avoid pressure on the eyes and ears. All airway connections
should be tightened and possibly reinforced with tape because after surgical draping,
access to the patient's face and airway is limited. Hypotension can be minimized by
gradual assumption of the beach chair position and by delaying head elevation until the

61
time of surgical incision. A tourniquet cannot be used during proximal upper extremity
surgical procedures, and significant blood loss may occur. Therefore, arterial cannulation
may be helpful for direct blood pressure measurement and monitoring of intraoperative
hemoglobin concentrations during total shoulder arthroplasty and reduction of humeral
fractures. In theory, venous air embolism (VAE) may occur during surgical procedures to
the shoulder because the operative site is higher than the heart. However, this
complication has not been reported in the literature. Patients with a documented right-to-
left shunt may be monitored with a precordial Doppler to allow prompt diagnosis and
treatment of VAE.

65) Risk of TNS is associated with all of the following EXCEPT


a) Needle type
b) Obesity
c) Ambulatory surgery
d) Lidocaine 2%
Answer A
2004 Q223. Increased risk of TRI associated with all except:
a) Lidocaine 2%
b) Lithotomy position
c) obesity
d) type of needle used
e) outpatient procedure

ANSWER D
BARASH CH 26
Risk factors for TRI included lithotomy position (with lidocaine, but not bupivacaine or
tetracaine), outpatient status, and obesity. Variables shown not to increase the risk of
TRI included lidocaine dose, type of spinal needle, addition of epinephrine to lidocaine,
paresthesia, hypotension, and blood-tinged CSF. In a separate study, Sakura et al has
shown that the addition of phenylephrine is a risk factor for TRI when 0.5% tetracaine is
used for spinal anesthesia.

Prospective, randomized studies reveal a 4–40% incidence of transient neurologic


symptoms (TNS), including pain or sensory abnormalities in the lower back, buttocks, or
lower extremities, after lidocaine spinal anesthesia.

Clearly, the etiology of TNS remains undetermined, and further studies are needed to
elucidate the underlying mechanism.

66) What is the most effective way of raising the temperature in a patient with
severe hypothermia

a) Warm airway gases


b) Dry convection forced air warmer
c) Warm IV fluid
d) Continuous arterio-venous hemofilteration
Answer D

62
Circulation. 2005;112:IV-136 – IV-138. (it is in 2006 Refs file)
Management of the patient with moderate to severe hypothermia is as follows:

 Hypothermia with a perfusing rhythm


–Mild (>34°C [>93.2°F]): passive rewarming
–Moderate (30°C to 34°C [86°F to 93.2°F]): active external rewarming , (radiant
heat, forced hot air, warmed IV fluids, warm water packs)
–Severe (<30°C [86°F]): active internal rewarming (peritoneal lavage,
esophageal rewarming tubes, cardiopulmonary bypass, extracorporeal
circulation, etc); consider extracorporeal membrane oxygenation
 Patients in cardiac arrest will require CPR with some modifications of
conventional BLS and ACLS care and will require active internal rewarming
–Moderate (30°C to 34°C [86°F to 93.2°F]): start CPR, attempt defibrillation,
establish IV access, give IV medications spaced at longer intervals, provide active
internal rewarming
–Severe (<30°C [86°F]): start CPR, attempt defibrillation once, withhold
medications until temperature >30°C (86°F), provide active internal rewarming
–Patients with a core temperature of >34°C (>93.2°F) may be passively
rewarmed with warmed blankets and a warm environment. This form of
rewarming will not be adequate for a patient with cardiopulmonary arrest or
severe hypothermia

67) What nerve will you block with a needle placed between the palmaris longus
and the flexor carpi radialis tendons?

a) Radial
b) Musculocutaneous
c) Ulnar
d) Median
Answer D

68) Use of ddAVP is indicated in all of the following, EXCEPT

a) Following CABG surgery


b) Von Willebrand’s disease
c) DI
d) Uremia induced platelet dysfunction
I would go w/ A
Miller
1-deamino-8-d-arginine vasopressin (DDAVP), a synthetic analog of the antidiuretic
hormone vasopressin. It increases the levels of factor VIII and von Willebrand
factor and is therefore well-established therapy for hemophilia and von Willebrand
disease. It also reduces blood loss and transfusion requirement in patients with
normal preoperative coagulation status who are undergoing spinal or cardiac
surgery. However, the ultimate role of DDAVP remains to be determined.[76] DDAVP can
cause hypotension, hyponatremia, and increased platelet adhesion.
A large meta-analysis using perioperative blood transfusion as the outcome in
cardiac surgery concluded that aprotinin and tranexamic acid, but not DDAVP,

63
decreased the exposure of patients to allogeneic blood transfusion
perioperatively.[80] The ultimate use of these drugs is still evolving.
Of Note: tranexamic acid, which is also an antifibrinolytic drug. Two studies found a
decreased blood loss from total-knee arthroplasty.[78][79] Presumably, release of the
pneumatic tourniquet releases fibrinolytic material, which is inhibited by tranexamic acid.

Miller ( Ch 50 )
Suggested treatment algorithm for patients with excessive post-cardiopulmonary bypass
microvascular bleeding

Uptodate
USE
Injection: Treatment of diabetes insipidus; control of bleeding in hemophilia A,
and mild-to-moderate classic von Willebrand disease (type I)
DOSING: ADULTS
Diabetes insipidus:
I.V., SubQ: 2-4 mcg/day (0.5-1 mL) in 2 divided doses or 1/10 of the maintenance
intranasal dose
Intranasal (100 mcg/mL nasal solution): 10-40 mcg/day (0.1-0.4 mL) divided 1-3
times/day;
Hemophilia A and mild-to-moderate von Willebrand disease (type I):
I.V.: 0.3 mcg/kg by slow infusion, begin 30 minutes before procedure
Warning:
Injection is not for use in hemophilia B, severe classic von Willebrand disease (type IIB),
or in patients with factor VIII antibodies. In general, the injection is also not
recommended for use in patients with less than 5% factor VIII activity level, although it
may be considered in selected patients with activity levels between 2% and 5%.

Platelet dysfunction & Uremia

64
Desmopressin (dDAVP) — The simplest and least toxic acute treatment for platelet
dysfunction in the uremic patient is the administration of desmopressin (dDAVP),
an analog of antidiuretic hormone with little vasopressor activity [21,22]. Desmopressin
is effective in at least one-half of patients and appears to act by increasing the release of
factor VIII:von Willebrand factor multimers from endothelial storage sites [23].

Intravenous administration of dDAVP is the preferred route of treatment, at a dose of 0.3


microg/kg given in 50 mL of saline over 15 to 30 minutes. Alternatively, the same dose
can be given subcutaneously or 3 microg/kg can be given intranasally. The improvement
in bleeding time begins within one hour and lasts 4 to 24 hours. dDAVP is most useful
for acute bleeding or preparation for a renal biopsy; tachyphylaxis typically develops
after the second dose, perhaps due to depletion of endothelial multimer stores. The
combination of dDAVP administration along with correction of anemia may have an
additive effect on lowering the bleeding time.

Desmopressin has no effect on platelet count or aggregation, but enhances platelet


adhesion to the vessel wall.

Barash 4ed

Desmopressin

Desmopressin (<DDAVP>) is a synthetic analogue of ADH with an intense antidiuretic


(V2) effect and decreased pressor (V1) effect. Through its V2 effects, <DDAVP> also
causes endothelial cells to release von Willebrand factor, tissue-type plasminogen
activator, and prostaglandins. The elimination half-time of <DDAVP> is 2.5 to 4.4 hours.
There are fewer side effects produced by <DDAVP> than are associated with
vasopressin, although nausea and increases in systemic blood pressure can occur.

Administered intranasally twice daily, using a calibrated catheter (Rhinyle), <DDAVP> is


the drug of choice in the treatment of diabetes insipidus due to inadequate production of
ADH by the posterior pituitary. <DDAVP>, like all the ADH analogues, is not effective in
the treatment of nephrogenic diabetes insipidus. Increased release of von Willebrand
factor accounts for the hemostatic activity of <DDAVP> in patients with uremia, chronic
liver disease, and certain types of hemophilia by promoting platelet adhesiveness to the
vascular endothelium. <DDAVP> has also been reported to minimize intraoperative
blood loss in patients undergoing cardiac surgery with cardiopulmonary bypass whereas
other reports find no effect on blood loss in patients undergoing cardiac surgery or spinal
fusion surgery .

<DDAVP administered IV may decrease systemic vascular resistance leading to


hypotension

69) Which of the following metabolites have antinociceptive properties?

a) Morphine-6-glucuronide
b) Morphine-3-glucuronide
c) Normorphine

65
d) Normeperidine
Answer B
Acta Anaesthesiol Scand. 1997 Jan;41(1 Pt 2):116-22.
M3G has been found to antagonise morphine and M6G induced analgesia and
ventilatory depression in the rat, which has led to the hypothesis that M3G may influence
the development of morphine tolerance. M3G exhibits no analgesic effect after ICV or IT
administration. Some studies do, however, indicate that M3G may cause non-opioid
mediated hyperalgesia/allodynia and convulsions after IT administration in rats. These
observations led to the hypothesis that M3G might be responsible for side-effects,
hyperalgesia/allodynia and myoclonus seen after high-dose morphine treatment.
Like morphine, M6G has been shown to be relatively more selective for mu-receptors
than for delta- and kappa-receptors while M3G does not appear to compete for opioid
receptor binding. The metabolism of morphine occurs not only in the liver, but may also
take place in the brain (only small amount) and the kidneys.

Anesth Analg. 2007 Jul;105(1):70-8.


M-6-G is a potent opioid agonist and M-3-G a mild opioid antagonist. Both are
poorly excreted in patients with renal failure.
P&P
Normeperidine has an elimination half-time of 15 hours (<35 hours in patients in renal
failure) and can be detected in urine for as long as 3 days after administration of
meperidine. This metabolite is about one-half as active as meperidine as an analgesic.
In addition, normeperidine produces CNS stimulation. Normeperidine toxicity manifesting
as myoclonus and seizures is most likely during prolonged administration of meperidine
as during patient-controlled analgesia, especially in the presence of impaired renal
function

The principal pathway of metabolism of morphine is conjugation with glucuronic acid in


hepatic and extrahepatic sites, especially the kidneys. About 75% to 85% of a dose of
morphine appears as morphine-3-glucuronide, and 5% to 10% as morphine-6-
glucuronide. An estimated 5% of morphine is demethylated to normorphine, and a small
amount of codeine may also be formed. Metabolites of morphine are eliminated
principally in the urine, with only 7% to 10% undergoing biliary excretion. Morphine-3-
glucuronide is detectable in the urine for up to 72 hours after the administration of
morphine. A small fraction (1% to 2%) of injected morphine is recovered unchanged in
the urine.

70) Which of the following predict risk of postoperative ventilation in a patient with
Myasthenia Gravis coming for trans-sternal thymoma resection

a) Number of years since diagnosis


b) steroid dose
c) Total Lung Capacity
d) Size of thymoma
Answer A
CoExisting p525
Criteria for need of post-op ventilation:

66
1. duration of Dx > 6 yr;
2. COPD;
3. Pyridostigmine dose > 750mg/Day in the 48 hr prior to surgery;
4. Pre-op VC < 2.9 L (VC<40ml/kg)

71) While inserting a large bore intravenous cannula in the antecubital fossa, the
patient complains of pain and tingling in the lateral aspect of the forearm. What
is responsible for this?

a) Contact with the median nerve


b) Contact with the radial nerve
c) Contact with the lateral cutaneous nerve of the forearm
d) Contact with the anterior interosseus nerve of the forearm
Answer C

72) Which of the following will cause a metabolic acidosis with a wide anion gap?

a) Renal tubular acidosis


b) Addison’s disease
c) Carbonic anhydrase inhibitor
d) Salicylate toxicity
Answer D
Table 9-5. DIFFERENTIAL DIAGNOSIS OF METABOLIC ACIDOSIS
Elevated Anion Gap Normal Anion Gap
Three Diseases
1. Uremia 1. Renal tubular acidosis
2. Ketoacidosis (Alcoholic / Starvation / DKA) 2. Diarrhea
3. Lactic acidosis 3. Carbonic anhydrase inhibition
Toxins (SEPTM) 4. Ureteral diversions
1. Methanol 5. Early renal failure
2. Ethylene glycol 6. Hydronephrosis
3. Salicylates 7. HCl administration
4. Paraldehyde 8. Saline administration
5. Tolwin

Addison’s Dx

Summary of Considerations
1. Physiologic effects:
a. Cardiac: hypovolemia, hypotension, myocardial depression
b. Metabolic: hyponatremia, hyperkalemia, hypoglycemia
c. MSK: weakness (titrate neuromuscular blockers)

67
d. Renal: renal insufficiency (↓ RBF)
2. Preparation to treat Addisonian Crisis:
a. Hydrocortisone 100 mg IV q 6 hours
b. Fluid replacement (Saline)
c. Electrolytes (treat potential hyponatremia, hyperkalemia, hypoglycemia)
Invasive monitoring (potential myocardial depression)

73) In the comatose patient, attenuation of the cardiovascular effects from tricyclic
antidepressant overdose is best achieved by which one of the following?

a) Forced diuresis
b) Hemodialysis
c) Physostigmine
d) Alkinalization of the blood
Answer D
RPT Q 151 2005
Tricyclic antidepressant overdose is life threatening, as the progression from an alert
state to unresponsiveness may be rapid. Intractable myocardial depression or ventricular
cardiac dysrhythmias are the most frequent terminal events.
Presenting features of tricyclic antidepressant overdose ( are CV , CNS and
anticholinergic Sx) include agitation and seizures followed by coma, depression of
ventilation, hypotension, hypothermia, and striking evidence of anticholinergic effects
including mydriasis, flushed dry skin, urinary retention, and tachycardia. The QRS
complex on the ECG may be prolonged to >100 ms. Indeed, the likelihood of seizures
and ventricular dysrhythmias is increased when the duration of the QRS complex is
>100 ms. conversely, plasma concentrations of tricyclic antidepressants do not allow
prediction of the likely occurrence of seizures or cardiac dysrhythmias.
The comatose phase of tricyclic antidepressant overdose lasts 24 to 72 hours. Even
after this phase passes, the risk of life-threatening cardiac dysrhythmias persists for up
to 10 days, necessitating continued monitoring of the ECG in these patients.
Treatment of a life-threatening overdose of a tricyclic antidepressant is directed toward
management of CNS and cardiac toxicity. Coma usually resolves in 24 hours but is
frequently severe enough to require invasive airway support. Extrapyramidal effects and
organic brain syndrome usually require supportive care only, although judicious use of
physostigmine, 0.5 to 2 mg given intravenously (IV), for treatment of anticholinergic
psychosis may be indicated.
Seizures may precede cardiac arrest and should be treated aggressively with diazepam.
After initial suppression of seizure activity with diazepam, it may be necessary to provide
sustained effects with a longer-acting drug such as phenytoin. Acidosis associated with
seizure activity may abruptly increase the unbound fraction of tricyclic antidepressants in
the circulation and predispose to cardiac dysrhythmias. In this regard, alkalization of
the plasma (pH >7.45) either by IV administration of sodium bicarbonate or
deliberate hyperventilation of the patient’s lungs can temporarily reverse drug-
induced cardiotoxicity. Lidocaine and phenytoin may be used subsequently to provide
sustained suppression of cardiac ventricular dysrhythmias.
Hypotension may be the result of direct tricyclic antidepressant–induced vasodilation,
alpha-adrenergic blockade, or myocardial depression. Patients remaining hypotensive
despite intravascular fluid replacement and alkalinization of the plasma may require
systemic blood pressure support with sympathomimetics, inotropics, or both.

68
Gastric lavage may be useful in the early treatment, but this is most safely performed
with a cuffed tracheal tube already in place. Activated charcoal significantly absorbs
drugs throughout the gastrointestinal tract (―intestinal dialysis‖). Conversely, avid
protein binding of tricyclic antidepressants negates any therapeutic value of
hemodialysis or drug-induced diuresis.

74) What is true regarding bolus dose propofol in terms of pharmacokinetics and
pharmacodynamics?

a) Termination of effect is by hepatic enzymes


b) Elimination half life is several hours
c) Small volume of distribution in vessel rich group
d) Elimination follows zero order kinetics
Answer B
P&P
TABLE 1-2. Body tissue composition
Body mass (% of 70-kg Blood flow (% of cardiac
adult) output)
Vessel-rich group 10 75
Muscle group 50 19
Fat group 20 6
Vessel-poor group 20 <1

Barash 4ed
Using a three-compartment model the initial and slow distribution half-life values are 1–8
min and 30–70 min, respectively. The elimination half-life depends largely on the
sampling time after discontinuing the administration of propofol and ranges from
2 to 24hrs . This long elimination half-life is indicative of the existence of a poorly
perfused compartment from which propofol slowly diffuses back into the central
compartment
Table 13-1. PHARMACOKINETIC VALUES FOR THE CURRENTLY AVAILABLE
INTRAVENOUSSEDATIVE-HYPNOTIC DRUGS
Protein Distribution Clearance
Distribution Binding Volume at Steady (ml·kg–1·min– Elimination
Drug Name Half-life (min) (%) State (L·kg–1) 1
) Half-life (h)
Thiopental 2–4 85 2.5–0.0 3.4 11
Methohexital 5–6 85 2.2–0.0 11 4

Propofol 2–4 98 2–10 20–30 4–23


Midazolam 7–15 94 1.1–1.7 6.4–11 1.7–2.6

69
Diazepam 10–15 98 0.7–1.7 0.2–0.5 20–50
Lorazepam 3–10 98 0.8–1.3 0.8–1.8 11–22
Etomidate 2–4 75 2.5–4.5 18–25 2.9–5.3

Ketamine 11–16 12 2.5–3.5 12–17 2–4

75) All of the following hormones regulate serum potassium EXCEPT

a) Epinephrine
b) Insulin
c) Aldosterone
d) Serotonin
Answer D

Epinephrine and insulin promote K+ entry into cells, thereby preventing the dangerous
rise in serum [K+] (hyperkalemia) that could lead to cardiac arrhythmias. Insulin is the
most important hormone that shifts K+ into cells after ingestion of K+ in a meal.

Stimulus Action Comment


Epinephrine - K+ entry into - Mediated by ß-2 receptor
cells - Acts quickly (minutes) to promote K+ uptake
Insulin - K+ entry into - Acts quickly (minutes) to promote K+ uptake
cells - Potential problem in Type I diabetes
Aldosterone - K+ entry into - Facilitates K+ movement into cells and also K+
cells urinary excretion (discussed above)
- Acts slowly (hours) to promote K+ uptake
High serum [K+] - K+ entry into - Passively, along its gradient
cells
Exercise - K+ release from - K+ increase varies depending on intensity of
cells exercise (from 0.3 to greater than 2 mEq/L above
normal)
+
Metabolic acidosis - K moves out of - Addition of inorganic ions produces much
cells, H+ moves greater plasma [K+] increase than accumulation of
into cells endgenous organic acids (e.g., lactic acid, etc.)
- Not found with respiratory acid-base disorders
+
Hyperosmolarity (of - K release - As plasma osmolality increases, water leaves
plasma) cells, causing intracellular [K+] to rise; this drives
K+ efflux from cells
Cell - K+ release - Severe trauma (e.g., burns) and other
breakdown/lysis syndromes where cell destruction is a prominent
feature

76) Transcutaneous drug patches work by?

70
a) Diffusion through skin
b) Places subcutaneous depot which then gets absorbed
c) Iontophoresis increases the ionized portion of lipid soluble drug which
enhances absorption
d) Lipid solubility decreases absorption
Answer A
Q188. 2000 Transcutaneous administration of medication:
a) absorption of medication occurs passively
b) iontophoresis ionizes lipid soluble medications
c) a stable plasma concentration cannot occur
d) lipid soluble medication decreases absorption
e) creates a subcutaneous depot for absorption

CORRECT ANSWER: A & E

Ref: Miller 5th ed, p 340; P & P, p8-9


Transdermal drug delivery generally requires high solubility in both water and oil
(lipid solubility to pass through the stratum corneum and aqueous solubility to move
through the dermis), low molecular weight, high potency (requiring absorption of only
small amounts for clinical effect), and little or no skin irritation. Under basal
conditions, 450 mL of blood, approximately 10 percent of the cardiac output, passes
through the skin. Under thermal stress, skin blood flow can increase as much as 10-
fold. Cutaneous vasoconstriction from cold stress can virtually abolish skin blood
flow. The transdermal permeability of a drug may vary over different areas of the
body.
Results of clinical trials utilizing fentanyl for postoperative analgesia have
demonstrated a high incidence of significant respiratory depression, and this
application is not recommended.
Plasma fentanyl levels rise and usually reach a plateau within 8 to 16 hours. Steady-
state fentanyl concentrations are achieved by the time a second patch is applied.
Fentanyl blood levels fall slowly after patch removal because absorption of
drug deposited in the skin continues. The half-life for the decline in fentanyl levels
after patch removal is high (17±2.3 h).

Most patients with cancer experience 72 hours of analgesia following application of


the patch, but some individuals only obtain about 48 hours of analgesia from each
unit. This may be due to tumor fever, a common phenomenon that occurs in about
40 percent of patients with active cancer. Elevated body temperature accelerates
either the release of fentanyl from the patch or the distribution from the
subcutaneous fat depot.

Because of the difficulties encountered in the passive delivery of drugs through the
skin, physical and chemical methods of enhancing transdermal drug delivery are
being investigated. Iontophoresis is a technique by which drug passage through the
skin is augmented by an external electric current. Transcutaneous absorption of
drugs from patches is passive, so the onset time is delayed. This disadvantage can
be overcome by using an electric current to ―drive‖ ionized drugs into the skin,
a process is called iontophoresis.

71
77) Alveolar anesthetic tension is increased by all EXCEPT?

a) Increased minute ventilation


b) Increased cardiac output
c) Decreased arterial-venous anesthetic agent gradient
d) Decreased anesthetic agent solubility in blood
Answer B
The F A/F I ratio is ultimately determined by the balance between the delivery of
anesthetic by ventilation and its removal by uptake.
Anesthetic uptake itself is the product of three factors: solubility (lambda), cardiac output
(Q), and alveolar to venous partial pressure difference (P A-P V). [1] That is:

Uptake = (lambda) × (Q) × (P A-P V) divided by barometric pressure

The fact that uptake is a product of the three factors rather than a sum means that if any
factor approaches zero, uptake must approach zero, and ventilation is free to rapidly
drive the alveolar concentrations upward. Thus, if solubility is small (as in the case of
oxygen), if cardiac output approaches zero (as in profound myocardial depression or
death), or if the alveolar to venous difference becomes inconsequential (as might occur
after an extraordinarily long anesthetic), uptake would be minimal, and F A/F I would
quickly equal 1.

P&P 3ed
TABLE 1-6. Factors determining partial pressure gradients necessary for
establishment of anesthesia
Transfer of inhaled anesthetic from anesthetic machine to alveoli (anesthetic
input)
I Inspired partial pressure
Alveolar ventilation
Characteristics of anesthetic breathing system
Functional residual capacity
Transfer of inhaled anesthetic from alveoli to arterial blood (anesthetic loss)
Blood:gas partition coefficient
Cardiac output
Alveolar-to-venous partial pressure difference
Transfer of inhaled anesthetic from arterial blood to brain (anesthetic loss)
Brain:blood partition coefficient
Cerebral blood flow
Arterial-to-venous partial pressure difference

78) All are characteristics of pressure control ventilation EXCEPT?

a) Flow vs. time is a square waveform


b) Inverse ratio is possible
c) Time-cycled
d) Pressure peaks quickly and then maintained throughout inspiration

72
Answer A (2002)
Pressure control ventilation (PCV) is a patient or time-triggered, pressure-limited,
time-cycled mode of ventilatory support. It is characterized by a rapid rise to peak
pressure afforded by a decelerating inspiratory flow pattern. A pressure-controlled
breath can be delivered in intermittent mandatory ventilation or assist mechanical
ventilation instead of volume-oriented breaths, or in conjunction with pressure
support ventilation. The potential advantage of pressure control ventilation is that
because flow rate is geared to reach peak inspiratory pressure quickly, flow will
exceed patient demand and therefore will improve patient–ventilator synchrony and
decrease the work of breathing. Furthermore, pressure control ventilation might
potentially improve distribution of gas within the lung by using the decelerating flow
pattern and square wave air pressure pattern. The major disadvantage of pressure
control ventilation is that tidal volume varies as compliance and resistance of the
airways change.
PCV is characterized by a rapid rise to peak pressure afforded by a decelerating
inspiratory flow pattern, until a point is reached when the delivered pressure equals
airway pressure and the pressure is maintained for the duration of inspiration

Miller
Each mode of ventilation is distinguished by how it initiates a breath (trigger), how it
sustains a breath (limit), and how it terminates a breath (cycle), which are referred to as
"phase variables"

73
Fig 4

Table 75-6 -- Characteristics of volume-targeted ventilation (VTV) and pressure-


targeted ventilation (PTV)
Variable VTV PTV
Trigger Patient or Time Patient or Time
Limit Flow Pressure
Cycle Volume Time or Flow
Tidal Volume Constant Variable
Peak
Variable Constant
Pressure
Assist/Control (synchronized) Assist/Control (synchronized)
Modes
intermittent intermittent

74
Variable VTV PTV
Mandatory ventilation Mandatory ventilation
Pressure support

79) Dead space in a circle system consists of?

a) Both inspiratory and expiratory limbs of the circuit


b) Inspiratory limb of the circuit
c) Expiratory limb of the circuit
d) Y piece and connector
Answer D (2004)
Reference: D&D 3rd ed. p. 213

In the circle system, dead space extends into the Y piece as far as the partition.
Use of a Y piece with a septum will decrease the dead space. When exhalation or
inhalation starts the gases in the breathing tubes move in the opposite direction from
their usual flow until stopped by closure of one of the unidirectional valves. This is
referred to as backlash and causes a slight increase in the dead space. If the
unidirectional valves are competent, however, backlash will be clinically insignificant

Taken from GASNET.com


Dead space is increased (by all respiratory apparatus). VD/VT= 0.33 normally, 0.46 if
intubated and 0.65 if mask case. Mechanical dead space ends at the point where
inspired and expired gas streams diverge (the Y-connector).

80) Which of the following is an absolute contraindication to lithotripsy

a) Pregnancy
b) Pacemaker
c) Abdominal Aortic aneurysm
d) BMI > 40
Answer A
Miller , The contraindications

Pregnancy, untreated bleeding disorders, and abdominally placed pacemakers are the
only absolute contraindications to lithotripsy. Women of childbearing age must have a
pregnancy test that is documented to be negative before lithotripsy. Standard tests of
coagulation such as the platelet count, prothrombin time, and partial thromboplastin time
should be obtained. Other conditions that were previously labeled as absolute
contraindications are no longer believed to be so, provided that appropriate precautions
are taken. These conditions include pacemakers, automatic implanted cardioverter-
defibrillators (AICDs), abdominal aortic aneurysm, orthopedic protheses, and obesity.

75
Patients with pacemakers can be treated safely if the pacemaker is pectorally placed
and the following precautions are observed.[209][210][211][212] Pacemaker programmability
should be established before the treatment, and a programmer should be available to
switch the pacemaker to a nondemand mode should the shock waves interfere with
pacemaker function. Alternative means of pacing should be available. Although most
pacemakers located pectorally are at a safe distance from the blast path, some may be
damaged. Weber and coworkers[209] examined 43 different pacemakers and found that 3
were affected. Dual-chamber pacemakers tend to be more sensitive to interference.
Treatment must be started at the lowest energy level and gradually increased while
observing pacemaker function.

Manufacturers of AICDs and lithotripters generally consider an AICD a contraindication


for lithotripsy. Patients with AICDs have been treated successfully with lithotripsy,
however.[211] Transvenous AICDs are less of a concern than the older abdominally
implanted defibrillators. AICD devices should be shut off immediately before lithotripsy
and then reactivated immediately after treatment.

Patients with small aortic aneurysms have been treated safely provided that the stone is
not close to the aneurysm. Orthopedic prostheses such as hip prostheses and even
Harrington rods are not a problem if they are not in the blast path, which is usually the
case. Positioning of obese patients may sometimes be problematic. Not only do
extremely obese patients present anesthetic challenges related to obesity, but also
focusing of the stone may be extremely difficult in the very obese, and it is not
uncommon for the procedure to be abandoned in these patients because of an inability
to bring the kidney stone into the Fz focal zone. It is therefore prudent for focusing of the
stone to be attempted before administering any anesthetic in this high-risk population.
With the newer lithotripters, some of these patients may have to be placed prone, a
position that they may not be able to tolerate safely.

Table 54-13 -- Changes on immersion during lithotripsy


Cardiovascular Increased Central blood volume
Central venous pressure
Pulmonary artery pressure
Respiratory Increased Pulmonary blood flow
Decreased Vital capacity
Decreased Functional residual capacity
Decreased Tidal volume
Increased Respiratory rate

81) All will decrease the amplitude of SSEPs EXCEPT:

a) Induced hypotension with nitroprusside


b) N2O + sufentanil boluses
c) Fentanyl infusion
d) Midazolam
C (see Q168 2005) From Barash

76
Table 28-8. EFFECTS OF INTRAVENOUS AND INHALED AGENTS ON
SENSORY EVOKED POTENTIALS
BAEPs cSSEPs VEPs

Lat Amp Lat Amp Lat Amp

INTRAVENOUS AGENTS
Thiopental
4–6 mgkg–1 0 0 0 0 — —

75 mgkg–1   
Pentobarbital   
9–18 mgkg–1
Droperidol — —  — —
0.1 mgkg–1
Diazepam 0 0 /0 /0 0 
0.1 mgkg–1
Midazolam 0 0 /0 /0 — —

Morphine — — /0 — —
Fentanyl 0 0 /0 /0 0 
Sufentanil 0 0 /0 /0 — —

Alfentanil 0 0 /0 /0 — —

Etomidate 0 0 0
0.05–0.3 mgkg–1min–1
Propofol 0 /0 — —
2–6 mgkg–1

Ketamine 0 /0
INHALATION AGENTS
Desflurane 0  — —
Enflurane 0 * ¯

Halothane 0 ¯ ¯
Isoflurane 0 ¯* ¯

77
Sevoflurane 0 ¯ — —
Nitrous oxide 0 Dec 0 Dec Dec
BAEPs = brain stem auditory evoked potentials; cSSEPs = cortical somatosensory
evoked potentials; VEPs = visual evoked potentials; = increased; ¯ = decreased;
0 = no change; — = no data; Lat = latency; Amp = amplitude.

* 1.5 MAC enflurane and isoflurane (but not halothane) will occasionally abolish
the cortical evoked response to median nerve stimulation.

The benzodiazepines produce minimal SSEP changes. Opioids produce


minimal changes in SSEP waveforms.

Miller
Table 38-9 -- Ability of an anesthetic drug to produce a change in sensory and
motor evoked potentials that could be mistaken for a surgically induced change
SSEPs BAEPs VEPs Transcranial MEPs
Drug Lat Amp Lat Amp Lat Amp Lat Amp
Isoflurane Yes * Yes No No Yes Yes Yes Yes
Enflurane Yes Yes No No Yes Yes Yes Yes
Halothane Yes Yes No No Yes Yes Yes Yes
Nitrous oxide † Yes Yes No No Yes Yes Yes Yes
Barbiturates Yes Yes No No Yes Yes Yes (p) Yes (p)
Etomidate No No No No Yes Yes No No
Propofol Yes Yes No No Yes Yes Yes Yes
Droperidol No No No No — — Yes Yes
Diazepam Yes Yes No No Yes Yes Yes (p) Yes (p)
Midazolam Yes Yes No No Yes Yes Yes (p) Yes (p)
Ketamine No No No No Yes Yes No No
Opiates No No No No No No No No
Amp, amplitude; BAEPs, brainstem auditory evoked potentials; latency; MEPs, motor
evoked potentials; p, prohibitive in clinically useful doses because use of this drug at any
dose may render this type of monitoring impossible for a significant period; SSEPs,
somatosensory evoked potentials; VEPs, visual evoked potentials.

Diazepam causes increases in latency and decreases in amplitude of SSEPs, increases


in latency in the cortical response after auditory stimulation, and no change in
BAEPs.[148][149] Midazolam causes decreases in amplitude without changes in the latency
of SSEPs.[144]

In general, opioids cause small, dose-dependent increases in latency and decreases in


the amplitude of SSEPs. These changes are not clinically significant. Effects on
amplitude are more variable than the latency increases.[150][151] Even at large doses of

78
fentanyl (≤60 µg/kg), reproducible SSEPs can be recorded.[151] Other opiates cause
similar dose-dependent changes in SSEPs.[150][152] Even in relatively large doses, opioids
can be used in patients requiring intraoperative SSEP monitoring without impairment of
ability to monitor neurologic function adequately. However, opioid-induced changes must
be taken into account when evaluating the recordings. Large intravenous bolus
administration of opioids should be avoided at times of potential surgical
compromise to neurologic function to prevent confusing the interpretation of SEP
changes if they develop.

A number of physiologic variables, including systemic blood pressure, temperature (local


and systemic), and blood gas tensions, can influence SEP recordings. With decreases in
mean arterial blood pressure to below levels of cerebral autoregulation due to blood loss
or vasoactive agents, progressive changes in SERs have been observed. The SSEP
changes observed are progressive decreases in amplitude until loss of the waveform
with no changes in latency.[154][155] BAEPs are relatively resistant to even profound levels
of hypotension . Changes in temperature also affect SERs. Hypothermia causes
increases in latency and decreases in amplitude of cortical and subcortical SERs after all
types of stimulation.[158][159][160] Hyperthermia also alters SERs, with increases in
temperature leading to decreases in the amplitude of SSEPs and loss of SSEPs at 42°C
during induced hyperthermia. Hypoxia produces SSEP changes (i.e., decreased
amplitude) similar to those seen with ischemia.[163] Decreased oxygen delivery
associated with anemia during isovolemic hemodilution results in progressive increases
in the latencies of SSEPs and VEPs, which become significant at hematocrits below
15%.

82) All the following are methods of directly measuring cardiac output EXCEPT:

a) Fick principle
b) Dye dilution
c) Oxygen consumption
d) Echocardiography
C (repeat Q-52 2002 ) ; ref miller
In addition to the Fick principle, cardiac output can be measured by several
other techniques. Injection of an indicator dye such as indocyanine green into the
venous blood allows determination of cardiac output by calculating the area under
the dye dilution curve as measured by densitometry of the arterial blood. The major
limitation of this method is the recirculation of dye, which then requires extrapolation
of the downward slope of the dilution curve to estimate the area under the peak and
hence cardiac output. Thus, repeated injections cause build-up of dye levels and
further hinder the usefulness of this method. This problem, however, has a hidden
advantage in that it can be used to determine the presence of shunts. The initial
bolus of dye injected into the blood normally produces a second or recirculation peak
after the primary dilution curve from the second passage of dye through the heart.
The presence of a right-to-left shunt results in the appearance of a more prominent
initial peak as well as a secondary recirculation peak, which occurs much earlier than
normal. Conversely, the presence of a left-to-right shunt results in the appearance of
a diminished initial dye curve and a barely detectable recirculation peak, which
occurs much later than normal, with marked prolongation of dye elimination.

Another method of determining cardiac output is Doppler echocardiography. This


noninvasive technique is based on measuring the cross-sectional area of the aortic

79
valve and the velocity of blood flow by ultrasound. Although it has shown good
experimental correlation with other methods of determining cardiac output, its use in
clinical situations has been limited by reliability issues.
Currently, the most widely used technique for measuring cardiac output is the
thermodilution method. Like the dye method, it relies on dilutional measurements of a
substance injected as a bolus. In this case, a thermistor sensor measures changes in
temperature of a known quantity of injectate of normal saline or dextrose at either
4°C or room temperature and calculates the cardiac output as the area under the
curve. Although it only measures output from the right side of the heart, this is
assumed to correlate well with that of the left. Thus, the limitations of this method
include the presence of shunts, the use of varying quantities of injectate, the speed
of injection, and the baseline temperature, all of which may overestimate or
underestimate the actual cardiac output.

83) How many grams of dextrose are in 1L of an isotonic dextrose solution?


a) 0.5 g
b) 5 g
c) 50 g
d) ?

C Reference: Miller. Chapter 45, Fluid and Electrolyte Balance

Solution Na mEq/L K mEq/L Dextrose g/L Osm pH


0.9% NS 154 0 0 308 6.0
RL 130 4 0 273 6.5
5% dextrose 0 0 50 252 4.5
D5NS 154 0 50 560 5.3
D5/0.45 77 0 50 406 4.0

84) Which would be a detrimental effect of therapeutic levels of magnesium in PIH

a) Increased uterine activity


b) Skeletal muscle weakness
c) Bronchospasm
d) ?

MAGNESIUM PHYSIOLOGY
Hypomagnesemia:
 risk of perioperative arrhythmias.
 Respiratory muscle power is impaired by hypomagnesemia,
 central nervous system irritability with seizures and hyperreflexia and skeletal
muscle spasm
Hypermagnesemia (>2.5 mEq/L), patients with kidney failure are at increased risk of
developing hypermagnesemia.
 anticonvulsant activity of magnesium is related to its powerful cerebral
vasodilator action that reverses cerebral vasospasm
 potentiates the action of local anesthetics

80
 cause significant presynaptic neuromuscular blockade and enhance the action of
the nondepolarizing muscle relaxants
 Magnesium prolongs the action of depolarizing relaxants (e.g., succinylcholine);
administration before the use of succinylcholine prevents the release of
potassium provoked by the relaxant.
 magnesium produces vasodilation by direct action on blood vessels. It also
reduces peripheral vascular tone by sympathetic blockade and inhibition of
catecholamine release.
 Decreased myocardial performance has been demonstrated after a bolus of 2.5
g of magnesium sulfate. It is also efficacious in certain arrhythmias induced by
hypokalemia, alcoholism, and myocardial infarction and may protect against
bupivacaine-induced arrhythmias and torsade
 effective bronchodilator and has been successfully used in severe asthma
 In the kidney, magnesium is a renal vasodilator and a diuretic

From UpTodate;
Symptoms and electrocardiographic changes of hypermagnesemia correspond to serum
levels; depressed cardiac conduction, widened QRS complexes, prolonged P-Q
intervals, and nausea appear between 5 and 10 mg/dL. Sedation, hypoventilation,
decreased deep tendon reflexes, and muscle weakness appear at levels between 20
and 34 mg/dL, with hypotension, bradycardia, and diffuse vasodilation occurring at levels
of 24 to 48 mg/dL. Areflexia, coma, and respiratory paralysis occur at 48 to 72 mg/dL.
Elimination of magnesium involves fluid loading followed by or with concomitant diuresis.
Definitive therapy involves dialysis. Temporary reversal of the effects of magnesium can
be managed with calcium therapy

Since magnesium sulfate is excreted by the kidneys, dosing should be adjusted in


women with renal insufficiency (defined as a serum creatinine greater than 1.0 mg/dL).
Such women should receive a standard loading dose (since their volume of distribution
is not altered), but a reduced maintenance dose (1 g per hour or no maintenance dose if
the serum creatinine is greater than 2.5 mg/dL) and close monitoring of their serum
magnesium level every six hours.

Magnesium sulfate is contraindicated in women with myasthenia gravis since it can


precipitate a severe myasthenic crisis. Concurrent use of magnesium sulfate with
calcium channel blockers may result in hypotension.

The maintenance phase is given only if a patellar reflex is present (loss of reflexes being
the first manifestation of symptomatic hypermagnesemia), respirations exceed 12 per
minute, and the urine output exceeds 100 mL per four hours.

There also does not appear to be a clear threshold concentration for insuring the
prevention of convulsions, although a "therapeutic range" of 4.8 to 8.4 mg/dL (2.0 to 3.5
mmol/L) has been recommended based on retrospective data. The dose should be
adjusted down if there is clinical evidence of magnesium toxicity.

Complications — Rapid infusion of magnesium sulfate causes diaphoresis, flushing,


and warmth, probably related to peripheral vasodilation and a drop in blood pressure.
Nausea, vomiting, headache, muscle weakness, visual disturbances, and palpitations
can also occur.

81
Magnesium toxicity is related to serum concentration: loss of deep tendon reflexes
occurs at 9.6 to 12.0 mg/dL (4.0 to 5.0 mmol/L), respiratory paralysis at 12.0 to 18.0
mg/dL (5 to 7.5 mmol/L), and cardiac arrest at 24 to 30 mg/dL (10 to 12.5 mmol/L).
Calcium gluconate (1 g intravenously over 5 to 10 minutes) should be administered only
to counteract life-threatening symptoms of magnesium toxicity (such as cardiorespiratory
compromise).

The risk of postpartum hemorrhage, possibly related to uterine atony from magnesium's
tocolytic effects, was slightly increased in one trial .

Magnesium freely crosses the placenta; as a result, the cord blood concentration
approximates the maternal serum concentration. Maternal therapy causes a slight
decrease in baseline fetal heart rate and fetal heart rate variability, which are not
clinically significant.

Magnesium therapy also results in a transient reduction of total and ionized serum
calcium concentration due to rapid suppression of parathyroid hormone release . Rarely,
the hypocalcemia becomes symptomatic (myoclonus, delirium, ECG abnormalities).

85) What reduces the incidence of intraoperative fires with CO2 lasers?
a) Using a red rubber ETT
b) Wrapping a PVC ETT with lead foil
c) Using N2O
d) Using the laser in a noncontiguous mode
A
Reference: Miller 6th ed, Ch-67

ENDOTRACHEAL TUBE FIRES


1. The estimated incidence of this complication during such operations is 0.005 %
2. With the energy delivery rates described earlier, any hydrocarbon material,
including tissue, plastic, or rubber, can ignite and burn, particularly in an O2-
enriched atmosphere.
3. Two strategies have been developed to reduce the incidence of airway fire:
1) reduce the flammability of the endotracheal tube, and
2) remove flammable materials from the airway by using a metallic
venturi jet ventilation cannula or intermittent extubation with or
without apnea.

Relative Flammability: Effect of Tube Composition


1. Modern PVC strongly absorbs far-infrared light and is thus very sensitive to
CO2 laser energy. PVC tubes appear to be much more easily ignited by CO2
lasers than red rubber tubes and to produce more toxic combustion products.
2. In vitro, PVC is transparent and thus is immune to Nd:YAG and visible laser
light; however, a thin coating of mucus or blood in vivo will absorb energy
and restore the hazard.]
3. Two studies of the effects of Nd:YAG laser energy on common types of tubes,
including those designed to be resistant to laser irradiation, revealed that
endotracheal tubes of all materials are, in fact, quite vulnerable

82
4. PVC ignited and developed intense flame the soonest, resulting in widespread
deposit of carbonaceous debris and significant ulceration and inflammation of the
trachea on postmortem analysis.
5. Red rubber tubes were more resistant to ignition and produced less debris and
inflammation.
6. Silicone tubes were the most resistant to ignition, but they produced copious
white silica ash, suggesting the potential for late development of silicosis.
7. Ossoff subsequently measured the time to intraluminal ignition during exposure
to CO2 laser energy to determine resistance to ignition and reversed the ranking
of silicone and red rubber, PVC remaining the most vulnerable.
8. Addition of 2 percent halothane vapor retarded ignition.

Effect of Respiratory Gas Mixture


1. Combustion is more vigorous when excess oxidizer is present, and most
clinicians recognize the need to reduce the F IO2 to less than 0.40 or to the
minimum concentration consistent with patient oxygenation.
2. That nitrous oxide also is a powerful oxidizer is less well recognized, but adding
nitrous oxide as a diluent for oxygen is just as dangerous as having a high F IO 2
3. Use of an air-oxygen mixture appears to be acceptable.
4. The volatile anesthetics currently used in clinical practice are nonflammable and
nonexplosive in clinically relevant concentrations. During an airway fire, however,
they may undergo pyrolysis to potentially toxic compounds.

Protection of the Endotracheal Tube


1. Wrapping tubes with metallized foil tape.Three
1) aluminum foil with adhesive backing,
2) copper foil with adhesive backing, and
3) plastic tape thinly coated with metal on one side and adhesive on the
other.
4) Lead foil (commonly used on windows for burglar alarms) is
similar in appearance but is toxic and should never be used in the
airway.

Protection of the Endotracheal Tube Cuff


1. Fill the standard Hi-Lo cuff with colored saline both to signal a puncture more
clearly and potentially to quench a small cuff fire.
2. The cuff should be placed as far distal in the trachea as possible, and the
surgeon should completely cover the visible cuff with moistened cotton pledgets.

Metal Endotracheal Tubes


1. The Laser Flex tube (Mallinckrodt Inc., St. Louis, Mo) is an airtight stainless steel
spiral with two distal, saline-inflatable PVC cuffs (redundant in case of puncture
of the proximal cuff). This tube is resistant to CO2 and KTP laser energy, but not
to the Nd-YAG laser.
2. The Bivona Fome-Cuf (Bivona Inc., Gary, Ind) is an aluminum spiral tube with an
outer covering of silicone and a unique, self-inflating foam sponge-filled cuff,
which prevents deflation following puncture.
3. The use of laser-resistant endotracheal tubes requires particular care to prevent
mucosal abrasions, because these tubes tend to be bulkier and more rigid than
conventional tubes.

83
4. Finally, use of a metal endotracheal tube does not imply absolute protection from
ignition; 50 W of an Nd:YAG beam focused to 0.68 mm has been reported to
ignite a Laser Flex tube in 6 seconds.

Similar to 2004 Q238. When using lasers, the best way to minimize an airway fire is?
a) Use a rubber endotracheal tube.
b) Wrap the endotracheal tube with lead foil.
c) Use the laser in short, intermittent pulses.
d) Use an FIO2 >0.4.
e) Nitrous is a combustable gas
ANSWER C

Also see 2004 Q26. Which of the following is True about laser surgery?
a) CO2 laser is absorbed by water and has deep penetration
b) ND-YAG laser penetrates tissue to 200 um
c) Nitrous oxide supports combustion
d) PVC tubes are safe in laser surgery
e) Rubber tubes are safe for CO2 laser
ANSWER: C (versus E?)

86) Which laser can cause corneal burns from scattered light ?

a) CO2
b) Argon
c) Nd-Yag
d) ?
Answer A
LASER HAZARDS

Atmospheric Contamination: Laser Plume


1. Odor  headaches, tearing, and nausea
2. Deposition of laser plume particulates in rat lung appears capable of producing
interstitial pneumonia, bronchiolitis,↓ mucociliary clearance, inflammation &
ephysema
3. Potentially, laser plume also may be mutagenic, teratogenic, or a vector for viral
infection (viral DNA has been detected in plume from condylomas and skin
warts, but not from laryngeal papillomas
4. CO2 lasers seem to produce the most smoke due to the vaporization of tissue,
and Nd:YAG contact probes produce much less.
5. The most effective means of preventing dissemination of the plume is to use an
efficient smoke evacuator at the surgical site.
6. Ordinary surgical masks efficiently filter particles only down to 3.0 mum, and
therefore, special high-efficiency masks (e.g., The Protector II, Anago, Fort
Worth, Tex) are required to catch laser plume particulates. However, the high-
efficiency masks are less effective when wet and may need to be changed
periodically.

Energy Transfer to an Inappropriate Location


Eyes, surgical drapes, endotracheal tube

84
EYE PROTECTION
1. Errant infrared energy from a CO2 laser can quickly cause a serious corneal
injury, whereas argon, (KTP) Nd:YAG, or ruby lasers may burn the retina.
2. The lids of patients' (nonoperated) eyes should be taped closed, then covered
with an opaque, saline-soaked knit or metal shield.
3. Operating room personnel must wear safety goggles or lenses specific for the
laser wavelength in use.
4. For CO2 lasers, any clear glass or plastic lenses will suffice because they are
opaque to far infrared. Regular eyeglasses may be sufficient, but contact lenses
are not.
5. Other lasers require color filters whose wavelength specificity and optical density
are regulated

87) According to the ACC/AHA guidelines for risk stratification of patients for non-
cardiac surgery, which of the following is NOT a major risk factor?

a) DM
b) Severe valvular disease
c) Significant arrhythmia
d) Congestive heart failure
A (also see Q-129 2005)
2005 Refs\Executive summary 2002, guidelines on Pre op assesmant of cardiac pts
for non cardiac surgery.pdf

88) Which of the following does NOT predispose to postpartum hemorrhage?

a) Stimulated labour
b) PIH
c) Macrosomia
d) Atony
B see Q- 2004 Q154
Chestnut table 37-2 p669
Factors predisposing to post-partum hemorrhage
Atony Lacerations, Placental Coagulation
disruptions abnormalities disorders
Precipitous labor X X
Instrumental delivery X
General anesthesia X
Prolonged labor X
Uterine leiomyomas X X X
Macrosomia X X
Twins X X
Chorioamnionitis X
Multiparity X X
Prior C/S X X
Prior hysterotomy or X X
curettage
Stimulated labor X X
Hx PPH X X
Fetal demise X X

85
Amniotic fluid X X
embolus
Tocolytic Rx X X

Miller 6th ed,Ch-58


Postpartum hemorrhage remains a major cause of maternal morbidity and mortality
worldwide. The causes of postpartum hemorrhage can be divided into uterine and
nonuterine causes. The most frequent uterine causes are atony and retained placental
products. Other uterine causes include uterine rupture and uterine inversion. Nonuterine
causes include vaginal tears, perineal hematoma, and maternal coagulopathy.

Uterine atony is the most common cause of postpartum hemorrhage, and it is caused
by ineffective uterine muscle contraction in the postpartum period. Risk factors include
prolonged labor, an overdistended uterus (macrosomia or multiple births), infection,
grand multiparity, and administration of drugs that relax the uterus (halogenated
anesthetics, β-sympathomimetic agonists, and magnesium sulfate).

89) With respect to the neurologic exam following brachial plexus block, which is
correct

a) Musculocutaneous nerve causes elbow flexion


b) Pronation is ulnar nerve
c) Supination is median
d) ?
A (repeat Q-210 exam 2006)

90) A diabetic patient has a persistent foot drop and sensory deficit 48h following
popliteal block. Which of the following is correct?

a) Patient requires CT scan


b) Patient requires surgical exploration
c) Patient should be told to wait 24h and reassess
d) Patient requires electromyography
D
Approach to patient with foot drop,see the link below:
http://www.acnr.co.uk/pdfs/volume3issue1/v3i1anatomy.pdf

also from The Internet Journal of Anesthesiology TM


http://www.ispub.com/ostia/index.php?xmlFilePath=journals/ija/vol8n1/foot.xml :
―Foot drop‖ may be due to a lesion of the common peroneal nerve, L5 radiculopathy or a
partial sciatic nerve lesion, or lesions involving the lumbosacral plexus or cauda equina.
EMG can help detect evidence of denervation in foot drop of recent onset and can also
help in establishing evidence of reinnervation in more chronic lesions.

Ref: miller 6th ed,Ch-44


NEUROLOGIC COMPLICATIONS
Postoperative sensory or motor deficits must also be distinguished from residual
(prolonged) local anesthetic effect. Imaging techniques, such as CT and MRI, are
useful in identifying infectious processes and expanding hematomas. Although
most neurologic complications resolve completely within several days or weeks,
significant neural injuries necessitate neurologic consultation to document the

86
degree of involvement and coordinate further workup. Neurophysiologic testing,
such as nerve conduction studies, evoked potentials, and electromyography, are
often useful in establishing a diagnosis and prognosis.

From e-Medicine:

Diagnostic Procedures:

 Electromyelogram :

o In addition to the metabolic disorders listed above, the differential


diagnosis of spontaneous foot drop includes spasticity, dystonia, motor
neuron disease, L5 radiculopathy, lumbosacral plexopathy, sciatic nerve
palsy, compressive peroneal neuropathy, peripheral neuropathy, and
some myopathies. An electromyelogram (EMG) is useful in
differentiating among these diagnoses.

o This study can confirm the type of neuropathy, establish the site of the
lesion, estimate extent of injury, and predict a prognosis.

91) Amniotic fluid embolus presents with all of the following EXCEPT

a) Acute renal failure


b) Cardiovascular collapse
c) Pulmonary edema
d) Postpartum hemorrhage
A ( see details above in Q-37-2007)

92) Regarding pain transmission, which is TRUE?

a) A-delta fibers are responsible for slow component or burning pain


b) C fibers are responsible for fast component or localized pain
c) A-delta and C neurons contain a wide variety of neurotransmitters
d) Inflammation only occurs peripherally
C (repeat q-2004 Q40)

TYPES OF PAIN (Pharm and Phys Chapter 43)


Two qualitatively different types of pain can be readily appreciated.
1. Fast pain is a short, well-localized, stabbing sensation that is matched to the
stimulus, such as a pinprick or surgical skin incision. This pain starts abruptly
when the stimulus is applied and ends promptly when the stimulus is removed.
Fast pain results from stimulation of small, myelinated type A-delta nerve
fibers with conduction velocities of 12 to 30 m sec -1.
2. The second type of sensation is categorized as slow pain and is characterized
as a throbbing, burning, or aching sensation that is poorly localized and
less specifically related to the stimulus. This pain may continue long after
the removal of the stimulus. Slow pain results from stimulation of more
primitive, unmyelinated type C nerve fibers with conduction velocities of
0.5 to 2 m sec -1.

87
The farther from the brain the stimulus originates, the greater is the temporal distinction
of the two components.

PAIN RECEPTORS
Pain receptors (nociceptors) are naked, afferent nerve endings of
myelinated A-delta and unmyelinated C fibers that encode the occurrence, intensity,
duration, and location of noxious stimuli and signal pain sensation. These receptors
transduce mechanical, thermal and chemical stimuli into action potentials that are
transmitted along their axons to the spinal cord.
Three categories of pain receptors are described in skin. Pain receptors that are
activated by mechanical stimulation and conduct impulses by way of A-delta fibers are
termed mechanosensitive pain receptors. A second type of pain receptors is
represented by mechanothermal receptors that are activated by mechanical and
thermal (greater than 43 C) stimulation. These receptors also conduct impulses by way
of A-delta fibers. A third category, known as polynodal pain receptors, responds to
mechanical, thermal, and chemical stimuli and conduct impulses by way of unmyelinated
C fibers. Chemicals capable of activating these receptors include acetylcholine,
bradykinin, histamine, prostaglandins, and potassium ions.

Peripheral Nerve Afferent Fibers (Barash Chapter 54)


Fiber Subclass Myelin Diameter Conduction Location Function
Class U Velocity
(m/s)
A Alpha + 6-22 30-120 Afferent to & efferent Motor,
from muscles & joints proprioception
beta + 6-22 30-120 Afferent to & efferent Motor,
from muscles & joints proprioception
gamma + 3-6 15-35 Efferent to muscle Muscle tone
spindles
Delta + 1-4 5-25 Afferent sensory Pain, temp., touch
B + <3 3-15 Preganglionic Various autonomic
sympathetic functions
C sC - 0.3-1.3 0.7-1.3 Postganglionic Various autonomic
sympathetic functions
d gamma - 0.4-1.2 0.1-2.0 Afferent sensory Various autonomic
C nerves functions; pain,
temperature, touch

Ref: Miller 5th ed, p495

II. The neural basis for pain (Massachusetts Manual Chapter 37)
C. Primary afferent fibers:
 The majority of nociceptive input to the CNS is carried by C-fibers. The cell
bodies of these primary afferents are located within the dorsal root ganglion. A-
delta-fibers conduct relatively rapidly (5 to 25 m/second), while C-fibers are
have a slower conduction velocity (less than 2 m/second). This difference
is the basis for ―first‖ and ―second‖ pain. The next key step in nerve
transmission occurs at the synapse between these primary afferent fibers and
neurons found within the dorsal horn. Cutaneous pain fibers travel in the sensory
nerves. Visceral afferent pain fibers travel in the parasympathetic and

88
sympathetic nerves (the cell bodies of these are also located in the DRG).
Numerous neuromodulators, including peptides, are located in neurons within the
DRG. Significantly, primary afferent fibers play an important role in hyperalgesia;
for example, substance P and CGRP are both released from the nerve terminals
to potentiate the inflammatory response.
 Neurotransmitters.
 Numerous neurotransmitters are involved in pain pathways.
Essentially, these can be divided into rapidly acting excitatory (aspartate
and kainate) or inhibitory (gamma-aminobutyric acid and glycine) amines
or more slow-acting excitatory (substance P) or inhibitory (enkephalin and
galanin) modulators. Some of these are neurotoxic at high levels (e.g.,
following nerve transection or damage), while others are neuroprotective
(e.g., galanin and enkephalin).
Pain modulation and descending analgesia.
The periaqueductal gray area (located in the midbrain) and raphe magnus (located in the
medulla) are not the only areas involved in pain modulation. Stimulation of specific
thalamic and hypothalamic nuclei also produce analgesia. Key elements within this
system include opioid, central noradrenergic, and serotonergic systems. Receptors for
these mediators have been found within pain modulating areas and, when stimulated,
cause analgesia that is effectively reversed by pharmacologic antagonists. Axons
descend in the dorsolateral funiculus to terminate in the dorsal horn where they directly
(onto STT neurons) or indirectly (via interneurons) modulate afferent input from the
periphery.

93) After left upper lobectomy, on chest X-ray there is LLL collapse. Which of the
following unlikely to be the reason?

a) Cardiac herniation
b) Obstruction of left mainstem bronchus
c) Obstruction of chest tube
d) Disruption of left mainstem bronchus
A ( Repeat Q- 62- 2004 )
Reference: Kaplan and Slinger p. 387
Post-thoracotomy complications:
1. Atelectasis: The chest X-ray [in atelectasis post thoracotomy] may show ―plate-
like‖ atelectasis, or segmental, lobar or total atelectasis. Although the operative
side is more frequently involved, contralateral lung and bilateral involvement
might be seen. Physiotherapy and post-operative suctioning are most important
in patients after left upper lobectomy, because as the left lower lobe rises to
fill the left hemithorax, the long left bronchus tends to kink as it passes
under the aortic arch

2. Lobar torsion: (K&S p. 388) Lobar torsion occurs after resection of pulmonary
tissue when the vessels of a remaining lobe or segment become twisted as the
lung expands. Torsion occurs most commonly after right upper lobectomy
because the loss of anchoring attachments allows mobile pulmonary
parenchyma to twist around its remaining hilar attachment. Lobar torsion can
also occur …after left-side lobectomy with torsion of the remaining lobe.
Typically, the pulmonary and bronchial arteries remain open; however, the low-

89
pressured pulmonary veins collapse, compromising venous outflow.
Engorgement causes massive enlargement of the lung area drained by the
involved veins. The patient could be asymptomatic or could demonstrate
significant atelectasis on the early post-operative chest x-ray. Later the
chest x-ray might demonstrate complete ―white-out‖ of the involved
segment or lobe.
Miller
Pulmonary torsion refers to rotation of the parenchyma on its bronchovascular pedicle
and is thought to be due to increased mobility of a lobe (as may occur to the remaining
lobes in one hemithorax after a lobectomy). Any patient with atelectasis or an expanding
intrathoracic mass and perhaps with severe chest pain after thoracotomy should have
lung torsion added to the differential diagnosis, which also includes intrathoracic
bleeding and progressive atelectasis. Because torsion compromises the pulmonary
vasculature (both arteries and veins), as well as the bronchi, prompt recognition and
surgical intervention are required to avoid the attendant morbidity and mortality (i.e.,
infarction and gangrene).
3. Cardiac Herniation: The heart becomes displaced and entrapped within a
pericardial defect, altering the geometry and obstructing the vena cavae and
cardiac outflow tracts. The edges of the defect compress the myocardium
leading to myocardial ischemia, arrhythmias, and obstruction of ventricular
outflow. Severe restriction in venous return and falling cardiac output are
typical of herniation after right pneumonectomy. Examination can reveal
jugular venous distention, a displaced cardiac impulse, and cardiovascular
collapse. Typically occurs after a large pericardial defect is created, usually
following intrapericardial pneumonectomy (Note the absence of any mention of
atelectasis or lobar collapse as a consequence of cardiac herniation)

4. Pneumothorax: After thoracotomy the surgeon usually places two chest tubes
into the operative hemithorax. One chest tube is placed inferiorly to preferentially
drain blood and fluids, whereas a second tube is placed superiorly to
preferentially vent air. Pneumothorax can develop postoperatively if a chest tube
is inadvertently kinked, clamped off or dislodged.

5. Bronchopleural fistula: Bronchopleural fistula occurs when there is disruption of a


bronchial stump or tracheobronchial anastomosis associated with a large air leak
from the lung. If the air leak exceeds the capacity of the chest tube to evacuate
the air, there will be a persistent pneumothorax.

94) Regarding noise pollution in OR. All are true EXCEPT

a) Background noise level of 75-90 dB common


b) Anaesthesiologists have on average greater hearing loss than others
c) Self-selected music does not improve performance
d) Noise pollution is a health concern
Answer C ( Repeat Q- 65 -2004)

Noise Pollution (Reference: Barash, Chapter 4, page 68)


Noise pollution is quantified by determining both the intensity of the sound in decibels
(db) and the duration of the exposure. The national institute for occupational safety and
health (NIOSH) has established a maximum level for safe noise exposure of 90 db for 8
hours. Furthermore, each increase in noise of 5 db halves the permissible exposure

90
time, so that 100 db is acceptable for only 2 hours per day. The maximum allowable
exposure in an industrial setting is 115 db.
The noise level that we routinely endure in our daily practice is surprisingly close to
what constitutes a health hazard. Continuous background noise at a level of 75 to 90
db is produced by ventilators, suction equipment, music, and conversation.
Superimposed on this are such sporadic and unexpected noises as dropped equipment
and monitor alarms. Resultant noise levels frequently exceed those of a freeway or rock-
and-roll concert.
Excessive levels of noise can have an adverse influence on the anesthesiologist‘s
capacity to perform his or her chores. Noise can interfere with an anesthesiologist‘s
ability to discern conversational speech and even to hear auditory alarms. Mental
efficiency and short term memory are diminished by exposure to excess noise. The
complex psychomotor tasks associated with anesthesiology, such as monitoring
and vigilance, are particularly sensitive to the adverse influences of noise
pollution.
There are also chronic ramifications of long-term exposure to excessive noise in the
workplace. At the very least, noise pollution is an important factor in decreased worker
productivity. At higher noise levels, workers are likely to show signs of irritability and
demonstrate signs of stress such as elevated blood pressure. Ultimately, hearing loss
ensues. In a sample of anesthesiologists less than 55 years of age, median
hearing threshold was worse than for the general population, and the hearing
deficits in the higher frequencies were significant enough to interfere with
detection of some equipment alarms. The specific cause of the anesthesiologist‘s
hearing loss could not be determined from the study data.
Conversely, music can provide beneficial effects as a different kind of ‗background
noise‘. Music proved advantageous as a supplement to sedation for awake patients
during surgery. Self-selected music contributed to reduced autonomic responses
in a group of surgeons and improved performance during a stressful non-surgical
task. The beneficial effects were less pronounced when the music was chosen by a
third party.

95) All are possible with the use of protamine to reverse heparin EXCEPT:

a) Pulmonary hypertension
b) Direct smooth muscle dilator
c) Anaphylaxis
d) Decreases platelets
B (see Q- 77-2004 & Q-105-2001)
Protamine:
Polycationic protamine binds polyanionic heparin: 1.3 mg P for 100 U H
H-P complex: eliminated by the reticuloendothelial system
Anticoagulant: interact with PLT, fibrinogen
Adverse effect of Protamin:
Hypotension 2nd to histamine release if injected rapidly, also binds Ca
Pulm. HTN (complement activation and Thromboxane release)
Allergic reaction (NPH insulin use, fish allergy)

Pharm and Phys, Chapter 27, page 457-8.


Protamine is the specific antagonist of heparin's anticoagulant effect. Protamine is a
strongly alkaline (nearly two thirds of the amino acid composition is arginine) polycationic
low-molecular weight protein found in salmon sperm. The positively charged alkaline

91
protamine combines with the negatively charged acidic heparin to form a stable complex
that is devoid of anticoagulant activity. These heparin-protamine complexes are
removed by the reticuloendothelial system.
Protamine administered intravenously in the absence of heparin interacts with
platelets and proteins, including fibrinogen. These interactions may manifest as an
anticoagulant effect of protamine. Adverse cardiovascular responses to protamine
include (1) hypotension, (2) pulmonary hypertension, and (3) allergic reactions.
Rapid intravenous injection of protamine may be associated with histamine release,
including facial flushing, tachycardia, and hypotension. Indeed, the alkaline characteristic
of protamine makes it predictable that histamine release could occur, especially
following rapid intravenous injection. Nevertheless, injection of protamine over at least 5
minutes is not associated with changes in circulating plasma concentrations of
histamine, and blood pressure is not altered. Furthermore, there is no compelling
evidence that protamine has direct negative inotropic effects. Patients with poor left
ventricular function, however, may be more susceptible to protamine-induced reductions
in blood pressure because compensatory increases in cardiac output to offset peripheral
vasodilation are limited.
The site of intravenous administration may influence the subsequent circulatory changes
evoked by protamine. For example, administration of protamine into the right atrium of
anesthetized dogs is followed by (1) elevations in the plasma concentration of histamine, (2)
increases in cardiac output, and (3) decreases in blood pressure and systemic vascular
resistance. These changes do not occur when protamine is injected into a peripheral vein or the
left atrium. It is speculated that the heparin-protamine complex that evokes the release of
histamine in the lungs is diluted before reaching the lungs when protamine is injected into a
peripheral vein or the left atrium. Despite these animal data, intravenous or intra-atrial injection of
protamine to patients has not been documented to produce different circulatory effects.
In rare cases, protamine neutralization of heparin can result in complement
activation and thromboxane release manifesting as pulmonary vasoconstriction,
pulmonary hypertension, and bronchoconstriction. Pre-treatment with
cyclooxygenase inhibitors such as indomethacin or aspirin blunts the increase in
pulmonary vascular resistance Conzen et al, 1989. Inhibition of NO release does not
play a role in the pulmonary HTN response to protamine.

Kaplan, Cardiac Anesthesia, page 1132-34.


Non-cardiogenic pulmonary edema: Systemic hypotension accompanied by massive
pulmonary capillary leak, accumulation of alveolar fluid, decreased pulmonary
compliance, wheezing and pulmonary edema can occur after CPB. Originally attributed
to protamine, these rare responses occur sporadically at least 20 minutes following
protamine administration. Others have attributed the problem to administration of
banked blood products or other substances. CPB itself activates complement via the
alternate pathway, which can result in leukocyte aggregation, free radical formation and
lung injury.
The delayed non-cardiogenic pulmonary edema seen clinically might arise from
protamine, from the administration of leukocytes that accompany banked blood or from
CPB.
Effects on Cardiac Inotropy: Most human studies document no change in CO with
rapid or slow administration. When volume infusion accompanies protamine, CO rises.
Well-conducted studies in intact organisms reveal no effect of protamine on
contractility in humans or animals.

92
96) Regarding sudden respiratory arrest with spinal anesthesia, which is the most
likely cause?
a) Phrenic nerve paralysis
b) Brainstem hypoperfusion
c) Intercostal paralysis
d) Local anesthetic toxicity
B
Cousin

Arterial blood gas tensions are unaffected during high spinal anesthesia in patients
spontaneously breathing room air.60 Resting tidal volume, maximum inspiratory volume,
and negative intrapleural pressure during maximal inhalation are similarly unaffected.47,51
They remain unaltered despite the intercostal paralysis associated with high thoracic
sensory levels of spinal anesthesia because diaphragmatic activity is unimpaired.
Maximum breathing capacity and maximum expiratory volumes, on the other
hand, are significantly diminished during high thoracic levels of anesthesia, as are
maximum intrapleural pressures during forced exhalation, including coughing.
Pulmonary mechanics during exhalation are impaired because the muscles involved in
forced exhalation, especially the anterior abdominal muscles, are denervated by high
thoracic levels of spinal anesthesia. The effects of high spinal anesthesia on forced
exhalation are of clinical importance in patients with tracheal or bronchial secretions in
whom the ability to maintain clear airways depends on their ability to cough.

The phrenic nerves are unaffected by even midcervical levels of sensory anesthesia
because the level of motor blockade is usually below the level of sensory anesthesia, as
discussed previously. Respiratory arrest owing to phrenic paralysis secondary to
excessively high or ―total‖ spinal is relatively rare. Nor is respiratory arrest caused by the
presence of local anesthetics in ventricular CSF in concentrations adequate to produce
direct depression of medullary respiratory neurons. Even the concentration of local
anesthetic in cisternal CSF during high spinal anesthesia, greater than that in ventricular
CSF, is below the threshold concentration of local anesthetic required to produce
depression of central respiratory neurons when applied directly to the medulla. The
most likely cause of transient respiratory arrest during high spinal anesthesia is
ischemia of medullary respiratory neurons secondary to decreases in blood
pressure and cardiac output severe enough to impair cerebral blood flow.
Medullary ischemia as the cause of apnea during high spinal anesthesia is evidenced by
the fact that respiratory arrest rarely occurs in the absence of hypotension severe
enough to be associated with impending loss of consciousness. Further, restoration of
blood pressure and cardiac output in cases of respiratory arrest during spinal
anesthesia, if done promptly, is associated with immediate return of spontaneous
respirations. This would not happen if the respiratory arrest were caused by
pharmacologic block of the phrenic nerves or central respiratory neurons.

97) All are beneficial in treating post-herpetic neuralgia, EXCEPT?


a) Intrathecal Methylprednisolone
b) Opioids
c) TCA’s
d) NSAID’s

93
D
Reference: UpTodate
TREATMENT OF PHN — A number of modalities have been investigated for the
treatment of postherpetic neuralgia (PHN):

Treatment of PHN — Spontaneous resolution of pain is common in patients whose


postherpetic neuralgia (PHN) has lasted less than six months; the decision to treat these
patients is not based upon significant data [36]. For PHN of longer duration, a complex
regimen of oral, intravenous, and topical agents may be required to achieve adequate
pain relief.

Systematic reviews of the literature have concluded that the treatments shown to be
more effective than placebo for PHN include TCAs, "strong" opioids, gabapentin,
tramadol, pregabalin, topical capsaicin (0.075 percent), and topical lidocaine (5
percent patch) [36,72]. The long-term benefits of most therapies are uncertain, and
side effects are common [36]. Intrathecal methylprednisolone may be of benefit in
patients refractory to the above measures [36,72].

A practice parameter from the American Academy of Neurology similarly recommends


tricyclic antidepressants, gabapentin, pregabalin, opioids, and topical lidocaine patches
as first-line therapies [55].

Based upon the available data and our own experience, we typically use the following
approach:
 Opioids and TCAs appear to provide the most benefit. TCAs are generally the
first drugs of choice because of their similar efficacy and lack of dependency.
However, opioids can be used if TCAs are unsuccessful. These drugs can also
be administered simultaneously, since TCAs may not be immediately effective..
 Anticonvulsant therapy with gabapentin is a reasonable alternative to treatment
with TCAs, or it may be added to TCA or opioid therapy. Other anticonvulsants
also appear to be effective.
 Topical capsaicin may also provide relief, although it is often poorly tolerated.
Topical lidocaine may provide short-term relief.
 Intravenous lidocaine may be effective in patients who do not respond to the
above measures.
 Intrathecal corticosteroid injection can be considered for patients who continue to
have intractable pain despite the above measures. These injections do not work
for trigeminal nerve-related pain.

c) Ref: The management of pain vol 1 2nd ED P260-261


Zoster is more common in the elderly who has higher incidence of longer, more
severe course
Prolonged PHN is very rare in patients < 60 and very common in patients > 60
When PHN develops, its wisest to start with noninvasive therapy like TENS or TCA
+/- phenothiazine. This type of denervation pain is not alleviated successfully by
narcotics which should be used cautiously, if at all.

98) Following Retrobulbar block, all may indicate the onset of brainstem
anesthesia EXCEPT:

94
a) Increased vitreous pressure
b) Apnea
c) Shivering
d) Contralateral amaurosis fugax

ANSWER: A
(question similar to 2004 exam Q14 – exact same reference there is the golden
paragraph here)
TABLE 34-4. COMPLICATIONS OF RETROBULBAR BLOCK
1. Stimulation of oculocardiac reflex arc
2. Retrobulbar hemorrhage
3. Puncture of posterior globe, resulting in retinal detachment and vitreous
hemorrhage
4. Intra-arterial injection, producing immediate convulsions
5. Central retinal artery occlusion artery
6. Penetration of optic nerve nerve
7. Inadvertent brain stem anesthesia brain
8. Inadvertent intraocular injection eye

One case report described the gradual onset of unconsciousness and apnea over 7
minutes without any accompanying seizures or cardiovascular collapse. Hence,
anesthesiologists and ophthalmologists should be exquisitely aware of the possibility of
accidental brain stem anesthesia after retrobulbar block. In a series of 6000
retrobulbar blocks, Nicoll reported 16 cases of apparent central spread of local
anesthesia; respiratory arrest developed in 8 of the 16 patients. Clearly, there is a
continuum of sequelae, depending on the amount of drug that gains entrance to the
CNS and the specific area to which the drug spreads. The protean CNS signs may
include violent shivering; contralateral amaurosis; eventual loss of
consciousness; apnea; and hemiplegia, paraplegia, quadriplegia, or hyperreflexia.
Blockade of cranial nerves VIII–XII results in deafness, vertigo, vagolysis, dysphagia,
aphasia, and loss of neck muscle power.

99) Line isolation monitor may be triggered when:


a) If ungrounded material is used in the OR
b) If the leakage current exceeds preset value
c) If the patient becomes grounded
d) If the electrocautery unit is used without a grounding pad
B

2004 Q239.Regarding the line isolation monitor, all of the following are true,
EXCEPT?
a) Faulty equipment plugged into the wall converts the system to a standard
grounded system.
b) It will alarm when a 2-5 mA leak is detected.
c) The number displayed on the gauge is the total current running on the system
at that time.
d) It continuously monitors the integrity of an isolated power system.

ANSWER C

Barash Ch.8 Golden paragraphs

95
The line isolation monitor (LIM) is a device that continuously monitors the integrity of an
isolated power system. If a faulty piece of equipment is connected to the isolated power
system, this will, in effect, change the system back to a conventional grounded system.
Also, the faulty piece of equipment will continue to function normally. Therefore, it is
essential that a warning system be in place to alert the personnel that the power is no
longer ungrounded. The LIM continuously monitors the isolated power to ensure that it is
indeed isolated from ground, and the device has a meter that displays a continuous
indication of the integrity of the system (Fig. 8-26). The LIM is actually measuring the
impedance to ground of each side of the isolated power system. As previously
discussed, with perfect isolation, impedance would be infinitely high and there would be
no current flow in the event of a first fault situation (Z = E/I; if I = 0, then Z = ¥). Because
all AC wiring and all AC-operated electrical devices have some capacitance, small
leakage currents are present that partially degrade the isolation of the system. The
meter of the LIM will indicate (in milliamperes) the total amount of leakage in the system
resulting from capacitance, electrical wiring, and any devices plugged into the isolated
power system.

The reading on the LIM meter does not mean that current is actually flowing; rather, it
indicates how much current would flow in the event of a first fault. The LIM is set to
alarm at 2 or 5 mA, depending on the age and brand of the system. Once this preset
limit is exceeded, visual and audible alarms are triggered to indicate that the isolation
from ground has been degraded beyond a predetermined limit (Fig. 8-27). This does not
necessarily mean that there is a hazardous situation, but rather that the system is no
longer totally isolated from ground. It would require a second fault to create a dangerous
situation.

100) What opioid has metabolites with CNS toxicity?


a) Morphine
b) Meperidine
c) Fentanyl
d) Sufentanil

101) A decrease in mixed venous oxygen tension is associated with all EXCEPT:
a) Malignant hyperthermia
b) Cyanide toxicity
c) Endobronchial intubation
d) Shift of oxyhemoglobin dissociation curve to the left (it really said
LEFT)
B
2004 Q269.(LOW?) Mixed venous oxygen saturation is associated with all
except:
a) low cardiac output LOW
b) malignant hyperthermia LOW
c) cyanide toxicity HIGH
d) endobronchial intubation LOW
e) shift of oxyhemoglobin dissociation curve to the left Low or no
change

Answer: C

96
Arif: One would think that the left shift will result in more affinity to O2 and hence a
subsequent high mixed venous due to the low O2 availability to tissues, But, Below P50
the curve is very steep and I think there will be a significant reduction in the O2 carrying
capacity below this value, Therfore result in a low mixed venous.
Abdullah: PaO2 is what is effected when the curve shifts , and the PaO2 is not in the
equation.

Figure 388.6 The oxyhemoglobin dissociation curve, relating the partial pressure of
oxygen (O2) in systemic arterial blood (PaO2), in millimeters of mercury, to systemic
arterial saturation (SaO2), in percent, and to the O2 content of systemic arterial blood
(CaO2), in milliliters per deciliter of blood. A normal hemoglobin (Hb) concentration of 15
g per deciliter is assumed, as is an unshifted dissociation curve. The curve descends
steeply below PaO2 values of 50 to 60 mm Hg, indicating severely reduced O2-carrying
capacity by Hb below this PaO2. The lower line represents O2 bound to Hb plus O2
dissolved. Dissolved O2 contributes little to CaO2 at a PaO2 in the normal range. (From
Luce JM, Tyler ML, Pierson DJ. Intensive respiratory care. Philadelphia: WB Saunders,
1984:26, with permission.)

97
Mixed Venous Saturation Formula (Miller p. 1183)

Cardiac Output: (Miller p. 1183) To the extent that arterial oxygen saturation, oxygen
consumption, and hemoglobin concentration remain stable, mixed venous oxygen
saturation may be used as an indirect indicator of cardiac output. For example, when
cardiac output falls, tissue oxygen extraction increases and the mixed venous blood will
become more desaturated.

Left Shift: (Miller p. 1269 fig. 33-9)

(The following is an interpretation of the


oxyhemoglobin dissociation curve). For any
type of saturation, arterial or venous, the
saturation (Arif: this should be Pao2) will drop
with a left shift of the curve by definition.

Cyanide toxicity: Mixed venous pO2 is increased in the presence of cyanide toxicity
indicating paralysis of cytochrome oxidase and inability of tissues to use oxygen. The
presence of signs of cyanide toxicity include elevated mixed venous O2 (pVO2),
requirements for increasing dose (tachyphylaxis) and metabolic acidosis.

Malignant Hyperthermia: …the first systemic effect of MH is increased metabolism


(increased oxygen consumption and CO2 production). The [MH] patient will display
peripheral mottling and, on occasion, sweating and cyanosis. Based on the equation for
SvO2 we can see that increasing oxygen consumption will decrease mixed venous O2.

Endobronchial Intubation: Unfortunately couldn‘t find a reference specifically saying it will


result in decreased SvO2, but we‘ve all seen that this decreases SaO2 slightly which,
according to the above equation, will result in reduced SvO2.

102) Which nerve does NOT need to be blocked for knee arthrotomy:

98
a) Sciatic nerve
b) Obturator nerve
c) Lateral femoral cutaneous nerve
d) Posterior tibial
D
REGIONAL ANESTHESIA
2004 Q299. All nerves need to be blocked for knee arthroscopy EXCEPT:
a) Sciatic nerve
b) Femoral nerve
c) Obturator nerve
d) Lateral femoral cutaneous nerve
e) Posterior tibial nerve

Answer: E

Surgical anesthesia for operative procedures on the knee in which a tourniquet


will be used requires blockade of all four nerves (femoral, lateral femoral
cutaneous, obturator, and sciatic nerves) innervating the leg. Although it is possible
to perform major knee surgery under peripheral nerve blocks, more often a femoral
three-in-one or lumbar plexus (psoas) block is combined with a spinal or general
anesthetic. This is less difficult technically, reduces the amount of local anesthetic (and
associated systemic toxicity), and provides postoperative analgesia for 12–24 hours.
Continuous lumbar plexus and sciatic techniques allow for prolonged postop analgesia.

103) What is not a contributing factor of negative pressure pulmonary edema?


a) Increase of the hydrostatic pressure
b) Acute right ventricular failure
c) Insufficient lymphatic drainage
d) Loss of the expiratory grunt effect

104) What is first line management if there is a bioterrorist attack using nerve
gas?
a) Atropine
b) Thiocyanate
c) Neostigmine
d) Dopamine

A
From Miller 6th, pg. 2504
- nerve agents are members of a very large group of chemical compounds called
organophosphates . Organophosphate anticholinesterase drugs combine with
acetylcholinesterase at the esteratic site to form a stable inactive complex that
does not undergo hydrolysis
- Echothiophate is the only organophosphate anticholinesterase drug used
clinically Echothiophate interacts with both the esteratic and anionic subsites,
thus accounting for its extreme potency

99
- the main action of nerve agents is the anticholinesterase effect against
acetylcholinesterase and butyrylcholinesterase
- both enzyme systems are familiar to anesthesiologists, who inhibit them on a
daily basis using the carbamate anticholinesterase neostigmine to reverse the
action of nondepolarizing neuromuscular blocking agents → thus DON’T give
more …
- inhibition of acetylcholinesterase causes a buildup of acetylcholine at muscarinic
and nicotinic synapses of the cholinergic nervous system

Signs and Symptoms of Organophosphate Overdose/Poisoning


Muscarinic
- miosis - bronchoconstriction
- sweating - vomitting
- glandular hypersecretion - severe diarrhea
- cardiac (incl. bradycardia) - fecal incontinence
- AV block and prolonged QT
Nicotinic
- fasciculation
- paralysis at neuromuscular junction

Treatment of Organophosphate Overdose/Poisoning


Atropine
- antagonistic action against acetylcholine at muscarinic synapse allows control of
muscarinic effects (most severe of which is bradycardia)
- 2mg IV ; paeds 0.02-0.05 mg/kg IV
- every 5-10 minutes until papillary dilatation occurs or HR > 80 bpm
Oximes ( pralidoxime )
- reverse action of the organophosphate at nicotinic receptor → reduces paralysis
Benzodiadepines - to counter CNS spikes and possible convulsions

105) Sign of addiction in colleague


a) Likes to work with residents
b) Meticulous charting
c) Refuses to relieve colleagues
d) Insists on giving narcotics in PACU personally
D 2003/23

106) Low levels of STP in the fetus after maternal injection are due to all of the
following EXCEPT:
a) Placental metabolism
b) Fetal liver metabolism
c) Uneven villous perfusion
d) Dilution in fetal blood volume
A

107) All are seen with hemolytic transfusion reactions except:


a) Rigidity
b) Fever
c) Hematuria
d) DIC

100
A

108) What is the first line treatment for hypotension in HOCM?


a) Phenylephrine
b) Ephedrine
c) Metoprolol
d) Epinephrine

A
Table 32-10. HYPERTROPHIC CARDIOMYOPATHY—HEMODYNAMIC GOALS
P Full, full, full; volume is first prescription for hypotension
A Up, up, up; a pure vasoconstrictor is next prescription for hypotension
C Depression is fine
R Not too slow, not too fast
Rhy Sinus, sinus, sinus; consider pacing pulmonary artery catheter to better control
atrial mechanism

M Usual precautions apply


O2
CPB Avoid inotropes post-CPB; the myocardial disease is still present; try
vasoconstrictors first
Abbreviations are defined in Table 32-2.

109) All are effects of alpha-2 agonists except:


a) Sedation
b) Hypotension and bradycardia
c) Spinal analgesic properties
d) Causes opioid withdrawal

Clonidine

Clonidine is a centrally acting selective partial alpha2-adrenergic agonist (220:1 alpha2


to alpha1) that acts as an antihypertensive drug by virtue of its ability to decrease
sympathetic nervous system output from the CNS. This drug has proved to be
particularly effective in the treatment of patients with severe hypertension or renin-
dependent disease. The usual daily adult dose is 0.2 to 0.3 mg orally.

Other Clinical Uses

Analgesia: Preservative-free clonidine administered into the epidural or subarachnoid


space (150 to 450 µg) produces dose-dependent analgesia and, unlike opioids, does not

101
produce depression of ventilation, pruritus, nausea and vomiting, or delayed gastric
emptying.

Activation of postsynaptic alpha2 receptors in the substantia gelatinosa of the spinal cord
is the presumed mechanism by which clonidine produces analgesia. As such,
clonidine and morphine, when used concomitantly as neuraxial analgesics, do not
exhibit cross tolerance . Hypotension, sedation, and dryness of the mouth may
accompany use of neuraxial clonidine to produce analgesia.

Preanesthetic Medication: Oral clonidine preanesthetic medication (5 µg/kg)


enhances the postoperative analgesia provided by intrathecal morphine plus tetracaine
without increasing the intensity of the side effects from morphine. This same
preanesthetic medication dose of clonidine (a) blunts reflex tachycardia associated with
direct laryngoscopy for intubation of the trachea, (b) decreases intraoperative lability of
blood pressure and heart rate, (c) decreases plasma catecholamine concentrations, and
(d) dramatically decreases anesthetic requirements for inhaled (MAC) and injected
drugs.

A small oral dose of clonidine (2 µg/kg) decreases the incidence of perioperative


myocardial ischemic episodes without affecting hemodynamic stability in patients with
suspected or documented coronary artery disease. Clonidine does not significantly
potentiate morphine-induced depression of ventilation.

Prolonging the Effects of Regional Anesthesia: Addition of clonidine, 75 to 150 µg,


to a solution containing tetracaine or bupivacaine and placed in the subarachnoid space
prolongs the duration of sensory and motor blockade produced by the local anesthetic.
Fetal bradycardia may limit the usefulness of subarachnoid clonidine in obstetrics.

However, clonidine premedication increases the risk of clinically significant bradycardia


and hypotension. Addition of 0.5 µg/kg of clonidine to 1% mepivacaine-containing
solution prolongs the duration of brachial plexus block performed via the axillary
approach.

Treatment of Opioid Withdrawal: Clonidine is effective in suppressing the signs and


symptoms of withdrawal from opioids. It is speculated that clonidine replaces opioid-
mediated inhibition with alpha2-mediated inhibition of CNS sympathetic activity. Similarly,
clonidine‘s inhibition of sympathetic nervous system output may be useful in attenuating
the symptoms associated with cigarette smoking and associated nicotine withdrawal.

Treatment of Shivering: Administration of clonidine, 75 µg IV, stops shivering. This


desirable effect may reflect the ability of clonidine, like volatile anesthetics, opioids, and
propofol, to inhibit central thermoregulatory control. Indeed, drugs that inhibit
thermoregulatory vasoconstriction may cause core hypothermia and inhibit shivering.

Attenuation of Hemodynamic Effects of Ketamine: Oral clonidine premedication, 5


µg/kg administered 90 minutes before induction of anesthesia, attenuates the blood
pressure and heart rate increases that normally follow the administration of ketamine, 1
mg/kg IV.

Mechanism of Action

102
Clonidine stimulates alpha2-adrenergic inhibitory neurons in the medullary vasomotor
center. As a result, there is a decrease in sympathetic nervous system outflow from CNS
to peripheral tissues. Decreased sympathetic nervous system activity is manifested as
decreases in systemic blood pressure, heart rate, and cardiac output. The ability of
clonidine to modify the function of potassium channels in the CNS (cell membranes
become hyperpolarized) may be the mechanism for profound decreases in anesthetic
requirements produced by clonidine and other even more selective alpha2-adrenergic
agonists such as dexmedetomidine. Neuraxial placement of clonidine inhibits spinal
substance P release and nociceptive neuron firing produced by noxious stimulation.

Pharmacokinetics

Clonidine is rapidly absorbed after oral administration and reaches peak plasma
concentrations within 60 to 90 minutes. The elimination half-time of clonidine is
between 9 and 12 hours, with approximately 50% metabolized in the liver whereas the
rest is excreted unchanged in urine. The duration of hypotensive effect after a single oral
dose is about 8 hours. The transdermal route requires about 48 hours to produce
therapeutic plasma concentrations.

Cardiovascular Effects

Homeo-static cardiovascular reflexes are maintained, thus avoiding the problems of


orthostatic hypotension or hypotension during exercise. The ability of clonidine to
decrease systemic blood pressure without paralysis of compensatory homeostatic
reflexes is highly desirable. Renal blood flow and glomerular filtration rate are
maintained in the presence of clonidine therapy.

Side Effects: The most common side effects produced by clonidine are sedation and
xerostomia. Consistent with sedation and, perhaps more specifically, an agonist effect
on postsynaptic alpha2 receptors in the CNS are nearly 50% decreases in anesthetic
requirements for inhaled anesthetics (MAC) and injected drugs in patients pretreated
with clonidine administered in the preanesthetic medication.

Patients pretreated with clonidine often manifest lower plasma concentrations of


catecholamines in response to surgical stimulation and occasionally require treatment of
bradycardia with an IV anticholinergic.

Rebound Hypertension: Abrupt discontinuation of clonidine therapy can result in


rebound hypertension as soon as 8 hours and as late as 36 hours after the last dose.
Symptoms of nervousness, diaphoresis, headache, abdominal pain, and tachycardia
often precede the actual increase in systemic blood pressure. Beta-adrenergic blockade
may exaggerate the magnitude of rebound hypertension by blocking the beta2–
vasodilating effects of catecholamines and leaving unopposed their alpha-
vasoconstricting actions. Likewise, tricyclic antidepressant therapy may exaggerate
rebound hypertension associated with abrupt discontinuation of clonidine therapy.
Indeed, tricyclic antidepressants can potentiate the pressor effects of norepinephrine.

110) All can be done if there is an accidental intra-arterial injection of thiopental


EXCEPT:

103
a) Intra-arterial flush with normal saline
b) Brachial plexus block
c) IV heparin
d) Intra-arterial bicarbonate
D
Intraarterial injection of thiopental usually results in immediate, intense
vasoconstriction and excruciating pain that radiates along the distribution of the artery.
Vasoconstriction may obscure distal arterial pulses, and blanching of the extremity is
followed by cyanosis. Gangrene and permanent nerve damage may occur. It is generally
accepted that the risk of initiating vascular damage with intraarterial thiopental
increases with increasing concentrations of the drug, and the use of a 2.5% solution is
relatively safe .

Mechanism of Damage

The pathogenesis of arterial occlusion, ischemia, and tissue necrosis that may follow
accidental intraarterial injection of thiopental (also thiamylal and methohexital) is not
fully resolved.

Of the many theories advanced, the most likely mechanism of damage seems to be
precipitation of thiopental crystals in the arterial vessels leading to an inflammatory
response and arteritis, which, coupled with the microembolization that follows, eventually
results in occlusion of the distal circulation. After accidental intraarterial thiopental
injection, endothelial cell destruction and subsequent exposure of the intimal surface
may be an important process in decreasing vessel diameter, by attenuating the local
release of nitric oxide and initiating or promoting thrombus formation. The adverse
response is not due to the alkalinity of thiopental as emphasized by the absence of
vascular damage when isotonic solutions buffered to a pH >10 are injected into animal
models. An earlier theory that arterial spasm was responsible for the thiopental-
induced gangrene has been discounted.

Treatment

Treatment of accidental intraarterial injection of a barbiturate includes immediate


attempts to dilute the drug, prevention of arterial spasm, and general measures to
sustain adequate blood flow. Dilution of the barbiturate is best accomplished by
injection of saline through the needle or catheter that still remains in the artery. At the
same time, injection of lidocaine, papaverine, or phenoxybenzamine may be
administered to produce vasodilation. A strategy that uses endothelium-dependent
vasodilation to enhance flow in thiopental-damaged vessels may be less effective than
drugs that are not dependent on an intact and functional endothelium.

If the needle or catheter has been removed from the artery before recognition of the
accidental injection, the injection of a vasodilator drug may be made into a more
proximal artery because the affected artery will be in spasm. Direct injection of heparin
into the artery may be considered. Sympathectomy of the upper extremity produced by
a stellate ganglion block or brachial plexus block may relieve vasoconstriction.
Urokinase may improve distal blood flow after accidental intraarterial injection of
thiopental.

104
111) All are contra-indications to radial arterial line EXCEPT:

a) Buerger’s disease
b) Raynaud’s disease
c) Infection at the site
d) Mediastinal mass
Answer D Coex
Buerger‘s Dx (Thromboangiitis obliterans):
Inflammatory vasculitis leading to occlusion of small and middle size arteries and veins
in the extremities, usually onsets before the age of 45, typically occurs in smokers, Dx
confirmed with biopsy. The Dx presents w/ intermittent claudication of the extremities,
Raynaud‘s phenomenon is assocw/ this Dx, The symptoms are exacerbated by cold
temp. Treatment is to stop smoking, No Surgical or pharmacological option exist.
Anesthetic Considerations
 Avoid arterial cannulation
 Keep OR warm , gasses humidified
 Press points padding on the extremities
 Smokers
 Remove epi from lacal anesthetic solutions ( can accentuate vasospasm)
 Pt may be on vasodilators or steroids

Raynaud‘s Syndrome
Raynaud‘s phenomenon can be primary (called disease ) or it can be secondary to other
Dx such as Scleroderma , SLE , RA and dermatomyositis.
Anesthetic Considerations
 Maintain OR & pt temp
 Low risk of ischemic complications w/ arterial line cannulations , however, a
theoretical risk still exists, so the risk vs benefit ratio should be considered.
 In pts w/ CREST , place a femoral line if an art line is req ( becos it is a larger
vessel)
 Remove Epi from local anesthetic solutions

112) In which one of the following situations does the pulse oximeter
overestimate the SaO2 when hypoxia is present?

a) Anemia
b) Methemoglobinemia
c) Methylene blue
d) Cold extremity
Answer B
Table 36-4 -- Artifacts in pulse oximetry (Miller)
Factor Effect References
Toxic Alterations in Hemoglobin
Carboxyhemoglobin Slight reduction of the assessment of Sao2 by pulse oximetry (Spo2) (i.e.,
[97][98][99][100][101]
(COHb) overestimates fraction of Hb available for O2 transport)

105
Factor Effect References
Cyanmethemoglobin Not reported
Methemoglobin At high levels of MetHb, Spo2 approaches 85%, independent of actual
[102][103]
(MetHb) oxygen saturation (Sao2)
Not reported (affects CO oximetry by producing a falsely high reading of
Sulfhemoglobin [104][105][106]
MetHb)
Structural Hemoglobinopathies
[63]
Hemoglobin F No significant effect
No significant effect (i.e., overestimates fraction of Hb available for O2
Hemoglobin H [111]
transport)
[112][113]
Hemoglobin Köln Artifactual reduction in Spo2 of 8–10%
Hemoglobin S No significant effect
Hemoglobin Replacement Solutions
Diaspirin cross- [114]
No significant effect
linked hemoglobin
Bovine polymerized
hemoglobin (oxygen
[115][116]
carrier-201) No significant effect
Dyes
[62][125]
Fluorescein No significant effect
Indigo carmine Transient decrease
[62][125]
Indocyanine green Transient decrease
Isosulfan blue No significant effect at low dose; prolonged reduction in Spo2 at high
[130][131][132]
(patent blue V) dose
[124][125][126][127][1
Transient, marked decrease in Spo2, lasting up to several minutes;
Methylene blue 28]
possible secondary effects due to effects on hemodynamics
Hemoglobin Concentration
If Sao2 normal: no effect; during hypoxemia, at Hb values less than 14.5
Anemia [120][121]
g/dL: progressive underestimation of actual Sao2
[122][123]
Polycythemia No significant effect
Other Factors
Acrylic fingernails No significant effect
Bright light, particularly if flicker frequency is close to a harmonic of
Ambient light
light-emitting diode switching frequency, can falsely elevate Spo2
interference [64][142]
reading.
Arterial O2 Depends on manufacturer. During hypoxemia, Spo2 tends to be
[5][73][141][303]
saturation artifactually low.
[146][147]
Blood flow Reduced amplitude of pulsations can hinder obtaining a reading or cause a

106
Factor Effect References
falsely low reading.
Red henna: no effect; black henna: may block light sufficiently to
Henna [134]
preclude measurement
No effect. Multi-wavelength laboratory oximeters may register a falsely
Jaundice [135][136][137][138]
low Sao2 and a falsely high COHb and MetHb.
[64]
Motion Movement, especially shivering, may depress Spo2 reading.
Slight decrease in Spo2 reading, with greatest effect using blue nail polish,
Nail polish [133][304]
or no change
"Optical shunting" of light from source to detector directly or by
Sensor contact [145]
reflection from skin results in falsely low Spo2 reading.
Small errors or no significant effect reported. Deep pigmentation can
Skin pigmentation [62]
result in reduced signal.
Transparent tape between sensor and skin has little effect. Falsely low
Tape [139][140]
Spo2 has been reported when smeared adhesive is in the optical path.
[148]
Vasodilatation Slight decrease
Venous pulsation
(e.g., tricuspid
[59]
insufficiency) Artifactual decrease in Spo2

113) Most common neurological sequelae with AIDS

a) Dementia
b) Meningitis
c) Autonomic neuropathy
d) Peripheral neuropathy
Answer D
Refs\HIV in labour.pdf
Central and Peripheral Nervous System. Some 30% of adults and 50% of children
suffering from AIDS will develop neurological disorders (11). In the early stage
of infection, headaches, photophobia, meningoencephalitis, depression, irritability,
Guillain-Barre-like syndromes, or cranial and peripheral neuropathy can be observed.
The latent phase of the disease is associated with demyelinating neuropathy and
cerebrospinal fluid pathology. The late period of HIV infection is associated with
meningitis, focal or diffused encephalopathy, myelopathy, myopathy, and peripheral
neuropathy.
As the central nervous system (CNS) is the first crucial organ to be affected by
anesthetic drugs, early diagnosis of HIV deserves careful evaluation of cognitive
and neurologic dysfunction. Patients with AIDS are more sensitive to opioids and
benzodiazepines, which also reflects the extent of neurological involvement.
The probable mechanism is based on increased interleukin-1 levels causing an
increased _ aminobutyric acid-mediator production (12). HIV infection, intracranial

107
masses, or opportunistic infections may cause cerebral edema, cerebral hemodynamic
disturbances, and increased intracranial pressure (ICP). These deserve anesthetic
consideration and measures for reducing ICP and generally preclude the use of
neuraxial anesthesia in patients with increased ICP. Peripheral neuropathy is the
most frequent neurological complication in HIV patients (13). It affects
approximately 35% of patients with AIDS and manifests clinically as a polyneuropathy
and myopathy. An autonomic dysfunction in the HIV-infected person may appear with or
without CNS abnormalities.
AIDS patients may present with uncommon autonomic disturbances, such as orthostatic
syncope, hypotension, and diarrhea .

Kelly
Dementia
Brain dysfunction has been reported in HIV-1-infected patients throughout all stages of
the disease. An <AIDS> dementia complex has been diagnosed in one-third of
adults and half of children with <AIDS>. Early cognitive changes include inattention
and forgetfulness. With declining mental activity, a global dementia may ensue. In 30%
of asymptomatic HIV-positive patients , CSF abnormalities have been documented.
<AIDS> dementia complex (ADC, also called HIV-associated cognitive–motor complex),
is a subacutely or indolently progressive CNS disorder affecting cognition, motor
performance and behavior.

Polyneuropathies

108
The most common and important polyneuropathy—indeed the most common
neuropathy overall in HIV infection—is a distal sensory polyneuropathy. This disorder
affects axons and their sensory ganglion cell bodies rather than myelin and presents
with paresthesias and pain in the feet. Indeed, the discomfort (―positive symptom‖) of the
sensory nerve involvement exceeds the extent of functional impairment related to
sensory loss or weakness (―negative symptoms‖). In mild form this neuropathy is very
common. In its less common severe form it can be a source of great morbidity related to
intractable, disabling neuropathic pain.

114) In a pregnant patient, the use of intrathecal morphine is associated with all
except:

a) Delayed onset (45-60 mins)


b) Side effects
c) 2nd stage of labor is not prolonged
d) Useful for episiotomy
Answer D
After reviewing Chestnut, A and B are true. I could not find a source for C, but chestnut
does say that a dense motor block can prolong the 2nd stage of labour, but this would not
happen w/ morphine.
Regarding D, in one chapter it says that ―dense anesthesia is required‖ but in another
chapter it says that they do use intrathecal opioids for pts w/ episiotomy….I think they
meant for post procedural pain relief.
Article found by Hani shows that IT morphine actually prolongs the 1st stage and doesnot
effect the 2nd stage, ( From A&A 1991) . Article found by Arif states that IT fentanyl
shortens the 1st stage , becos it speeds up Cx dilatation

115) Pregnant patient in first trimester, which is true about anesthesia for
nonobstetric surgery?
a) TIVA is the method of choice
b) No IV anesthetic agents have been found to be teratogenic
c) Hyperventilation is desirable
d) Risk of miscarriage is tripled
Answer B
Chestnut
―Teratogenecity has not been assoc w/ the use of any of the commonly used induction
agents, including barbiturates , ketamine and Benzos, when they are given in clinical
doses during anesthesia.‖
Maternal Hypercapnia can cause fetal acidosis , if it is mild, it has little clinical
significance, if severe, it would cause fetal myocardial depression and hypotension.
Maternal hyperventilation, w/ low maternal PaCO2, and high pH, can effect fetal
oxygenation by several mechanisms:
Resp and metabolic alkalosis can cause umbilical artery constriction, and can also shift
the Oxy-Hb diss curve to the Lt. Thus Hyperventilation should be avoided in surgical pts,
rather , the PaCO2 should be kept in the normal range for pregnancy (30mmHg from 12
weeks gestation)!! However, the increase is from 12 weeks , that according to Arif , that
is the end of the 1st trimester. So hyperventilation is not necessary, also, Sandra, said
that at term , hyperventilation is not necessary.
Although anesthesia and surgery are assoc w/ increased incidence of abortion, IUGR,
amd perinatal mortality, these can often be attributed to the procedure ,the site of

109
surgery, and the maternal condition. Evidence does not suggest increase in cong
abnormalities or a clear relationship between outcome and type of anesthesia.

116) What effect does Isoflurane have on spontaneous ventilation?

a) Increased respiratory rate & decreased tidal volume


b) Decreased resp rate & increased tidal volume
c) Decreased resp rate & decreased tidal volume
d) Increased resp rate & increased tidal volume
Answer A
Barash

117) Which is correct regarding a p-value <0.05

a) The difference between groups is clinically significant


b) Less than 5% chance that the result is due to chance
c) Not effected by sample size (It is)
d) Also known as beta error
Answer B
Miller
In hypothesis testing we choose a level of significance, termed the alpha value, and
declare that if P is less than alpha, the result is statistically significant and accepted as

110
true. If P is not less than alpha, we reject the hypothesis (equivalent to accepting the null
hypothesis). Remember the logical sequence: we choose alpha in advance, then
compute P from the data. If P is less than alpha, we agree to accept the hypothesis and
we reject the null hypothesis. The P value is the chance that the null hypothesis was true
but, through our bad luck, the data came out as observed anyhow. Typically, we set
alpha equal to .05 and want P less than that. For that P value, it would mean that the
chance of the data coming out this way was less than a probability of 5% (P = .05) if the
null hypothesis were in fact true.
When we pick an alpha value of .05, we are roughly saying that about 5% of the time we
will accept misleading random errors as though they were true. This is roughly
equivalent to recognizing that for every 20 experiments we do, perhaps 1 will be wrongly
accepted as true based solely on bad luck and random chance.

118) Regarding autonomic hyperreflexia (except Q in UWO)


a) It is not found in hemi-cord injury
b) Begins 48h post-injury
c) Most commonly initiated by pelvic visceral afferent stimulation
d) Triggered by tracheal suctioning
Answer either C (A is rare but appears possible) , if except Q then D most likely
AUTONOMIC DYSREFLEXIA IN ACUTE SPINAL CORD INJURY: AN UNDER-RECOGNIZED CLINICAL ENTITY.
DEPARTMENT OF SURGERY, DIVISION OF NEUROSURGERY, UNIVERSITY OF TORONTO, TORONTO,
ONTARIO, CANADA. J NEUROTRAUMA. 2003 AUG;20(8):707-16.
The earliest episode of AD occurred on the 4(th) post-injury day. The trigger
mechanisms for AD were somatic pain, fecal impaction, and abdominal distention.
Although numerous reports emphasize AD as a potential complication of chronic SCI,
our study demonstrates that AD occurs in 5.7% (others say 48%) of patients with acute
SCI above T6. Patients with severe cervical SCI are particularly susceptible to the early
onset of AD.
Although the systolic blood pressure during an episode of AD can reach up to 300 mm
Hg, numerous observers define episode of AD as an increase in systolic blood pressure
by at least 20% associated with heart rate disturbances and at least one of the following:
sweating, chills, piloerection, facial flashing, headache, and blurred vision In fact, both
severity and level of SCI are well recognized in the literature as predisposing factors for
the development of AD (Mathias and Frankel, 1992). Moreover, several observations
emphasize that a SCI at T6 level or above is essential in order to develop AD

Table 3:

Miller
One aspect of care that is frequently overlooked is the effect of tracheal suctioning on
patients with high spinal transection. Given that many of these patients depend on
artificial respiration because of their respiratory muscle paralysis, unopposed vagal
stimuli may contribute to profound bradycardia. This vagal response is particularly
accentuated during hypoxemia.
The autonomic reflex includes a dramatic rise in blood pressure, a marked reduction in
flow to the periphery, and flushing and sweating in areas above the lesion. The patient's
heart rate may decline as a reflex. An additional problem arising from the autonomic
denervation of spinal cord transection is thermogenesis. In these patients, hypothermia
may result from cutaneous vasodilation and the inability to shiver. Similarly,

111
hyperthermia can occur because the normal sweating mechanism is impaired. It is
therefore important to monitor temperature assiduously in these patients during the
course of anesthesia..

Barash

After recovery from spinal shock, 85% of patients exhibit autonomic hyper-reflexia when
there has been complete cord transection above T5. The syndrome can also occur with
injuries at lower levels and is characterized by severe paroxysmal hypertension with
bradycardia, arrhythmias, and cutaneous vasoconstriction below and vasodilation above
the level of the injury. The episode is typically precipitated by distention of a viscus
(bladder or rectum), but can be induced by any noxious stimulus. The lack of supraspinal
inhibition allows the sympathetic outflow below the lesion to react to the stimulus
unopposed. If untreated, the hypertensive crisis may progress to seizures, intracranial
hemorrhage, or myocardial infarction. If autonomic hyper-reflexia occurs, it should be
treated by removal of the stimulus, deepening anesthesia, and administration of direct-
acting vasodilators.

Coex

Appears following spinal shock and is associated with return of spinal cord reflexes
(usually the period of spinal shock abates 1 to 3 weeks after spinal cord transection).
Incidence depends on the level of the transection (e.g. 85% of pts. with transection
above T6 have this reflex, and its unlikely to be associated with transection below
T10)

The Brown-Séquard syndrome is characterized by ipsilateral loss of motor function,


vibration, and proprioception below the level of the lesion with contralateral loss of pain
and temperature sensation beginning approximately two levels below the lesion (show
picture 2 and show picture 3). This pattern of deficits occurs because the spinothalamic
tract travels on the ipsilateral side of the cord before crossing over to the contralateral
side. In addition, ipsilateral loss of pain and temperature sensation may occur at one or
two levels below the lesion. Most cases of Brown-Séquard syndrome are partial cases
with varying degrees of paresis and analgesia [9]. The Brown-Séquard syndrome also
may be associated with Horner syndrome (ptosis, miosis, and anhydrosis) if there is
coexistent injury to the cervical paravertebral sympathetic chain and inferior cervical
(stellate) ganglion

119) Which medication is contraindicated in a patient with MH:

a) Calcium gluconate
b) Sodium bicarbonate
c) Metoprolol
d) Verapamil
Answer D
2005 Q40- Which medication is contraindicated in a patient with MH?
1. Calcium gluconate
2. Sodium bicarbonate
3. Metoprolol
4. Verapamil

112
Answer is 4
Barash 4ed Ch20
Dysrhythmia control usually follows hyperventilation, dantrolene therapy, and correction
of acidosis. Calcium channel blockers should not be used in the acute treatment of
MH. Several studies have shown that verapamil can interact with dantrolene to
produce hyperkalemia and myocardial depression Lidocaine can be safely used to
treat dysrhythmias during an MH crisis.
For Tx of MH see 2005 Refs\Treatment of Malignant Hyperthermia Barash 4ed.doc

120) Most common cause of ER visit following ambulatory surgery

a) Nausea & vomiting


b) Urinary retention
c) Pain
d) Surgical complication
Answer D
Miller
The most common causes for unexpected admission are pain, bleeding, intractable
vomiting, surgical misadventure (e.g., bowel or uterine perforation), more extensive
surgery, urinary retention, or lack of an escort.[11]
The likelihood of an unexpected hospital admission is related more to the type of surgery
performed than to specific patient characteristics or the choice of anesthetic technique.
The frequency of return hospital visits after discharge from ambulatory units is
another useful outcome measure. One published study found that 3% of patients
returned to the hospital after discharge following ambulatory surgery.[13] Bleeding
at the surgical site was the most common reason (42%) for patients to seek
emergency medical attention

Twersky R, Fishman D, Homel P: What happens after discharge? Return hospital visits
after ambulatory surgery. Anesth Analg 1997; 84:319.

Barash
Nausea and vomiting are the most common reasons both children and adults have
protracted stays in the PACU or unexpected hospital admission. Nausea and vomiting
are also the most common adverse effect in patients in the PACU.

113
121) What is the afferent limb of the oculocardiac reflex

a) CNIII – oculomotor nerve


b) CNIV – trochlear nerve
c) CNV – trigeminal nerve
d) CNVI – abducens nerve
Answer C
Miller
Oculocardiac Reflex
The oculocardiac reflex is provoked by pressure applied to the globe of the eye or
traction on the surrounding structures. Stretch receptors are located in the extraocular
muscles. Once activated, stretch receptors will send afferent signals through the short
and long ciliary nerves. The ciliary nerves will merge with the ophthalmic division of the
trigeminal nerve at the ciliary ganglion. The trigeminal nerve will carry these impulses to
the gasserian ganglion, thereby resulting in increased parasympathetic tone and
subsequent bradycardia. The incidence of this reflex during ophthalmic surgery ranges
from 30% to 90%. Administration of an antimuscarinic drug such as glycopyrrolate or
atropine reduces the incidence of bradycardia during eye surgery

122) Which of the following is an excitatory neurotransmitter


a) Glycine
b) Epinephrine
c) Glutamate
d) GABA
Answer C
Arif : All inhibitory neurotransmitters end w/ ―ines‖ plus GABA but not prostagladins
Excitatory Neurotransmitters (found in 1storder neurons central terminals)
L-glutamate, Aspartate, Vasoactive intestinal peptide, Calcitonin, Gene-related
peptide, Neuropeptide Y, Substance P, Prostaglandins

Inhibitory Neurotransmitters
GABA, Enkephalins, Β-endorphins, Norepinephrine, Dopamine, Adenosine,
Somatostatin, Cholinergic agonists (Ach),
(Note about norepinephrine… although it does play a role in brainstem inhibitory
modulation of pain, it also plays a role in the excitatory spinal modulation of pain via the
spinal reflex, which releases it peripherall, in injured tissue, generating more pain.)

Numerous neurotransmitters are involved in pain pathways. Essentially, these can


be divided into rapidly acting excitatory (aspartate and kainate) or inhibitory (gamma-
aminobutyric acid and glycine) amines or more slow-acting excitatory (substance P) or
inhibitory (enkephalin and galanin) modulators. Numerous receptors are present in these
pathways. Some unusual ones include capsaicin-specific receptors and cannabinoid
receptors. Finally, some receptors are present in the peripheral nerve under normal
conditions (e.g., N-methyl-D-aspartate receptors) or following nerve damage (e.g., opioid
receptors).

114
Local synthesis in the traumatized area of the body of histamine, bradykinin, serotonin,
kinins, prostaglandins, and substance P has been shown to facilitate the initiation of
afferent neural stimuli, Many putative neurotransmitters are known, including aspartate,
glutamate, somatostatin, and NO

123) 35yo M with 3week history of progressive dyspnea, presents for


appendectomy. Preinduction ABGs: pH 7.47, pO2 55, pCO2 25, HCO3 16.3.
a) ?
b) ?
c) ?
d) ?
UWO Version
pt 2 mth hx of dysp . . ankle fract. . t, pH7.47, pO2 55, HCO3 16.5 . . . intraop
10cc/kg x10 . .pH 7.25, pCO2 40, HCO3 16.5 No cause of metabloc acidosis found
a. Iatrogenis met. Acid 2o to inapprop vent settings
b. Bicarb loss second to CRF
c. Acute resp alkalos 2o to pain
d. Chronic resp alk 2o to primary pulmonary path
Answer A
Expected compensation for acid base disorders

Disorder PaCO2 change HCO3- change


Acute Respiratory Acidosis ↑ 10 mmHg ↑ 1 mEq/L
Chronic Respiratory Acidosis ↑ 10 mmHg ↑ 3.5 mEq/L
Acute Respiratory Alkalosis ↓ 10 mmHg ↓ 2 mEq/L
Chronic Respiratory Alkalosis ↓ 10 mmHg ↓ 4 mEq/L (some say 5)
Metabolic Acidosis ↓ 1.2 mmHg ↓ 1 mEq/L
Metabolic Alkalosis ↑ 0.7 mmHg ↑ 1 mEq/L

Adverse Consequences of Severe Acidosis (pH < 7.2)

Nervous System
 Obtundation
 Coma

Cardiovascular System
 Impaired cardiac contractility
 Decreased cardiac output
 Decreased arterial blood pressure
 Reentrant cardiac dysrhythmias
 Decreased threshold for ventricular fibrillation
 Decreased responsiveness to catecholamines
 Decrease SVR

Ventilation
 Hyperventilation
 Dyspnea
 Fatigue of muscles of breathing

115
Metabolism
 Hyperkalemia
 Insulin resistance
 Inhibition of anaerobic glycolysis

Adverse consequences of Severe Alkalosis (pH > 7.6)

Nervous System
 Decreased cerebral blood flow → in general vasoconstriction is more
pronounced with respiratory alkalosis
 Seizures
 Lethargy
 Delerium
 Tetany

Cardiovascular System
 Arteriolar vasoconstriction
 Decreased coronary blood flow
 Decreased threshold for angina
 Predisposition to refractory supraventricular and ventricular dysrhythmias
 Hypotension

Ventilation
 Hypoventilation
 Hypercarbia
 Arterial hypoxemia

Metabolism
 Hypokalemia
 Decreased serum ionized calcium concentrations
 Hypomagnesemia
 Hypophosphatemia
 Stimulation of anaerobic glycolysis and production of organic acids and lactic
acid

124) Induced and ventilated with RR10 Vt 10mL/kg, FiO2 0.4. At end of surgery,
had the following ABGs: 7.27/55/40/16.3. What best describes this patient’s
acid-base status?

a) Respiratory alkalosis from primary respiratory disease


b) Iatrogenic induced metabolic acidosis from inappropriate vent settings
c) Renal failure with bicarbonate loss
d) ?
UWO Version

116
pt 2 mth hx of dysp . . ankle fract. . t, pH7.47, pO2 55, HCO3 16.5 . . . intraop
10cc/kg x10 . .pH 7.25, pCO2 40, HCO3 16.5 No cause of metabloc acidosis foind
e. Iatrogenis met. Acid 2o to inapprop vent settings
f. Bicarb loss second to CRF
g. Acute resp alkalos 2o to pain
h. Chronic resp alk 2o to primary pulmonary path

125) In which of the following patients would an ambulatory inguinal hernia


repair be contraindicated?

a) 101 year old patient


b) Morbid obesity and asthma
c) Infant born pre-term now 60 weeks post-conceptual age
d) Patient know to be MH susceptible
Answer B
Miller , Ambulatory Surgery

The risk of complications can be reduced if preexisting medical conditions are under
good control for at least 3 months before the operation. Therefore, the ASA physical
status should not be considered in isolation because the type of surgical procedure, the
anesthetic technique, and a multitude of social factors can also influence decisions
regarding patient suitability.[48] Even morbid obesity (body mass index >35 kg/m2) is no
longer considered an exclusionary criterion for day-case surgery[49]; morbidly obese
outpatients can be successfully managed with a facemask for brief procedures.[50]
However, patients with preexisting cardiovascular (e.g., hypertension, congestive
heart failure, angina) and respiratory (e.g., asthma, chronic obstructive pulmonary
disease) conditions, as well as morbid obesity, had a higher incidence of
perioperative morbidity than did healthy outpatients undergoing similar
procedures on an ambulatory basis.[30] As expected, smoking was associated with an
increased risk of respiratory complications and postoperative would infection in patients
undergoing ambulatory surgery.[51] Interestingly, the presence of obstructive sleep apnea
syndrome was not associated with an increased risk of unanticipated admission to the
hospital.[18]

Susceptibility to Malignant Hyperthermia

MH-susceptible patients can be successfully managed with nontriggering anesthetics


(e.g., local anesthesia) in the outpatient setting (see Chapter 29 ). MH-susceptible
patients should be observed for at least 4 hours postoperatively and may then be safely
discharged home.

Extremes of Age

Although the acceptability of patients at the extremes of age (i.e., <6 months and >70
years) has been questioned, age alone should not be considered a deterrent in the
selection of patients for ambulatory surgery (see Chapter 60 and Chapter 62 ). Many
studies have failed to demonstrate an age-related increase in recovery time or incidence
of complications after outpatient anesthesia.[54] Even the socalled elderly elderly
patient (>100 years) should not be denied ambulatory surgery solely on the basis

117
of age.[55] In fact, elderly patients experience less postoperative pain, dizziness,
and emetic symptoms than their younger counterparts.[56] Nevertheless, elderly
outpatients may experience a higher incidence of perioperative cardiovascular
events, and recovery of fine motor skills and cognitive function is slowed with
increasing age. Therefore, elderly outpatients may require a greater degree of
supervision after discharge than their younger counterparts. Social factors, including lack
of transportation, a responsible escort, or a caretaker at home, may make it difficult for
some elderly patients to undergo ambulatory procedures.

Most studies suggest that the risk is greatest in premature infants younger than 46
weeks' postconceptual age.[59][60] However, some authors have reported that the risk of
apnea may persist until the 60th postconceptual week.[61] Anemia (hematocrit <30%)
independently increases the risk of postoperative apnea in former preterm infants less
than 60 weeks' postconceptual age.[62] Treatment with high-dose caffeine may prevent
prolonged apnea and desaturation in this patient population, but careful postoperative
monitoring is still recommended. No relationship exists between apnea and a history of
necrotizing enterocolitis, respiratory distress syndrome, bronchopulmonary dysplasia, or
intraoperative use of opioid analgesics and muscle relaxants.[64]

Contraindications to Outpatient Surgery

Patients with the following conditions may be at increased risk for postoperative
complications and should be offered the option of overnight hospitalization[29]:

1. Serious, potentially life-threatening diseases that are not optimally managed (e.g.,
brittle diabetes, unstable angina, symptomatic asthma)
2. Morbid obesity complicated by symptomatic cardiovascular (e.g., angina) or
respiratory (e.g., asthma) problems
3. Multiple chronic centrally active drug therapies (e.g., use of monoamine oxidase
inhibitors such as pargyline and tranylcypromine) and active cocaine abuse
because of the increased risk of intraoperative cardiovascular complications,
including death
4. Ex-premature infants less than 60 weeks' postconceptual age requiring general
endotracheal anesthesia
5. Lack of a responsible adult at home to care for the patient on the evening after
surgery

126) By how much will FRC decrease in an ASA I patient under general
anesthesia for total knee replacement ?

a) 0-10%
b) 15-25%
c) 30-40%
d) 45-50%
B (see 2004 Q-6)
Barash 4th ed. p. 808
• FRC decreases 10-15% when healthy, spontaneously breathing individuals lie supine.
• Controlled ventilation further reduces FRC only slightly.

118
• General anesthesia FRC by a further 5-10%, and FRC reaches its nadir within first 10
min of anesthesia.

Supine  10%  ventelated  more drop  GA  20%


|-- 10 min --|

Miller,6th ed,Ch-17
Induction of general anesthesia is consistently accompanied by a significant (15% to
20%) decrease in FRC,which usually causes a decrease in compliance.The maximum
decrease in FRC appears to occur within the first few minutes of anesthesia and, in the
absence of any other complicating factor, does not seem to decrease progressively
during anesthesia. During anesthesia, the reduction in FRC is of the same order of
magnitude whether ventilation is spontaneous or controlled. Conversely, in awake
patients, FRC is only slightly reduced during controlled ventilation.In obese patients, the
reduction in FRC is far more pronounced than in normal patients, and the decrease is
inversely related to the body mass index (BMI).The reduction in FRC continues into the
postoperative period.For individual patients, the reduction in FRC correlates well with the
increase in the alveolar-arterial PO2 gradient during anesthesia with spontaneous
breathing, during anesthesia with artificial ventilation,and in the postoperative period.The
reduced FRC may be restored to normal or above normal by the application of PEEP.

127) Which of the following cardiac anomalies requires endocarditis


prophylaxis?

a) Previous bacterial endocarditis


b) Isolated secundum atrial septal defect
c) Implanted defibrillator
d) VSD repaired greater than 6 months ago
A (see 2004 Q67)

128) After major surgery, level of which of the following hormones will
decrease?

a) Cortisol
b) Insulin
c) ACTH
d) ADH
B

129) 18 yo Pregnant female with acute cocaine intoxication and fetal distress
requires C/S. Which of the following should be avoided ?

a) Metoprolol
b) Esmolol
c) Hydralazine
d) Nitroglycerin
A (see 2005 Q-144)
See ..\..\Toxicology.doc for the considerations
Phamacology and Physiology. 3rd Edition. P178

119
Cocaine toxicity
Treatment. Nitroglycerin has been used to treat cocaine-induced myocardial ischemia.
Although esmolol has been recommended to treat tachycardia due to cocaine overdose,
there is also evidence that beta-blockade accentuates coronary vasospasm in the
setting of acute cocaine overdose. Whether beta-adrenergic blockade is harmful in the
setting of chronic coaine use is not known. Alpha-adrenergic blockade may be
effective in treatment of coronary vasoconstriction due to cocaine, but in the presence of
hypotension this intervention is questionable. IV administration of a benzodiazepine
such as diazepam is effective for control of seizures associated with cocaine toxicity.

Harrison‘s Principle‘s of Internal Medicine p.1410

Cocaine abuse is associated with a variety of life-threatening cardiac complications,


including sudden death, myocarditis, dilated cardiomyopathy, and acute myocardial
infarction (resulting from coronary spasm and/or thrombosis with or without underlying
coronary artery stenosis). Nitrates, calcium channel blockers, and benzodiazepines
have been used to treat cocaine-induced cardiotoxicities; Beta-adrenergic blockers
should be avoided.

130) What is the best strategy for preventing postop acute renal failure ?
a) Maintain spontaneous ventilation
b) Lasix
c) Dopamine 2.5mg/kg
d) IV rehydration with NS
D
Miller,6th ed,Ch-52
Most clinical studies suggest that optimization of systemic hemodynamics, including
maintenance of intravascular volume, is the most effective means of renal protection
during and after aortic cross-clamping.[256][257][258][259] I believe this is clearly the best
prophylaxis against post-operative renal dysfunction. The goal is to achieve a preload
adequate to allow the left ventricle to cope with cross-clamping induced changes in
contractility or afterload and maintain cardiac output.

ACTA ANAESTHESIOLOGICA SCANDINAVICA;VOLUME 50 ISSUE 2 PAGE 255-


256, FEBRUARY 2006
http://www.blackwell-
synergy.com.proxy2.lib.uwo.ca:2048/action/showFullText?submitFullText=Full+Text+HT
ML&doi=10.1111%2Fj.1399-6576.2006.00930.x

The above-cited Cochrane analysis neatly sums up the findings of the many studies
on the use of diuretics, dopamine and its analogues, angiotensin-converting enzyme
(ACE) inhibitors, calcium channel blockers and several hydration fluids to prevent or
treat post-operative renal failure: Other than maintaining normovolaemia,none of the
therapeutic approaches used has appeared to be effective in preventing renal failure in
the post-operative period.

131) Which of the following is least affected by hepatic failure ?

a) Pancuronium
b) Rocuronium

120
c) Cis-atricurium
d) Mivacurium
C ref: miller,6th ed,Ch-13

Patients with hepatobiliary disease may exhibit prolonged blockade with dTc,
pancuronium, doxacurium, vecuronium, rocuronium, and mivacurium.

The influence of hepatobiliary disease on the pharmacokinetics of neuromuscular


blockers is complex. In most studies, hepatic disease is associated with an increased
volume of distribution, and as a result, patients have an apparent resistance to the
effect of dTc, pancuronium, atracurium, and rocuronium. The effect of hepatic disease
on the pharmacokinetics of neuromuscular blockers suggests that initial doses may
need to be greater than for patients with normal hepatic function but that once the
desired level of block has been achieved, subsequent recovery may be slower. This is
illustrated in the case of vecuronium, in which doses up to 0.15 mg/kg have a normal
duration of action but a dose of 0.2 mg/kg has prolonged action

Hepatic disease can alter the elimination of neuromuscular blockers by several


mechanisms. The principal route of metabolism of pancuronium and vecuronium is
deacetylation at the 3-position. This metabolic process is presumed to occur in the
liver because 10% to 20% of the total dose of pancuronium and 40% percent of the
total dose of vecuronium are found in the liver and bile as both parent drug and
metabolite. In hepatic disease, an increased plasma concentration of bile salts can
reduce the hepatic uptake of pancuronium and vecuronium, which may be an
explanation for the decreased clearance of these drugs observed by some
investigators. Excretion of vecuronium is diminished in the presence of decreased
hepatic function. The duration of action of vecuronium is longer in these patients, and
recovery is slower than in young healthy individuals.

132) Regarding Gabapentin ?

a) It raises the nociceptive threshold


b) It acts peripherally to decrease pain perception
c) It acts via binding to GABA receptors
d) It is an effective treatment for post-herpetic neuralgia
D ( repeat 2004 Q-61)

133) An MH negative mother requires an urgent C/S under GA. The father is MH
susceptible. Which is the BEST management?

a) Both succinylcholine and volatile anesthetics are safe.


b) Succinylcholine is safe, but volatiles should be avoided.
c) Volatiles are safe, but succinylcholine should be avoided.
d) Both succinylcholine and volatiles should be avoided.
D
ref; http://www.cja-jca.org/cgi/content/abstract/41/12/1220
Golden paragraph;
In addition to conducting a safe anaesthetic for the pregnant MH patient, it must be
remembered that the infant may also be at risk for MH. Two case reports detail possible

121
MH reactions including elevated CK levels in neonates whose mothers had received
general anaesthesia for Caesarean delivery. Since susceptibility could be inherited from
the father, if a family history suggests MH on the father's side, this should be taken into
account when planning the anaesthetic. If triggering agents which cross the placenta are
used one must watch for signs of neonatal MH.

134) All are hemodynamic effects of positive end-expiratory pressure except ?

a) Increased Left ventricular afterload


b) Decreased Left ventricular preload
c) Increased pulmonary artery pressures
d) Decreased right ventricular preload
A

135) Which of the following increases carbon monoxide production in a circle


circuit anesthetic setup?

a) Soda lyme is worse than baralyme


b) Moisture
c) Increased absorbent temperature
d) Decreased volatile concentration
C ( see 2003 Q-3)

• Carbon monoxide formation reflects the degradation of volatile anesthetics that contain
a CHF2-moiety (desflurane, enflurane, and isoflurane) by the strong bases present in
carbon dioxide absorbents
• increases in intraoperative carboxyhemoglobin concentrations (as high as 30%) have
been attributed to this degradation
CO production increased by :
 Inhaled anesthetic used (des> enflurane>isoflurane>>halothane=sevoflurane
 Dryness of absorbent
 Type of Absorbent (Baralyme> Soda lime)
 Temperature
 Anesthetic concentration
 Low fresh gas flow rates
 prolonged high fresh gas flows that cause dryness of the absorbent
136) In which of the following would a right-sided double lumen tube be
indicated

a) Left upper lobectomy with sleeve resection


b) Left lower lobectomy
c) Left lung lavage
d) Left pneumonectomy
D Miller ;

Device Indication Tube Size Placement and


Confirmation

122
Left-Sided DLT Majority of left or Determined from FOB
right thoracic measurements of
surgical procedures the tracheal width
from chest
radiograph
Right-sided DLT Left bronchus— FOB with guided
distorted anatomy technique
Left
pneumonectomy
Fogarty occlusion Critically ill patient Standard FOB
catheter Small bronchus endotracheal tube
Difficult airway at least 6.0-mm ID
Nasotracheal
intubation
Univent blockers Selective lobar FOB
blockade
Difficult airway
requiring lung
separation
WEB blockers Critically ill patient Standard FOB with guided
Selective lobar endotracheal tube technique
blockade at least 8.0-mm ID
Difficult airway
Nasotracheal
intubation requiring
lung separation

Contraindications to the use of a left-sided DLT are carinal and proximal left main
stem bronchial lesions that could be traumatized by the passage of a left-sided
tube. Such lesions include strictures, endoluminal tumors, tracheobronchial
disruptions, compression of the airway by an external mass, and tenting of the left
main stem bronchus so that the angle of the takeoff from the trachea is
approximately 90 degrees.

One-Lung Anesthesia Update, Semin Cardiothorac Vasc Anesth 2005 :


Right-Sided Endobronchial Tubes
There are many clinical situations in which it may be advantageous to avoid intubation of
the left mainstem bronchus. The only absolute contraindication to right-sided double-
lumen tube use, aside from contraindications to use of a double-lumen tube, is an
anomalous right upper-lobe takeoff from the trachea, which is present in 1 of 250
otherwise normal subjects.Indications for right-sided double-lumen tubes are:
• exophytic tumor that compresses the entrance
of left bronchus,
• intraluminal tumor near entrance of left bronchus,
• left-sided tracheobronchial disruption,
• left bronchus stent,
• descending thoracic aortic aneurysm compressing
the entrance of left bronchus,
• sharp angle of the entrance of left bronchus,
and

123
• unilateral left lung transplantation.

137) The clearest indication for intubation in a patient with croup is ?


a) ?
b) The amount of stridor
c) Cyanosis on room air.
d) Elevated PaCO2
D (repeat 2003 Q65 )

Miller 6th ed Ch-76

Croup is the most common cause of acute upper airway obstruction in an otherwise
healthy child. This viral infection (parainfluenza, influenza, adenovirus) produces
swelling and edema of the tissues in the upper airway, particularly in the immediate
subglottic region. Croup occurs most commonly in young infants and children between
the ages of 3 months and 3 years. Children usually have a few days' history of an upper
respiratory infection followed by the development of hoarseness, a croupy cough, and
possibly stridor. The degree of respiratory distress and the child's ability to compensate
for the increased work of breathing are best assessed by clinical examination.
Infants are initially treated with mist and aerosolized racemic epinephrine to reduce local
mucosal swelling. The use of steroids is both common and controversial. Intubation is
required when the infant can no longer sustain the necessary work of breathing.
The child is intubated with the smallest possible ETT (usually 3.0 mm in a child younger
than 2 years or 3.5 mm in a child aged 2 to 4 years) that will permit suctioning of
secretions. Spontaneous resolution in 3 to 7 days is common. In my experience, the
average duration of intubation is 4 to 5 days. As the trachea grows,
laryngotracheobronchitis becomes an uncommon cause of significant airway obstruction
in children older than 4 years.

Endotracheal intubation is reserved for children with severe symptoms who do not
respond to the previous therapies. This decision should be based on criteria such as
hypercarbia, impending respiratory failure and changes in mental status.

138) Regarding Midazolam in children, which of the following is true?

a) Can be given orally or rectally


b) Is palatable
c) Does not interfere with opioids as pre-med
d) Optimally leads to a sleeping child prior to OR
A
Miller 6th ed,Ch-69
Midazolam is water soluble and therefore not usually painful on intravenous
administration. It should be noted that because of its water solubility, it takes three
times as long to reach a peak electroencephalographic effect as the more fat-soluble
diazepam. The clinical importance of this observation is that one should wait at least
3 minutes between intravenous doses to avoid "stacking" of effect. The short
elimination half-life (≈2 hours) in comparison to diazepam (18 hours) offers an
advantage for use as a premedicant in children. Midazolam is the only
benzodiazepine approved by the Food and Drug Administration for use in neonates;
in this population, the half-life is much longer (6 to 12 hours). In addition, severe

124
hypotension has been reported in neonates after bolus administration, and the
potential for this problem is apparently increased in neonates also receiving fentanyl.
Midazolam is rapidly absorbed after intramuscular (0.1 to 0.15 mg/kg, maximum of
7.5 mg), oral (0.25 to 1.0 mg/kg, maximum of 20 mg), rectal (0.75 to 1.0 mg/kg,
maximum of 20 mg), nasal (0.2 mg/kg), or sublingual (0.2 mg/kg) administration.
Nasal administration is uncomfortable for most children.The major problem with oral
or sublingual administration is the strong aftertaste, but a variety of syrups may be
used to disguise its aftertaste. The sedation achieved is not usually sleep (with doses
up to 3.0 mg/kg rectally), but rather a compliant, happy state.If sleep occurs, a
relative overdose has probably been given. Midazolam must always be used with
caution when administered with narcotics because of the potential for respiratory
depression. One important interaction is that erythromycin, calcium channel blockers,
protease inhibitors, and even grapefruit juice produce a clinically important delay in
midazolam metabolism because of cytochrome P450 inhibition.In this circumstance,
either midazolam should be avoided or the dose reduced by 50%. One further
concern is that with nasal administration there is the theoretical possibility of CNS
toxicity as a result of drug entering the CNS along neural connections (olfactory
nerves). Because neurotoxicity has never been examined and most children cry with
nasal administration, I believe that this route should generally be avoided.

139) A patient with severe COPD requires elbow surgery of 2.5h in duration. The
best regional technique would be?

a) IV regional (Bier) block


b) Infraclavicular block
c) Supraclavicular block
d) Interscalene block
B (having said that supaclavicular is the best for elbow surgery, but higher rates of
pneumothorax which is more risky in COPD ,also interscaline associated with
diaphragmatic paresis. Bier is limited to sx < 90 min duration)

Miller;
Infraclavicular Block

Clinical Applications
Infraclavicular block provides anesthesia to the arm and hand. Blockade occurs at the
level of the cords and offers the theoretical advantages of avoiding pneumothorax while
affording block of the musculocutaneous and axillary nerves. No special arm positioning
is required.

Side Effects and Complications


Because of the necessarily blind approach to the plexus, the risk of intravascular
injection may be increased. Exaggerated medial needle direction may result in
pneumothorax. Other rare complications such as infection and hematoma are
theoretically possible.

Supraclavicular Block
Clinical Applications Indications for supraclavicular block are operations on the elbow,
forearm, and hand. Blockade occurs at the distal trunk-proximal division level. At this
point, the brachial plexus is compact and a small volume of solution produces rapid

125
onset of reliable blockade of the brachial plexus. An additional advantage is that the
block can also be performed with the patient's arm in any position.

Side Effects and Complications


The prevalence of pneumothorax after supraclavicular block is 0.5% to 6% and
diminishes with experience. The onset of symptoms is usually delayed and may take up
to 24 hours. Routine chest radiography after the block is not justified. The
supraclavicular approach is best avoided when the patient is uncooperative or cannot
tolerate any degree of respiratory compromise because of underlying disease. Other
complications include frequent phrenic nerve block (40% to 60%), Horner's
syndrome, and neuropathy. The presence of phrenic or cervical sympathetic
nerve block usually requires only reassurance. Although nerve damage can occur,
it is uncommon and usually is self-limited.

Intravenous Regional Anesthesia (Bier)


For short superficial surgical procedures (<60 minutes) limited to a single extremity, the
intravenous regional (Bier) block technique with 0.5% lidocaine is a simple and reliable
technique. This procedure, which can be used for either upper or lower extremity
surgery, involves the use of a double tourniquet to decrease tourniquet pain. Intravenous
regional anesthesia has been reported to be a more cost-effective technique for
outpatient hand surgery than general anesthesia is.The addition of adjuvants (e.g.,
ketorolac, 15 mg, clonidine, 1 µg/kg, dexmeditomidine, 0.5 µg/kg) will improve the quality
of postoperative analgesia.

Side Effects and Complications


Technical problems with this block include tourniquet discomfort, rapidity of recovery
leading to postoperative pain, difficulty in providing a bloodless field, and the necessity of
exsanguination in the case of a painful injury. Accidental or early deflation of the
tourniquet or use of excessive doses of local anesthetics can result in toxic reactions.
Injection of the drug as distally as possible at a slow rate has been shown to decrease
blood levels and theoretically may increase safety.The use of bupivacaine for
intravenous regional anesthesia has been associated with local anesthetic toxicity and
death and is not recommended. Cyclic deflation of the tourniquet at 10-second intervals
has been shown to increase the time to peak arterial lidocaine levels that may decrease
potential toxicity.Other rare complications associated with this technique include phlebitis
(with 2-chloroprocaine), development of compartment syndrome, and loss of a limb.

140) In adults, the risk factors of pulmonary artery rupture with Swan-Ganz
catheter include all of the following EXCEPT:

a) Anticoagulation
b) Advanced age
c) Heparin-bonded catheters
d) Intraoperative hypothermia
C
Similar 2005 Q-20 & Q-53- 2001
Pulmonary artery rupture is associated with the following except:
o Age > 60 years
b) Balloon > 5 cm out from midline
c) Hypothermia
d) Pulmonary hypertension

126
e) Balloon inflated 1.5 cc for insertion
ANSWER: E

Ref: Miller & Barash


Pulmonary artery rupture occurs in approximately 0.02 to 0.2% of catheterized patients
and appears to be a fatal complication in nearly 50% of cases. Although it may occur in
a variety of monitoring situations, many reported cases have involved patients
undergoing heart surgery requiring cardiopulmonary bypass. Several patient factors
common to this clinical setting have been proposed to increase the risk of this
complication, including hypothermia, anticoagulation, advanced age, and
pulmonary hypertension (Arif adds female , from the new Barash).

Several mechanisms for pulmonary artery injury have been proposed and investigated.
Hardy et al, studying cadaveric specimens, determined that the pressure required to
rupture the pulmonary artery of patients older than 60 years was well within the range of
pressures normally exerted during balloon inflation, suggesting that a balloon forcefully
inflated in a pulmonary artery smaller than the balloon diameter may rupture the vessel
wall. Other proposed mechanisms for pulmonary artery rupture include chronic erosion
by a catheter tip abutting the vessel wall or eccentric balloon inflation that forces the
uncushioned catheter tip through the vessel wall.

From a right internal jugular vein puncture site, the right atrium should be reached when
the PAC is inserted 20 to 25 cm, the right ventricle at 30 to 35 cm, the pulmonary artery
at 40 to 45 cm, and the wedge position at 45 to 55 cm. When other vascular puncture
sites are chosen for catheter placement, additional distance is required, typically an
additional 5 to 10 cm from the left internal jugular and left and right external jugular
veins, an additional 15 cm from the femoral veins, and an additional 30 to 35 cm from
the antecubital veins. These distances serve only as a guide. Waveform morphology
must be verified from the monitor display, and catheter position must be confirmed with a
chest radio-graph. The tip of the PAC should be within 2 cm of the cardiac silhouette on
a standard anteroposterior chest film.

141) Regarding extreme masseter muscle rigidity, all of the following are true
EXCEPT:
a) It occurs in the presence of full twitch depression in the extremities
b) It is associated with a susceptibility to malignant hyperthermia
c) It causes difficulty with bag mask ventilation
d) It should lead to consideration of canceling the intended surgical
procedure
Answer C
Cote 622
Cote page 622 Also see Barash 5ed p531
The administration of SUX always followed by transient increase of master mascle tone,
which relaxes with in sec. Inabilty to open the mouth of the pt. is subjective judgment.
The inability to open the mouth despite loss of train of four is highly suggestive of
master tetany. Furter SUX. dose not results in paralysis of master muscle. It may last
few minutes to 30 min, most of the pt can be ventilated with face mask despit
inability to open mouth, because the chest and the glottis muscle are relaxed.

127
142) Which would lead to a worsening of right-to-left shunting with TOF
a) Halothane
b) Isoproterenol
c) Ketamine
d) Propranolol
B
Japanese circulation journal
Vol.31, No.12(19680115) pp. 1857-1863
Japanese Circulation Society ISSN:00471828
Hemodynamic Effects of Isoproterenol and Propranolol in Tetralogy of Fallot. :
Production and Treatment of Anoxic Spells

Results and Discussion: In tetralogy of Fallot the isoproterenol in-fusion resulted in an


increase of right ventricular systolic pressure and a decrease of pulmonary systolic
pressure. Pulmonary blood flow was decreased and systemic blood flow and right to left
shunt were increased, so arterial oxygen saturation was markedly decreased.

Reference: Coexisting Disease, 4th Edition, page 53-57.


Hypercyanotic Attacks
Hypercyanotic attacks are characterized by sudden spells of arterial hypoxemia
associated with worsening cyanosis, tachypnea, and in some instances loss of
consciousness, seizures, CVA and death.
The magnitude of a right-to-left intracardiac shunt can be increased by: (1) decreased
SVR (2) increased PVR and (3) increased myocardial contractility which accentuates
infundibular obstruction to ejection of blood by the right ventricle.

Induction of anesthesia in patients with TOF is often accomplished with ketamine 1-2
mg/kg IV. The onset of anesthesia after ketamine injection may be associated with
improved arterial oxygenation, presumably reflecting increased PBF due to ketamine-
induced increases in SVR, which can lead to a decrease in the magnitude of the right-to-
left intracardiac shunt. Ketamine has also been alleged to increase PVR which would be
undesirable in patients with a right-to-left shunt. The efficacious response to ketamine in
patients with TOF however suggests that this concern is not clinically significant.

143) All of the following would be negatively affected by increased contractility


EXCEPT
a) Mitral stenosis
b) Mitral valve prolapse
c) IHSS
d) Tetralogy of Fallot
A
A is the least affected , but this will increase the gradient across the valve

LAP – LVEDP = [(cardiac output)/(diastolic time)]2

Therefore, whenever cardiac output increases or the diastolic filling period decreases,
the gradient across the mitral valve is altered by the square of the original changes. This
explains why tachycardia or increases in forward flow, seen classically with pregnancy,
thyrotoxicosis, or infection, can precipitate pulmonary edema. As LAP increases, left
ventricular filling pressure may actually decrease. Thus, the development of atrial

128
fibrillation causes hemodynamic embarrassment, not so much because of the loss of
atrial kick but because of the rapid rate that ensues. We are left with the paradoxical
situation of a patient in pulmonary edema with a relatively empty left ventricle. The
treatment in this situation is therefore not inotropic or vasodilator therapy, but rather
attempts to reduce the heart rate or diagnose and treat the cause responsible for the
increased flow.

144) Best diagnostic test for malignant hyperthermia:


a) Resting CPK levels
b) Ryanodine receptor identification
c) Genetic testing
d) Halothane-Caffeine contracture test
D

145) A patient is undergoing a transurethral resection of the prostate. The


surgeon is using glycine 1.5%. A specific reaction to this irrigating solution is:
a) Hypertension
b) Confusion
c) Convulsions
d) Visual changes
D

146) In a 5 lead EKG set-up, what lead would not be affected by disconnection of
the L arm lead:
a) AVR
b) III
c) AVL
d) II
D

147) All of the following are true regarding scavenging systems, EXCEPT?
a) Passive systems must have a closed interface.
b) Reservoir bags and negative pressure relief valves can be used by
either active or passive systems.
c) Active systems must have an open interface.
d) Open does not have valves

C See 2004 Q236

148) Sudden disappearance of the end-tidal CO2 tracing is LEAST likely in


which of the following:
a) Bronchospasm
b) Dislodgement of the endotracheal tube
c) Cardiac arrest
d) PE
A

129
149) Which local anesthetic has the largest therapeutic index with regard to CNS
toxicity:
a) Chloroprocaine
b) Lidocaine
c) Bupivicaine
d) Ropivicaine
A
Table 17-8 Barash p458
Relative potency for systemic CNS toxicity by local anesthetics and ratio of dosage
needed for CV system:CNS toxicity

Agent Relative Potency for CNS CVS:CNS


Toxicity
Bupivicaine 4.0 2.0
levo-Bupivicaine 2.9 2.0
Chloroprocaine 0.3 3.7
Etidocaine 2.0 4.4
Lidocaine 1.0 7.1
Mepivicaine 1.4 7.1
Prilocaine 1.2 3.1
Procaine 0.3 3.7
Ropivicaine 2.9 2.2
Tetracaine 2.0

130
Figure 4-8. Spectrum of local anesthetic agents. Agents are arranged in approximate
order of increasing toxicity; it should be noted, however, that comparisons of all of the
agents at "equi-effect" concentration, under the same conditions, have not yet been
made in humans.

Copyright © 2001 by Lippincott Williams and Wilkins

Figure 4-9. Relationship of signs and symptoms of local anesthetic toxicity to plasma
concentrations of lidocaine.

150) All are true regarding PEEP except:


a) Decreased work of breathing
b) Improved V/Q matching
c) Decreased lung water
d) ?

C
133-Regarding PEEP:
1. increases WOB
2. worsens V/Q mismatch
3. reduces extravascular lung water
4. increases venous return
Answer ? 3?, (however Hani strongly feels its 1)

Barash 4th, Chapter 56:


If adequate oxygenation cannot be maintained with ↑ inspired O2, mechanical ventilatory
support should be instituted. The supportive benefits of PEEP therapy are well

131
documented. The clinical goals include improvement in arterial oxygenation, ↓WOB &
improvement in ventilation-perfusion inequality. With diffuse lung injury, PEEP improves
FRC, compliance & arterial oxygenation. In addition, PEEP ↓shunting, (dead space
ventilation) & venous admixture. This allows adequate arterial oxygenation with a lower
inspired O2.

Potential adverse effects of PEEP


CVS ↓venous return
↓ventricular filling
↑PVR
interference with subendocardial blood flow
↓LV afterload
altered configuration & compliance of RV & LV
Resp Interstitial emphysema
Pneumothorax
Pneumomediastinum
Other Changes in ICP
Alterations in renal function
Abnormalities in hepatic & GI function

Miller
Positive end-expiratory pressure/continuous positive airway pressure (PEEP/CPAP)
therapy produces two well-described salutary effects on the pulmonary system and thus
on oxygenation: (1) redistribution of extravascular water, and (2) increase in the
functional residual capacity (FRC).. The redistribution of extravascular water leads to
improved oxygenation, lung compliance, and ventilation-perfusion matching. The
increase in FRC results from an increase in the volume of patent alveoli at lower levels
of PEEP and from inflation of previously collapsed alveoli, a process known as alveolar
recruitment, By expanding atelectatic alveoli and increasing the FRC, the proportion of
alveoli that is perfused but not ventilated is decreased—that is, shunt is decreased and
thus oxygenation improved.
CV effects
PPV causes a decrease in cardiac output that can be attributed to at least three
mechanisms: (1) decreased venous return (2) right ventricular dysfunction, and (3)
alteration of left ventricular distensibility. Decreased venous return is generally the most
significant factor causing decreased cardiac output with PPV. Increased intrathoracic
pressure results in decreased end-diastolic volume and stroke volume of both ventricles.

151) Which is least indicative of a serious chest injury?


a) 7th and 8th rib fractures
b) Flail chest
c) Hemothorax
d) Widened mediastinum
A
Q2005-136

152) Which of the following is the most common etiology of hearing impairment
associated with anesthesia?
a) Spinal anesthesia
b) Cardiopulmonary bypass

132
c) Dental anesthesia
d) Nitrous oxide
A
2005-158

153) The volatile that best preserves hypoxic drive at 0.1 MAC is:
a) Desflurane
b) Sevoflurane
c) Halothane
d) Isoflurane
A
2005-161

154) All of the following are useful in preventing venous air embolism except?
a) Use of bone wax
b) Inducing hypovolemia
c) Use of positive pressure ventilation
d) Avoidance of nitrous oxide
B
2005-170

155) Which of the following is associated with Trisomy 21


a) DM
b) Hypothyroidism
c) Cushing’s
d) Addison’s
B
Miller: Anesthesia, 5th Ed., Ch.25
Down syndrome (trisomy 21) occurs 1.5 times in 1,000 live births. It is associated with
congenital cardiac lesions, such as endocardial cushion defects (40%), ventricle septal
defects (27%), patent ductus arteriosus (12%), and tetralogy of Fallot (8%),
necessitating prophylactic antibiotics prior to predictable bacteremic events.

Down syndrome is also associated with upper respiratory infections; with atlanto-
occipital instability (in about 15% of patients, in which it is asymptomatic in most cases)
and laxity of other joints; with thyroid hypofunction (50%); with an increased incidence
of subglottic stenosis; and with enlargement of the tongue. No abnormal responses to
anesthetics or anesthetic adjuvants have been substantiated.

156) Regarding IABP, all of the following are contraindications except:

a) Thrombocytopenia
b) Aortic incompetence
c) Thoracic aortic aneurysm
d) Atherosclerosis
A
Miller;

133
IABP counterpulsation is contraindicated in patients with aortic incompetence and
arterial dissection and is relatively contraindicated in patients with severe
atherosclerosis.

Complication
1-ischemia distal to site off ballon inseration (to which thromboembolism and mechanical
obstruction of the femoral artery may contribute)
2-thromboembolic
3-direct truma to vessel leads to aortic perforation
4-thrombocytopenia
5-ballon ruptures and gas emboli
6-infection

Barash;
The IABP is the only method that decreases myocardial oxygen demand while
increasing supply. Aortic blood volume is moved in a direction ―counter‖ to normal flow.
Immediately prior to systole, the intra-aortic balloon pump deflates, ―removing blood,‖
precipitously reducing blood pressure (afterload reduction), enhancing forward flow, and
reducing MVO2. Proper timing of balloon deflation is necessary to reduce end-diastolic
pressure as much as possible to maximally off-load the ventricle. This blood is then
―returned‖ during diastole as the balloon inflates, elevating aortic diastolic blood pressure
(diastolic augmentation), increasing the gradient for coronary perfusion.

IABP counterpulsation may also have favorable effects on right ventricular function.
Although the mechanisms are complex, they probably include accentuation of RV
myocardial blood flow, unloading of the LV with decreases in LA and PAP and RV
afterload, and improvements in RV mechanical function secondary to changes in LV
performance and ventricular interdependence

Reference: Kaplan, Cardiac Anesthesia, 4th Edition, Chapter 33, page 1196.

134
157) Which constricts coronary arteries?

a) Adenosine
b) Histamine
c) Hyperoxia
d) Acetylcholine
C
2004 Q-1. Which directly dilates coronary arteries?
a) Adenosine
b) Histamine
c) Hyperoxia
d) Ach

ANSWER: A (if an except then choose C)

However, changes in myocardial oxygen demand alter autoregulation. The


involved metabolite, therefore, is oxygen (specifically myocardial oxygen tension, Po2)
acting through mediators such as adenosine. Adenosine-induced vasodilation is
inversely related to arterial diameter, with the greatest dilation occurring in smaller
vessels. Hyperoxia decreases and a low Po2 of 49 mm Hg increases coronary
blood flow and myocardial oxygen consumption independently of changes in
oxygen content or delivery The threshold oxygen tension for autoregulation is 32 mm
Hg. Coronary autoregulation is also closely coupled with coronary venous Po2,
particularly at Po2 <25 mm Hg. Decreased heart rate attenuates autoregulation,
whereas pharmacologic coronary constriction augments it.
The most important regulators of coronary vascular tone are metabolic and
involve multiple pathways. Modulation of basal tone and normal autoregulation is
mediated by glybenclamide-sensitive KATP channels, which are enhanced by adenosine
receptor activation. Other mediators and pathways such as oxygen, potassium, pH,
carbon dioxide, endothelium-derived relaxing factor, prostaglandins, prostacyclin,
histamine, and adenosine triphosphate may also be involved in regulating coronary tone.
Prostaglandin E1 dilates the coronary arteries, probably acting through adenosine as a
mediator. Prostaglandin E2, however, is a coronary vasoconstrictor. Acetylcholine
increases coronary flow. Acting through the H1 receptor, histamine contracts
epicardial coronary arteries, provoking spasm, but the H2 receptor mediates
vasodilation. Histamine also promotes production of prostaglandin in the heart. Although
norepinephrine (or sympathetic cardiac nerve stimulation) causes coronary constriction
through its a effects, the associated increase in myocardial contractility increases
coronary flow. Similarly, vagal stimulation may directly produce coronary vasodilation,
but the associated decrease in heart rate and contractility causes secondary coronary
vasoconstriction.

During controlled hypotension produced by adenosine infusion, there is a marked


reduction in systemic vascular resistance and a modest increase in heart rate.
Cardiac output is enhanced, coronary sinus blood flow is increased, and cardiac
filling pressures are maintained, suggesting the absence of negative inotropic effects
and minimal alterations in venous capacitance and venous return to the heart

Histamine (Pharm and Phys p. 386)

135
The predominant cardiovascular effects of histamine are due to dilatation of arterioles
and capillaries, leading to (1) flushing, (2) decreased peripheral vascular resistance, (3)
decreases in blood pressure, and (4) increased capillary permeability. Vascular dilatation
results from a direct effect of histamine on the blood vessels, mediated by both H-1 and
H-2 receptors independently of autonomic nervous system innervation. Activation of
either type of receptor can evoke maximal vasodilation, but H-1 receptors are activated
at lower concentrations of histamine, producing a rapid onset and transient
vasodilatation compared with a slower onset and more sustained vasodilation in
response to H-2 receptor activation.
In addition to peripheral vasodilatation, histamine can produce inotropic, chronotropic,
and antidromic effects:
o Positive inotropic effects are due to histamine-mediated stimulation of H-2
receptors as well as the ability of histamine to evoke the release of
catecholamines from the adrenal medulla.
o Positive chronotropic effects and the development of cardiac dysrhythmias reflect
direct activation of H-2 receptors by histamine as well as an indirect effect due to
histamine-induced catecholamine release.
o Slowed conduction of cardiac impulses through the atrioventricular node is due to
histamine activation of H-1 receptors.
Changes in the threshold for ventricular fibrillation may be caused by the liberation of
small amounts of histamine that are not detectable as changes in the plasma
concentration. It is conceivable that regional tissue release of histamine could contribute
to cardiac dysrhythmias. Coronary artery vasoconstriction is mediated by H-1
receptors, whereas coronary artery vasodilation is mediated by H-2 receptors .

158) In the pregnant patient, when do gastric motility and pH return to normal?

a) 1-3 days postpartum


b) 10-14 days post partum
c) 3-6 weeks post partum
d) Greater than 6 weeks
A (repeat 2004 Q4) Chestnut p453; ―The incidence of reflux returns to normal by the
second day after delivery.‖

159) Decreased levels of Antithrombin III are associated with all of the following
except:

136
a) Pregnancy
b) Cirrhosis
c) Nephrotic syndrome
d) Recent use of heparin
A
From Kevin‘s cards; physiologic changes of pregnancy
Coagulation: Increased platelet turnover, increased clotting and increased fibrinolysis
→ pregnancy is a state of compensated intravascular coagulation (returns to to
prepregnancy state by 2 weeks postpartum)
 II and V unchanged
 XIII and XI decreased
 all other factors increased

From UpToDate;
PHYSIOLOGY OF ANTITHROMBIN
Antithrombin (AT, formerly called AT III, also known as heparin cofactor I) is a vitamin K-
independent glycoprotein that is a major inhibitor of thrombin and other coagulation
serine proteases, including factors Xa and IXa. Augmentation of its inhibitory activity by
heparin constitutes the basis for the clinical use of heparin.
AT slowly inactivates thrombin in the absence of heparin which is called progressive
antithrombin activity. In the presence of heparin, thrombin or factor Xa is rapidly
inactivated by AT; this is referred to as the heparin cofactor activity of AT.

Acquired antithrombin deficiency — A variety of pathophysiologic conditions lower


the concentration of AT in the blood. There is no evidence that acquired AT deficiency
contributes to increased thrombosis and there are no solid data supporting the use of AT
replacement therapy in these conditions.
There are four general causes of acquired AT deficiency: accelerated consumption,
reduced synthesis, increased losses, and drugs.
Accelerated consumption — Acute thrombosis by itself can transiently reduce AT levels.
During acute disseminated intravascular coagulation (DIC), for example, clotting factors
and inhibitors are consumed faster than they can be replenished. The consumption of
AT is significant in DIC and sepsis, and plasma AT levels predict outcome.
Major surgery is associated with a modest fall in AT levels which reaches a nadir around
the third postoperative day, followed by an increase to normal at the fifth postoperative
day. AT levels do not change substantially during normal pregnancies, but may
decrease significantly in women with pregnancy-induced hypertension,
preeclampsia, or eclampsia.
Reduced synthesis — Lowered AT concentrations occur in patients with liver disease
(primarily cirrhosis) due to diminished protein synthesis. The clinical significance of
this change is uncertain.
Increased excretion — The nephrotic syndrome is a hypercoagulable state associated
with an increased incidence of venous and arterial thrombi. A variety of hemostatic
abnormalities have been described including decreased AT levels due to urinary losses
and perhaps to intravascular consumption. There is, however, no convincing evidence
that AT deficiency is responsible for the tendency to thrombosis in this disorder.
Drugs — Modest reductions in plasma AT concentration are found in users of oral
contraceptives and in individuals receiving estrogen for other purposes.
The administration of heparin decreases plasma AT levels, presumably via
enhanced in vivo clearance.

137
P&P
Decreased <Antithrombin> Concentrations
Paradoxically, patients who receive intermittent or continuous therapy with heparin
manifest a progressive reduction of <antithrombin> activity to values that are
approximately one-third of normal (Marciniak and Gockerman, 1977 -- Marciniak E,
Gockerman JP. Heparin-induced decrease in circulating <antithrombin>-III. Lancet
1977;2:581–584. -- ). Thus, a heparin-induced decrease in the activity of
<antithrombin> may paradoxically increase the thrombotic tendency in humans.
Estrogen-containing contraceptives also decrease concentrations of <antithrombin>;
this is consistent with the clinical impression that the incidence of thromboembolic
episodes is increased in patients who take these drugs. Patients with genetically
determined <low> levels of <antithrombin> have a tendency to develop
thromboembolism and may manifest increased dose requirements for heparin
(Anderson, 1986 -- Anderson EF. Heparin resistance prior to cardiopulmonary bypass.
Anesthesiology 1986;64:504–507. -- ). When heparin resistance is secondary to a
deficiency in <antithrombin, administration of fresh frozen plasma restores the levels to
normal and promotes the anticoagulant effects of heparin.

Barash:
Congenital deficiency of ATIII does occur and is associated with a hypercoagulability
state. ATIII deficiency is inherited as an autosomal dominant disorder in which carriers
have ATIII levels between 40% and 60% of normal. Affected persons have a high
prevalence (30%–80%) of thromboses, which often occur before the fifth decade, are
recurrent, may occur in unusual locations, and are associated with a positive family
history of thrombosis.

Antithrombin III deficiency in pregnancy. Laurie S.


Primary Care Update for OB/GYNS. Volume 6, Issue 4, 8 July 1999, Pages 111-114
Levels of AT-III have been measured during pregnancy. Studies of AT-III levels in
healthy pregnant women have detected no appreciable change in AT-III activity during
each trimester, or during the postpartum period

160) Risk of postop nausea and vomiting include all of the following EXCEPT

a) Female gender
b) Smoking
c) Opioid use
d) History of motion sickness
B

138
Apfel et al. (7) created a simplified risk factor chart identifying four primary risk
factors for PONV in patients receiving balanced inhaled anesthesia: female sex,
nonsmoking status, history of PONV, and opioid use. The incidence of PONV with
the presence of
none, one, two, three, or all four of these risk factors was approximately 10%, 20%,
40%, 60%, and 80%, respectively. This simplified risk score was recently validated in
inpatients

161) Regarding the use of phenol for neurolytic blocks

a) It is hypobaric
b) It is the agent of choice for celiac plexus block
c) It has increased affinity for vascular tissue compared to neural tissue
d) It does not cause meningeal irritation
C
2001Q- 176. Phenol in glycine for neurolytic blockade:
a) is hypobaric compared to CSF
b) agent of choice for celiac plexus blocks
c) lasts 8-10 years
d) higher affinity for vascular tissue than neural tissue
e) drug of choice for celiac plexus block

139
CORRECT ANSWER: D
Also See 2005 Q-146

Cousins: Neural Blockade, Chapter 31


Phenol (Carbolic Acid)
Phenol (carbolic acid) has been used extensively for neurolysis since the 1950s, and
because of an apparent lower incidence of neuritis, has to some degree supplanted
alcohol. Preparations of phenol in water, saline, glycerine, and contrast medium have
been used clinically in concentrations ranging from 3% to 15%. Injectable phenol is not
available commercially and requires preparation by a compounding pharmacist.
Solutions of phenol and water (aqueous phenol) or phenol in glycerine (hyperbaric
phenol) are used most commonly.

Aqueous preparations of phenol are used in most clinical settings except for
subarachnoid administration, in which case glycerine-based solutions are absolutely
indicated. Phenol mixed with glycerine is hyberbaric with respect to CSF, a property that
is essential for controlling its spread in the subarachnoid space. Preparations of phenol
and glycerine are thick and viscid, and so it is difficult to force them through small-caliber
needles.

Perineural injection of phenol is less commonly associated with the burning pain
observed after alcohol injection. Phenol produces a biphasic response characterized by
an initial local anesthetic effect, producing subjective warmth and numbness that gives
way to chronic denervation. The hypalgesia that follows may not be as dense as after
alcohol, and quality and extent of analgesia may fade slightly within the first 24 hours of
administration.

Considerations Common to Subarachnoid Alcohol and Phenol Neurolysis


Despite early suggestions that phenol was associated with preferential effects on small
fibers subserving pain, subsequent animal and autopsy studies have confirmed that the
effects of alcohol and phenol are nonselective, resulting in indiscriminate damage to
nerve fibers, the extent of which is dependent on volume and concentration. The
neurolytic effects of alcohol may be more potent and lasting. Alcohol is preferred to
phenol when the patient cannot lie on his or her painful side. In addition, failed neurolysis
with one agent may be best followed with a trial of treatment with another.

Choice of Drug and Needle


Although controlled trials of alcohol versus phenol are lacking, alcohol (50% to 100%)
would seem to be the agent of choice based on global experience. Alcohol, however,
has the disadvantage of producing severe, though transient, pain on injection and is less
miscible with contrast medium than is phenol. Intoxication is unlikely, except in the case
of accidental intravascular injection, but occasionally an acetaldehyde syndrome is
encountered, though it is usually (not always) innocuous. Phenol (6% to 10%) mixes
readily with contrast medium, but must be prepared by a compounding pharmacist. The
apparently greater affinity of phenol for vascular tissue has been cited as a
theoretic disadvantage, since this might be predicted to correlate with higher
incidences of injury to the spinal cord’s nutrient vessels, but the actual clinical
significance is unknown. Large volumes of phenol may be associated with systemic
and cardiac toxicity, and accidental intravascular injection has been reported to produce
seizures. Phenol may be best reserved for (lower volume) splanchnic block, especially if
high concentrations are used, due to potential systemic toxicity.

140
Barash 4th page 1453 Ch55
Alcohol and phenol are the agents most commonly used for prolonged interruption of
neural function.
There is relatively little difference in overall efficacy between these agents, but there are
major differences in the initial responses. Phenol produces no pain on injection, has
an initial anesthetic effect, and takes about 15 minutes to exert its neurolytic effect.
Alcohol causes significant pain on injection and produces neurolysis promptly. When
used for intrathecal (IT) neurolysis, alcohol is hypobaric, whereas phenol in glycerine,
the usual intrathecal preparation, is hyperbaric.
Loss of motor function and inability to control bowel or bladder function following
neurolysis can be devastating to a patient and will greatly impair the quality of remaining
life

162) A patient develops DI following pituitary surgery. Which is true?

a) It is the result of damage to the anterior pituitary


b) It is transient
c) It occurs 24-48hr later
d) It is treated with Chlorpropamide
B
Co-Existing Disease 4th Ed, Chap.52
Diabetes insipidus results when there is destruction of neurons in or near the supraoptic
and paraventricular nuclei of the hypothalamus. It will not occur when the posterior
pituitary alone is damaged because the transected fibers of the pituitary stalk can still
continue to secrete ADH. Diabetes insipidus, which develops in association with pituitary
surgery, typically is due to trauma to the posterior pituitary and is usually transient.

• chlorpropramide (a sulfonylurea) is an effective treatment for diabetes insipidus


by sensitizing the renal tubules to ADH therefore is useful for nephrogenic DI (not
central DI)

Barash 4thEdition Chapter28,page 769:

After oral endotracheal intubation with a RAE tube, the oropharynx is packed with saline-
soaked gauze to minimize blood pooling in the glottis, esophagus, and stomach.
Intraoperative C-arm fluoroscopy of the skull (lateral views) is used during this
procedure, rendering the patient's head and arms relatively inaccessible once the patient
is draped. Cranial nerve weakness secondary to trauma or stretching and the chosen
anesthetic technique should permit gross visual acuity examination before patient
extubation. If vision is the same or improved, extubation can proceed. If acuity is worse,
further diagnostic studies and emergent decompressive surgery may be required. After
transsphenoidal surgery, the patient will awaken with nasal packing, necessitating
mouth-breathing postoperatively. Therefore, these patients must be fully awake and
following commands before extubation of the trachea.

141
Potential intraoperative complications during transsphenoidal procedures relate to the
anatomic landmarks surrounding the sella turcica.
1-The cavernous sinuses occupy the lateral walls of the sella and contain venous
structures, the internal carotid artery, and cranial nerves III, IV, V, and VI.

2-The optic chiasm, with its associated optic nerves and tracts, lies directly above the
diaphragma sella in front of the pituitary stalk. Visual complications secondary to
damage of the optic nerve or chiasm may occur.
3- Surgical manipulation in the region surrounding the sella can result in the following:
hemorrhage from the venous sinuses or internal carotid artery, arterial spasm or
thrombotic occlusion secondary to arterial manipulation,
4-Venous air embolism if head-up tilt is excessive.

Postoperative Concerns. In the immediate postoperative period after either


transsphenoidal or transcranial procedures, the primary concerns are corticosteroid
coverage and fluid balance. Dexamethasone followed by prednisone is given for 5 days
after surgery or until postoperative testing shows an intact pituitary–adrenal axis. Fluid
balance is assessed by strict attention to hourly fluid intake and output and urine specific
gravity. Development of diabetes insipidus is uncommon during surgery but may occur
early in the postoperative course. Diabetes insipidus is commonly seen during the first
12 hours postoperatively and usually lasts for 2–4 days. Diagnosis is based on the
following: polyuria (2–15 l·day–1), hypernatremia, high serum osmolality (≥300 mOsm·kg–
1
), decreased urine osmolality (200 mOsm·kg–1), and decreased urine specific gravity
(1.005 or less). Other complications of pituitary tumor surgery include CSF rhinorrhea,
hypothalamic injury or stroke, cerebral ischemia, and meningitis. After transsphenoidal
surgery, patients must be carefully monitored in the recovery room for airway obstruction
caused by bleeding and secretions in the pharynx. Frequent neurologic examinations
are performed to note any changes in mental status. Patients who have had an
uncomplicated hospital course after transsphenoidal surgery are often discharged within
5–6 days.

163) At how many days post-partum does anatomic closure of the normal term
infants PDA occur

a) 1-3days
b) 5-7days
c) 10-14days
d) >14days
A
Cote 3rd ed P355;
The ductus arteriosus has functionally (but not anatomically) closed in 58% of Normal
term infants by 2 days of life and in 98% by day 4 .

From eMedicine

Persistence of this fetal structure beyond 10 days of life is considered abnormal.

The ductus arteriosus is normally patent during fetal life. This patency is promoted by
continual production of prostaglandin E2 (PGE2) by the ductus. Prostaglandin
antagonism, such as maternal use of nonsteroidal anti-inflammatory medications, can

142
cause fetal closure of the ductus arteriosus. This can be associated with severe fetal
cardiovascular compromise.

Even though functional closure usually occurs in the first few hours of life, true anatomic
closure, where the ductus loses the ability to reopen, may take several weeks.

Medical Care:

Indomethacin is currently the drug of choice for closure of the ductus. Other studies have
shown equal effectiveness with ibuprofen. The dose used for ibuprofen is 10 mg/kg
bolus followed by 5 mg/kg/d for 2 additional days.

Ibuprofen was initially thought to have less adverse effects such as decreased incidence
of oliguria, gastrointestinal toxicity, and cerebral hypoperfusion. However, the use of
ibuprofen has been shown to increase the incidence of pulmonary hypertension and
chronic lung disease. A Cochrane review has showed no statistically significant
difference in closure between ibuprofen and indomethacin.

Surgical Care:

When performed by an experienced pediatric cardiac surgeon, PDA ligation is a low-risk


procedure with excellent results. This is true even in the smallest premature babies. The
risks include hemorrhage, vessel damage, ligation of the wrong vessel (left pulmonary
artery or aorta), recurrent laryngeal nerve or phrenic nerve damage, or infection.

164) How much does atrial contraction normally contribute to LV filling?

a) 5%
b) 15%
c) 25%
d) 35%
B
Miller 6th ed Ch -32

The atrial contribution to left ventricular end-diastolic volume and pressure is normally
less than 20% , but with diastolic dysfunction, this atrial contribution may approach 50%.
Under these conditions, the wedge pressure a wave will be unusually prominent and will
provide a close estimate of left ventricular end-diastolic pressure, whereas mean wedge
pressure will underestimate left ventricular filling.

165) Regarding IV regional techniques, all are correct EXCEPT

a) Lidocaine 0.5% 3mg/kg


b) Avoid deflating tourniquet for at least 20min following injection
c) Use preservative free lidocaine
d) Tourniquet should only be inflated to a maximum of 250 mmHg
D
Miller 6th ed Ch -32

143
Intravenous regional blocks were first described by a German surgeon, August Bier, in
1908. Early methods involved two tourniquets and the first synthetic local anesthetic,
procaine.

Clinical Applications

The Bier block has multiple advantages, including ease of administration, rapidity of
recovery, rapid onset, muscular relaxation, and controllable extent of anesthesia. It is
an excellent technique for short (<90 minutes) open surgical procedures and for
closed reductions of bony fractures.

Technique

After exsanguination of the arm, the proximal cuff is inflated to approximately 150 mm
Hg more than the systolic pressure, and absence of a radial pulse confirms adequate
tourniquet pressure. The total dose of local anesthetic is based on the patient's
weight, and it is injected slowly (3 mg/kg of 0.5% prilocaine or lidocaine, without
epinephrine). The onset of anesthesia is usually within 5 minutes. When the patient
complains of tourniquet pain, the distal tourniquet, which overlies anesthetized skin, is
inflated, and the proximal tourniquet is released. Data suggest that the use of a single
wide cuff allows use of lower inflation pressures during intravenous regional
anesthesia. The postulated advantage is that the lower pressures will decrease the
incidence of neurologic complications related to high inflation pressures with the
narrow double cuffs.The tourniquet may be safely released after 25 minutes, but the
patient should be closely observed for local anesthetic toxicity for several minutes
after the tourniquet release.

Side Effects and Complications

Technical problems with this block include tourniquet discomfort, rapidity of recovery
leading to postoperative pain, difficulty in providing a bloodless field, and the necessity
of exsanguination in the case of a painful injury. Accidental or early deflation of the
tourniquet or use of excessive doses of local anesthetics can result in toxic reactions.
Injection of the drug as distally as possible at a slow rate has been shown to decrease
blood levels and theoretically may increase safety. The use of bupivacaine for
intravenous regional anesthesia has been associated with local anesthetic toxicity
and death and is not recommended. Cyclic deflation of the tourniquet at 10-second
intervals has been shown to increase the time to peak arterial lidocaine levels that
may decrease potential toxicity.Other rare complications associated with this
technique include phlebitis (with 2-chloroprocaine), development of compartment
syndrome, and loss of a limb.

In general, approximately 3 mg/kg (40 mL of a 0.5% solution) of preservative-free


lidocaine without epinephrine is used for upper extremity procedures. For surgical
procedures on the lower limbs, 50 to 100 mL of 0.25% lidocaine has been used.

UWO version 60 year old man presents for ORIF wrist fracture. In regards iv
regional all true except:
a. Dose of 0.5% Lido of 3mg/kg
b. Must keep cuff inflated 45min

144
c. Must use preservative-free lidocaine
d. Must keep cuff pressure <250mmHg

From NYSORA
The cuffs should be inflated to a pressure of 100 mm Hg above the systolic blood
pressure, or at least 300 mm Hg. The Esmarch is then unwrapped and the extremity is
checked for color (pale skin) and arterial occlusion (absence of the radial pulse).

166) Which is consistent with severe preeclampsia

a) SBP>140
b) DBP>90
c) Proteinuria of 300mg/24h
d) Oliguria <500mL/24h
D
See 2004 Q173. All are definitive diagnostic signs of severe pre-
eclampsia EXCEPT:
a. oliguria (<400cc/24hr)
b. increased systolic arterial pressure (> 160mmHg)
c. increased diastolic blood pressure (> 105 mmHg)
d. proteinuria (> 5 grams / 24 hr)
e. pulmonary edema

Answer: C

2007 UpToDate®
Criteria for severe preeclampsia

New onset proteinuric hypertension and at least one of the following:

Symptoms of central nervous system dysfunction:


Blurred vision, scotomata, altered mental status, severe headache
Symptoms of liver capsule distention:
Right upper quadrant or epigastric pain
Nausea, vomiting
Hepatocellular injury:
Serum transaminase concentration at least twice normal
Severe blood pressure elevation:
Systolic blood pressure >160 mm Hg or diastolic >110 mm Hg on two occasions at
least six hours apart
Thrombocytopenia:
Less than 100,000 platelets per cubic millimeter
Proteinuria:

145
5 or more grams in 24 hours
Oliguria <500 mL in 24 hours
Severe fetal growth restriction
Pulmonary edema or cyanosis
Cerebrovascular accident

167) All will decrease plasma potassium concentration in hyperkalemia


EXCEPT:

a) Kayexalate
b) Epinephrine
c) Sodium bicarbonate
d) Calcium chloride
D

168) A single dose of etomidate should be used with caution in the critically ill
because:

a) Venodilation
b) Myocardial depression
c) Suppression of adrenocortical axis
d) Affects cytochrome p450 system
C
Anaesthesia ;Volume 54 Issue 9 Page 861-867, September 1999
‗Adrenocortical function in critically ill patients 24 h after a single dose of etomidate‘
etomidate was associated with a smaller cortisol increment after ACTH stimulation. This
suggests that there was still some residual biochemical effect on the ability of the
adrenal cortex to respond to ACTH 24 h after a single dose of etomidate.

2007 UpToDate®
ADVERSE REACTIONS SIGNIFICANT
>10%:
Gastrointestinal: Nausea, vomiting on emergence from anesthesia
Local: Pain at injection site (30% to 80%)
Neuromuscular & skeletal: Myoclonus (33%), transient skeletal movements,
uncontrolled eye movements
1% to 10%: Hiccups
WARNINGS / PRECAUTIONS — Etomidate inhibits 11-B-hydroxylase, an enzyme
important in adrenal steroid production. A single induction dose blocks the normal
stress-induced increase in adrenal cortisol production for 4-8 hours, up to 24 hours in
elderly and debilitated patients.
PHARMACODYNAMICS / KINETICS
Onset of action: 30-60 seconds
Peak effect: 1 minute
Duration: 3-5 minutes; terminated by redistribution
Distribution: Vd: 2-4.5 L/kg
Protein binding: 76%;
Metabolism: Hepatic and plasma esterases

146
Half-life elimination: Terminal: 2.6 hours

169) Comparing a highly hydrophobic local anesthetic with a moderately


hydrophobic one, which of the following is true

a) Longer duration of action


b) Faster onset
c) Lower potency
d) ??Something about tissue absorption(Less tissue bounding@UWO
version)
A
Miller 6th ed,Ch-14

Moderately hydrophobic local anesthetics will act more rapidly than either hydrophilic
or highly hydrophobic ones when delivered at the same concentration for the following
reasons. Moderately hydrophilic local anesthetics, such as lidocaine, are less bound
to tissues than very hydrophobic drugs (e.g., tetracaine), but are still more membrane
permeant than very hydrophilic ones (e.g., chloroprocaine). Highly hydrophobic local
anesthetics also have higher intrinsic potencies (see Table 14-2 ); therefore, they are
used in lower concentrations, and their rate of onset is correspondingly reduced.

General principles of Local Anesthetics (Massachusetts Manual Chapter 15)


A. Chemistry.
 The half-life of esters in the circulation is very short (about 1 minute). The
degradation product of ester metabolism is para-aminobenzoic acid.
 Amides. The amide linkage is cleaved through initial N-dealkylation followed by
hydrolysis, which occurs primarily in the liver. Patients with severe hepatic
disease may be more susceptible to adverse reactions from amide local
anesthetics. The elimination half-life for amide local anesthetics is approximately
2 to 3 hours.
B. Mechanism of action
 They have no effect on the resting or threshold potentials but decrease the rate
of rise of the action potential such that the threshold potential is not reached.
 Local anesthetics interact directly with specific receptors on the Na+ channel,
inhibiting Na+ ion influx. The anesthetic molecule must traverse the cell
membrane through passive nonionic diffusion in the uncharged state and then
bind to the sodium channel in the charged state.
 Physiochemical properties of the local anesthetics affect neural blockade.
 High lipid solubility increases the potency.
 Agents with a high degree of protein binding will have a prolonged
duration of effect.
 pKa determines speed of onset of neural blockade.
 Lower pH of the drug solution, by decreasing the proportion of molecules
in the uncharged form, results in a slower onset time of anesthesia.

170) A patient with a history of CAD develops complete heart block under GA.
HR falls to 38bpm and BP 80/60. What is the BEST initial management?

147
a) Epinephrine 50ug IV
b) Isoproterenol 5ug/min
c) Dopamine 5mcg/kg/min
d) Atropine 1 mg
D
Hani; epi, Isoproterenol,and dopamine all have B1 adrenergic stimulation / +ve inotrops
which increase myocardial Oxygen consumption that might precipitate or worsen
ischemia in CAD pt. Isoproterenol is a useful chemical pacemaker in 3rd degree HB until
artificial pacemaker inserted, but by B2 stimulation it increase CO through afterload
reduction (reduce resistance) while dropping BP. Barash 5th ed P308&312

From Circulation ;
Atropine
In the absence of reversible causes, atropine remains the first-line drug for acute
symptomatic bradycardia (Class IIa). IV atropine improved heart rate and signs and
symptoms associated with bradycardia. An initial dose of 0.5 mg, repeated as needed to
a total of 1.5 mg, was effective in both in-hospital and out-of-hospital treatment of
symptomatic bradycardia.5–7 Transcutaneous pacing is usually indicated if the patient
fails to respond to atropine, although second-line drug therapy with drugs such
as dopamine or epinephrine may be successful (see below). Use transcutaneous pacing
without delay for symptomatic high-degree (second-degree or third-degree) block.
Atropine sulfate reverses cholinergic-mediated decreases in heart rate and should be
considered a temporizing measure while awaiting a transcutaneous pacemaker for
patients with symptomatic high-degree AV block. Atropine is useful for treating
symptomatic sinus bradycardia and may be beneficial for any type of AV block at the
nodal level.The recommended atropine dose for bradycardia is 0.5 mg IV every 3 to 5
minutes to a maximum total dose of 3 mg.Doses of atropine sulfate of _0.5 mg may
paradoxically result in further slowing of the heart rate.8 Atropine administration should
not delay implementation of external pacing for patients with poor perfusion.

148
171) Regarding regional anesthesia in neonate, all are true EXCEPT:

a) Conus medullaris ends at L1


b) Hemodynamic instability is uncommon due to limited parasympathetic
influence on cardiac function
c) They require 10 times the dose/kg of local anesthetic to get the same
level of block compared to adults
d) They have more CSF/kg than adults
Answer A
Miller
The conus medullaris. This distal termination of the spinal cord , because of differential
growth rates between the bony vertebral canal and central nervous system (CNS),
varies from L3 in the infant to the lower border of L1 in the adult.
Barash

149
172) Which of the following surgeries require pacemaker interrogation
postoperatively

a) Lithotripsy
b) Thyroid surgery
c) Knee surgery
d) Gyne surgery
Answer B or A

Fleisher et al ACC/AHA 2007 Perioperative Guidelines 51


The electrical current generated by electrocautery can cause a variety of responses
by the implanted device, including the following: 1) temporary or permanent resetting to
a backup, reset, or noisereversion pacing mode (eg, a dual-chamber pacemaker may
be reset to VVI pacing at a fixed rate); 2) temporary or permanent inhibition of
pacemaker output; 3) an increase in pacing rate due to activation of the rate-responsive
sensor; 4) ICD firing due to activation by electrical noise; or 5) myocardial injury at the
lead tip that may cause failure to sense and/or capture. Cardioversion can have similar
effects on pacemaker or ICD function. Although the probability of any of these adverse
interactions occurring has fallen dramatically owing to the almost universal use of bipolar
leads (which greatly reduces the probability of electromagnetic interference) and
improved pacemaker and ICD design, they still may occur. When the patient is not pacer
dependent and/or the cautery is remote and will be administered in brief bursts, and the
operative team can monitor the ECG and pulse oximeter (which allows pulse
determination even when electrical interference by cautery interferes with the ECG), it
may be unnecessary to interrogate the pacer at all. Patients with permanent

150
pacemakers, who are pacemaker dependent, should have their device evaluated within
3 to 6 months before significant surgical procedures, and also after surgery. Significant
surgical procedures include major abdominal or thoracic surgery, particularly when the
surgery involves large amounts of electrocautery. This evaluation should include 1)
determining the type of device, 2) determining whether the patient is pacemaker
dependent for antibradycardia pacing, and 3) determining device programmed settings
and battery status. If a patient is pacemaker dependent, the device should be
reprogrammed to an asynchronous mode during surgery (VOO or DOO), or a magnet
should be placed over the device during surgery. Implantable cardioverter defibrillator
devices should have their tachyarrhythmia treatment algorithms programmed
off before surgery and turned on after surgery to prevent unwanted shocks due to
spurious signals that the device might interpret as ventricular tachycardia or fibrillation

Miller

Patients with pacemakers can be treated safely if the pacemaker is pectorally placed
and the following precautions are observed.[209][210][211][212] Pacemaker programmability
should be established before the treatment, and a programmer should be available to
switch the pacemaker to a nondemand mode should the shock waves interfere with
pacemaker function. Alternative means of pacing should be available. Although most
pacemakers located pectorally are at a safe distance from the blast path, some may be
damaged. Weber and coworkers[209] examined 43 different pacemakers and found that 3
were affected. Dual-chamber pacemakers tend to be more sensitive to interference.
Treatment must be started at the lowest energy level and gradually increased while
observing pacemaker function. Manufacturers of AICDs and lithotripters generally
consider an AICD a contraindication for lithotripsy. Patients with AICDs have been
treated successfully with lithotripsy, however.[211] Transvenous AICDs are less of a
concern than the older abdominally implanted defibrillators. AICD devices should be shut
off immediately before lithotripsy and then reactivated immediately after treatment.

173) Which of the following will cause the greatest change in heart rate in the
transplanted heart?

a) Atropine
b) Pancuronium
c) Ephedrine
d) Neostigmine
Answer D
Bradycardia and/or sinus arrest may occur when an anticholinesterase drug is
administered to a patient with a denervated heart, as accompanies a heart transplant. It
is possible that denervation hypersensitivity may have rendered the sino-atrial nodes
of these heart transplant patients exquisitely sensitive to neostigmine despite the
concomitant administration of an anticholinergic drug

151
Table 52-18. ALTERED RESPONSES TO COMMON CARDIOVASCULAR DRUGS
AFTER CARDIAC DENERVATION
Drug Response

Atropine No vagolytic effect


Pancuronium No vagolytic effect
Edrophonium No vagotonic effect
Ephedrine Less cardiostimulatory effect
Nifedipine No depression of nodal conduction
Digoxin No acute vagotonic effect
Norepinephrine Enhanced ß-stimulatory effect
Phenylephrine Diminished vasoconstrictive effects with long-standing heart
failure

174) Which is correct regarding management of traumatic rupture of the


thoracic aorta?

a) Most involve the ascending aorta


b) Place arterial line in left radial artery
c) Use of vasodilators and beta-blockers to control hemodynamics
d) NG tube deviates to the Left
Answer C
Barash
It occurs most frequently in the proximal portion of the descending aorta; injury to the
ascending aorta and the arch is much less likely. Clinical findings are not always
detectable in the emergency department (Table 48-8). A widened mediastinum on the
chest radiograph should prompt a search for this injury. This sign is a reflection of
mediastinal hemorrhage due to aortic injury in only 20% of cases. Aortography is the
gold standard, but TEE and Spiral CT are reliable.
Although the aorta should be repaired as early as possible, control of active hemorrhage
from other sites and surgery for intracranial hematomas have a higher surgical priority,
unless the aorta is leaking.191 In most instances, a blood clot between the aorta and the
mediastinal pleura occludes the vessel. Thus, during management of patients with aortic
injury, every effort should be made to prevent hypertension
Table 48-8. COMMON CLINICAL, RADIOGRAPHIC, AND ULTRASOUND FEATURES OFTHORACIC
AORTIC INJURIES
Spiral Computed
Clinical Radiographic Tomography Ultrasound
Increased arterial pressure and pulse Widened mediastinum Mediastinal Intimal flap
amplitude in upper extremities Blurring of the aortic hematoma Turbulent flow
Decreased arterial pressure and pulse contours Aortic wall irregularity
amplitude in lower extremities Widened paraspinal Intimal flap
Absent or weak left radial artery pulse interfaces

152
Osler's sign: discrepancy between left Opacified pulmonary
and right arm blood pressure window
Retrosternal or interscapular pain Broadened paratracheal
Hoarseness stripe
Systolic flow murmur over the Displacement of the left
precordium or medial to the left mainstem bronchus
scapula Rightward deviation of
Neurologic deficits in the lower the esophagus and
extremities trachea
Left hemothorax
Sternal or upper rib
fractures

175) Thallium scan would be most useful for which one of the following:

a) Identification of myocardium at risk


b) To look at abnormal wall motion
c) ?
d) ?
Answer A

176) By how much has the hydrogen ion concentration changed if the pH goes
from 6.38 to 7.38

a) Increased 10x
b) Increased 100x
c) Decreased 10x
d) Decreased 100x
Answer C
Barash
The conventional approach to describing acid–base equilibrium is the Henderson-
Hasselbach equation:

where 6.1 is the p Ka of carbonic acid and 0.03 is the solubility coefficient in blood of
carbon dioxide (CO2).

This equation clearly expresses the relationship between the three major variables
measured or calculated in blood gas samples. Conversion of pH to [H+] can be
accomplished by knowing that the [H+] is 40 mmol·l–1 at a pH of 7.4; that an increase in
pH of 0.10 pH units reduces [H+] to 0.8 times the starting [H+] concentration; that a
decrease in pH of 0.10 pH units increases the [H+] by a factor of 1.25; and that small
changes (i.e., <0.05 pH units) produce approximately a 1.0 mmol·l–1 increase in [H+] for
each 0.01 decrease in pH or a decrease in [H+] of 1.0 mmol·l–1 per 0.01 increase in pH.

153
177) When using a variable bypass vaporizer, which of the following is
increased by an increase in altitude (decrease in barometric pressure).
a) Vapor pressure
b) Partial pressure of anesthetic in vaporizer chamber
c) Vaporizer output in volume%
d) Saturated vapor pressure
Answer C Explanied beautifully in Eisenkraft p69 Also see Table in p576 Barash 5ed
When the altitude increases 2 things increase :
 Output conc in percent
 Partial pressure ( not a sig increase)
The reason is that at hypobaric conditions more anesthetic vapour is picked up by
the gas flowing through the vaporizing chamber even though the Splitting ratio and
SVP does not change . The opposite occurs in Hyperbaric chambers.

For the Des tec 6 Vaporizer, it is different as this vaporizer will produce a constant
V/V % , not a constant ( or really a kind of relatively constant Partial pressure)
The required dial setting may be calculated using the following formula:75
Required dial setting

= Normal dial setting (vol%)× (760 mm Hg)/(ambient pressure (mm Hg))

For example, at an altitude of 2000 m (6564 feet), where the ambient pressure is 608
mm Hg, the operator must advance the concentration control dial from 10 to 12.5% to
maintain the required anesthetic partial pressure.75 In hyperbaric settings, the operator
must decrease the dial setting to prevent delivery of an overdose.

178) Which of the following is correct regarding administration of sodium


bicarbonate?

a) Can cause paradoxical CNS alkalosis


b) Can cause hyponatremia
c) Can cause severe respiratory acidosis
d) Can cause post-resuscitation metabolic acidosis
Answer C
(Barash 3rd ed. p.1401)
Miller 5th Ed p.2547
• adverse effects of excessive sodium bicarbonate administration are well documented:
metabolic alkalosis, hypernatremia, and hyperosmolarity were common after
administration of bicarbonate during resuscitation attempts .
• sodium bicarbonate combines with hydrogen ions to produce carbonic acid that
dissociates into CO2 and water, PCO2 in blood is temporarily elevated until the excess
CO2 is eliminated through the lungs. Carbon dioxide readily diffuses across cell
membranes and the BBB, whereas bicarbonate diffuses much more slowly.
It is possible that sodium bicarbonate administration could result in a paradoxical
worsening of intracellular and cerebral acidosis by further raising intracellular and
cerebral CO2 without a balancing increase in bicarbonate, direct evidence for this has
not been found, therefore it remains a concern primarily on theoretical grounds

-Adverse effect of NaHCO3 in arrest include : severe plasma hyperosmolality,


paradoxic CSF acidosis, and CO2 generation

154
-in light of all these observations, sodium bicarbonate should NOT be used routinely in
treatment of cardiac arrest

 hypernatraemia
 hyperosmolality
 volume overload
 rebound or ‗overshoot‘ alkalosis
 hypokalaemia
 impaired oxygen unloading due to left shift of the oxyhaemoglobin dissociation
curve
 acceleration of lactate production by removal of acidotic inhibition of glycolysis
 CSF acidosis
 hypercapnia

179) Calculate the ejection fraction in a patient with mitral regurgitation if the
stroke volume is 250 mL, regurgitant volume is 175 mL, and end-diastolic
volume is 500 mL.

a) 15%
b) 30%
c) 45%
d) 50%
The answer is D
Ejection Fraction = End Diastolic Volume – End Systolic Volume / End Diastolic Volume
Ejection Fraction = Stroke Volume / End Diastolic Volume
EF = 250/ 500
EF = 50%

180) Which is true regarding anatomic dead space?

a) It is determined by the Bohr equation


b) It is the major portion of physiologic dead space
c) Comprised of trachea and mainstem bronchi
d) It is increased in pulmonary embolism
Answer B

Physiologic dead space consists of anatomic and alveolar dead space. Anatomic dead
space ventilation, approximately 2 ml·kg–1 ideal body weight, accounts for the
majority of physiologic dead space.86 It arises from ventilation of structures that
do not exchange respiratory gases: the oronasopharynx to the terminal and
respiratory bronchioles. Clinical conditions that modify anatomic dead space include
tracheal intubation, tracheostomy, and large lengths of ventilator tubing between the
tracheal tube and the ventilator Y-piece.

Alveolar dead space ventilation arises from ventilation of alveoli where there is little or no
perfusion. Because disease changes anatomic dead space little, physiologic dead space
is primarily influenced by changes in alveolar dead space. Rapid changes in physiologic
dead space ventilation most often arise from changes in pulmonary blood flow, resulting

155
in decreased perfusion to ventilated alveoli. The most common etiology of acutely
increased physiologic dead space is an abrupt decrease in cardiac output.87 Another
pathologic condition that interferes with pulmonary blood flow is pulmonary embolism,
whether due to thrombus or to fat, air, or amniotic fluid.

Figure 29-6. Distribution of ventilation, blood flow, and ventilation–perfusion ratio in the normal,
upright lung. Straight lines have been drawn through the ventilation and blood flow data. Because
blood flow falls more rapidly than ventilation with distance up the lung, ventilation–perfusion ratio
rises, slowly at first, then rapidly.

The most quantitative technique used to measure physiologic dead space utilizes a
modification of the Bohr> <equation:

where PECO2 is the PCO2 from the mixture of all expired gases over the period of time
during which measurements are made. This calculation estimates the fraction of each
breath that does not contribute to gas exchange. In spontaneously breathing patients,
normal VD/Vt is between 0.2 and 0.4, or ∼0.33. In patients receiving positive-pressure
ventilation, VD/Vt becomes ∼0.5

181) What would the flow-volume loop look like from a patient with
tracheomalacia following resection of a cervical goiter?

a) Normal inspiratory and expiratory flow

156
b) Decreased inspiratory flow, normal expiratory flow
c) Decreased expiratory flow, normal inspiratory flow
d) Decreased inspiratory and expiratory flow
Answer B

182) A patient with pheochromocytoma has received adequate medical


optimization prior to surgery when:

a) Decreased urinary VMA levels


b) Decreased plasma volume
c) Hematocrit decreases
d) Decreased serum glucose
Answer C
Barash 4ed CD
Preoperative Preparation.The reduction in perioperative mortality rates from a high of
45% to between zero and 3% with the excision of <pheochromocytoma> followed the
introduction of alpha -antagonists for preoperative therapy. Perioperative blood pressure
fluctuations, myocardial infarction, congestive heart failure, cardiac dysrhythmias, and
cerebral hemorrhage all appear to be reduced in frequency when the patient has been
treated before surgery with a blockers and the intravascular fluid compartment has been
re-expanded. During this time, the contracted intravascular volume and hematocrit return
toward normal and the blood pressure is stabilized. Despite the real possibility of
hypotension after vascular isolation of the tumor, most clinicians continue a blockers up
until the morning of surgery.

157
b-Adrenergic blockade is often added after a blockade has been established. This
addition is considered in patients with persistent tachycardia or cardiac dysrhythmias
that may be exacerbated by a blockade. b Blockers should not be given until adequate a
blockade is ensured to avoid the possibility of unopposed a-mediated vasoconstriction

183) All are true about LMW heparin compared to unfractionated heparin,
EXCEPT

a) Decreased platelet antibody formation


b) Decreased dosing frequency
c) Decreased requirements for monitoring aPTT levels
d) Decreased interaction with antiplatelet agents
Answer D

184) The beneficial effects of Heliox in patients with upper airway obstruction is
due to:

a) Reductions in viscosity of gas mixture


b) Reduction in density of gas mixture
c) Bronchodilation
d) Decreased airway edema
Answer B
Miller

Stridor is noisy inspiration from turbulent upper airway gas flow. It is frequently seen with
airway obstruction and always commands attention. Wherever possible, attempts should
be made immediately to establish the cause of the obstruction. Endoscopy can be
particularly useful.

The first issue of clinical concern in the setting of stridor is whether intubation is
immediately necessary. If intubation can be delayed for a period, a number of potential
options can be considered:

• Expectant management with oxygen by facemask and positioning the head up at


45 to 90 degrees
• Nebulized racemic epinephrine
• Dexamethasone (Decadron), 4 to 8 mg intravenously every 8 to 12 hours, in
patients with airway edema
• Heliox (e.g., 70% helium, 30% oxygen)

Heliox has been used for decades for the treatment of acute airway obstruction.[98] The
usual respiratory airflow is laminar, and resistance is dependent on gas viscosity. In the
setting of airway obstruction, however, airflow becomes turbulent and dependent on gas
density. Because heliox has a density roughly a third that of air or oxygen, it can lead to
a dramatic decrease in airway resistance and improvement in ventilation

Heliox is most often used to decrease work of breathing in patients with upper airway
obstructions, such as from tumors or angioedema.[8] The use of heliox in small airway
obstruction, as in asthma, chronic obstructive pulmonary disease (COPD), and

158
bronchiolitis, is controversial and is not typically used in the management of these
patients

185) What is the most likely effect of adding 25mcg of fentanyl to 1mL of 0.5%
bupivicaine injected intrathecally:

a) Increased respiratory depression at 6-8h


b) Increased block success
c) Increased nausea and vomiting
d) Increased hypotension
Answer B
See Q127 , 2005
respiratory depression more than 2 h after intrathecal injection of fentanyl or sufentanil
has never been described.
The addition of intrathecal fentanyl (10–25mcg) to surgical spinal anesthetics
hastens the onset of surgical anesthesia, enhances intraoperative analgesia, and
provides several hours of postoperative analgesia without prolonging motor block
or delaying discharge.

186) Which volatile is least likely to cause immune mediated hepatic injury:
a) Halothane
b) Sevoflurane
c) Desflurane
d) Isoflurane

Answer B 2005-21
Barash 414 5th Ed
Binding of an oxidatively derived metabolite of halothane to liver cytochromes that could
then act as a hapten and induce an immune reaction. This hypersensitivity reaction has
been associated with severe liver damage and fulminant hepatic failure.

159
This metabolic pathway involving the cytochrome P4502E1 system during halothane
exposure is identical to the metabolic pathway noted with enflurane, isoflurane, and
desflurane . However, the expression of the neoantigens should be related to the
amount of metabolism of each agent. This would suggest that, in terms of antigenic
load, halothane > enflurane > isoflurane > desflurane. Indeed, case reports have
appeared in the literature linking each of these anesthetics with immune-mediated
hepatitis. The incidence of fulminant hepatic failure after halothane is 1 in 35,000.
The incidence is lower in Paeds ( I in ? 80000)
There are no reports of fulminant hepatic necrosis associated with sevoflurane in
humans. Sevoflurane is not metabolized to a trifluoroacetyl halide; rather, it is
metabolized to hexafluoroisopropanol that does not serve as a neoantigen

There are many causes of postoperative jaundice and abnormal liver function tests:
 viral hepatitis
 coexisting liver disease (such as Gilbert's disease)
 blood transfusions
 septicemia
 drug reactions
 intra- and postoperative hypoxia and hypotension
 direct tissue trauma as a result of the surgical procedure.
The diagnosis of halothane hepatitis is generally made based on ―incomplete exclusion,‖
defined as the appearance of liver damage within 28 days of halothane exposure in a
person in whom other known causes of liver disease have been excluded.
Halothane hepatitis appears more common in females (2:1), obese patients & adults.

187) Closing capacity increases with all EXCEPT?


a) Smoking
b) Supine
c) Obesity
d) Increased cardiac output
D
In Miller 5th edition : Chapter 15 Respiratory physiology;
Increase in closing capacity with smoking, obesity, aging, and supine position.

188) A 6-year-old boy with cerebral palsy is scheduled for an emergency


appendectomy. All of the following are possible considerations for this case,
EXCEPT?
a) Risk of hyperkalemia with Sux
b) Gastroesophageal Reflux
c) Hypersensitivity to the CNS depressant effects of volatile anesthetics
d) Depressed laryngeal reflexes
A
Review article Cerebral palsy and anaesthesia Paediatric Anaesthesia 2002 12: 296–
303
They include a range of communication difficulties; the presence of scoliosis perhaps
restricting ventilation; GOR (some have had a corrective procedure); contractures
restricting access for examination and positioning; decubitus ulcers and skin infections;
presence of gastrostomy or tracheostomy and problems associated with malnutrition.

160
Medical comorbidities, in particular epilepsy and chest infection, need to be evaluated. A
clear history of current medication should be elicited, particularly of antacids and
anticonvulsants. Cisapride, commonly used for medical treatment of GOR in these
patients has been reported to be associated with prolonged QT interval and life
threatening ventricular dysrhythmias. However, if liver function is normal, in the absence
of drug interactions impairing metabolism of cisapride and using modest, accurately
measured doses, ventricular dysrhythmia is unlikely to be a concern (19,20). There is
evidence that sodium valproate may increase bleeding due to platelet dysfunction,
thrombocytopaenia or aquired von Willebrand factor deficiency type 1.
Preoperative assessment should also include screening questions for latex allergy.
Children with CP, because of multiple surgical procedures and exposure to latex
allergens from an early age, are at increased risk of developing latex allergy
Studies of muscle relaxants in children with CP indicate a slight sensitivity to
succinylcholine. In a dose response study by Theroux et al. a slightly lower ED50 was
found compared to historical controls. Although this difference was detected, it is unlikely
to be clinically significant. This finding may be due to different structure and composition
of the neuromuscular junction (25) or an upregulation of acetylcholine receptors
(AchR‘s), perhaps secondary to chronic anticonvulsant medication (26). Vecuronium
resistance has also been demonstrated in CP.

Summary of Considerations

1. GERD & Recurrent aspiration


2. Mental retardation
3. Seizure
4. Scoliosis - restrictive defect
5. Cor pulmonale
6. Contractures
a. Access
b. Positioning

189) All of the following are true regarding ondansetron, EXCEPT?


a) Delay of discharge from the PACU
b) Headache is a common side effect
c) No extrapyramidal side effects
d) Increase in LFTs
A Q2005-35
P&P
Ondansetron is a carbazalone derivative that is structurally related to serotonin and
possesses specific 5-HT3 subtype receptor antagonist properties, without altering
dopamine, histamine, adrenergic, or cholinergic receptor activity
The most commonly reported side effects from treatment with ondansetron are
headache, diarrhea, and transient increases in the plasma concentrations of liver
transaminase enzymes. Cardiac dysrhythmias (Increased QTc and AV block) have
been reported after the IV administration of ondansetron and metoclopramide

161
Sedation, hypotension, dysphoria, and extrapyramidal symptoms that may accompany
administration of alternative antiemetic drugs (promethazine, droperidol, and
metoclopramide) do not accompany administration of ondansetron.

Barash
useful for PONV related to stimulation of gastric enterochromaffin cells by blood.
Ondansetron has few side effects. Headache occurs after iv administration, and
transient liver enzyme elevation is seen in a small percentage of patients.
Ondansetron is cost effective if its use shortens PACU length of stay, avoids
admission, and improves patient satisfaction. Granisetron, tropisetron, dolasetron, and
the newer ramosetron all are effective antiemetics, but none appears to surpass
ondansetron for efficacy.

190) All of the following are causes of Absolute R to L shunt, EXCEPT?


a) VSD
b) Advanced liver failure
c) Pulmonary thromboembolism
d) Drainage from bronchiolar veins
C
I could not find a reference. I think that PE will mainly cause an increase in the dead
space since no blood is shunting fro the right side of the heart.

191) All of the following are signs of brainstem anesthesia following


retrobulbar block, EXCEPT?
a) Increased intra-vitreal pressure
b) Contra-lateral Amaurosis Fugax
c) Shivering
d) Loss of consciousness
2004-200
A

192) Factors facilitating bupivicaine transfer across the placenta EXCEPT


a) Increased fetal pH
b) Decreased molecular weight
c) Decreased protein binding
d) Increased amount unionized drug
?? C or A ( I think Hani is correct it is A)
Chestnut p57
Factors affecting drug transfer across the human placenta include lipid solubility,
protein binding, tissue binding, pKa of the drug, PH of fetal blood and blood flow.

Hani
Correct stem in Q-80 UWO version (didn‘t say Bup which makes more sense) ;
All increase placental transfer of drugs except?
e. Low protein binding
f. Low molecular weight
g. High dergree of non-ionized fraction
h. Increased fetal pH

162
Can See Q-94 2003 & Q-178 1999
Accumulation of LA in Fetus by :
 ed Lipophilicity or un-ionized fraction
 ed Protein Binding
 ed Fetal pH
 Prilocaine>Mepivacaine>Lidocaine>Bupivacaine>Etidocaine

BUPIVICAINE WILL BE SLOWER FOR TWO REASONS: 1 ITS HIGH PROTEIN


BINDING (LESS FREE DRUG FOR MEMBRANE TRANSFER) 2 AT PHYSIOLOGIC PH
IT WILL HAVE A HIGHER CONCENTRATION OF IONIZED MOLECULES WHICH
DON'T CROSS THE PLACENTA, ONLY NONIONIZED MOLECULES CONTRIBUTE
TO THE EQUILIBRIUM ACROSS THE PLACENTA

193) Regarding hyperventilation in acute head injury, all of the following


statements EXCEPT ONE are true. Indicate the exception.
a) May not effectively decrease intracranial pressure in the face of severe
head trauma
b) Will attenuate the vasodilatation of inhaled anesthetics
c) The effect on ICP will be terminated by CNS buffering
d) The effects will last 24-48 hours
D
Barash

Hyperventilation has been the mainstay of acute and subacute management of


intracranial hypertension. As discussed previously, hyperventilation reduces brain
volume by decreasing CBF through cerebral vasoconstriction. For every 1 mm Hg
change in PaCO2, CBF changes by 1–2 ml·100 g–1·min–1. The duration of effectiveness
of hyperventilation for lowering ICP may be as short as 4–6 hours, depending on the
pH of the CSF. Hyperventilation is only effective when the CO2 reactivity of the
cerebrovasculature is intact. Impaired responsiveness to changes in CO2 tension occurs
in areas of vasoparalysis, which are associated with extensive intracranial disease such
as ischemia, trauma, tumor, and infection.

The typical target PaCO2 is 30–35 mm Hg. A PaCO2 below 20 mm Hg (or 25–30 mm Hg
in some pathologic conditions) may be associated with ischemia due to extreme cerebral
vasoconstriction. By monitoring global cerebral oxygenation with, say, jugular venous
oxygen saturation (SjO2), the therapeutic effectiveness of hyperventilation can be
determined and more safely applied.249

194) Regarding the 5 lead EKG. All of the following leads EXCEPT ONE will be
affected by disconnection of the Left arm lead. Indicate the exception.
a) AVR
b) III
c) AVF
d) II
D

163
195) All of the following EXCEPT ONE predisposes patients to increased risk of
LATE respiratory depression after epidural opiods.
a) Repeat doses of opiods
b) Elderly patients
c) Hydrophobic opiods
d) Comorbid lung disease
C
A2003 Q100 Regional
With respect to epidural opioids, all of the following statements EXCEPT ONE
are associated with an increased risk of respiratory depression.
a) repeated epidural opioid dosing
b) highly lipid-soluble narcotics
c) elderly patient
d) co-existing respiratory disease
e) concomitant parenteral opiod administration

ANSWER: B
Factors Predisposing to the Development of Respiratory Depression Following Epidural
Opioids:
1. Hydrophilic drug
2. Large doses
3. Repeated doses
4. Concomitant administration of parenteral opioids or other CNS depressants
5. Elderly or debilitated (Age may influence spinal fluid volume and pressure, and
the brains of elderly
6. Coexisting respiratory disease
7. Thoracic epidural
8. High sensitivity to opioids (i.e., no previous exposure to opioids
9. Intrathecal administration
10. Raised intrathoracic pressure (e.g., controlled ventilation, coughing, vomiting)
11. Lack of opioid tolerance

196) Which of the following is a component of modern vaporizer design:


a) Constant derived flow
b) High pumping effect
c) Low heat of latency
d) High thermal conductivity

B, C and D are clear. A is wrong for the variable bypass vaporizers which have variable
outputs at different (very low or very high) flow rates.

Barash

Latent Heat of Vaporization


The latent heat of vaporization is defined as the number of calories required to change 1
g of liquid into vapor without a temperature change. The energy for vaporization must

164
come from the liquid itself or from an outside source. The temperature of the liquid
decreases during vaporization in the absence of an outside energy source. Energy loss
can lead to significant decreases in temperature of the remaining liquid. This
temperature drop will greatly decrease vaporization.41,43

Thermal Conductivity
Thermal conductivity is a measure of the speed with which heat flows through a
substance. The higher the thermal conductivity, the better the substance conducts
heat.41 Vaporizers are constructed of metals that have relatively high thermal
conductivity, which helps maintain a uniform temperature.

Factors That Influence Vaporizer Output

Flow Rate. With a fixed dial setting, vaporizer output varies with the rate of gas flowing
through the vaporizer. This variation is particularly notable at extremes of flow rates. The
output of all variable bypass vaporizers is less than the dial setting at low flow rates (less
than 250 ml·min–1). At extremely high flow rates, such as 15 l·min–1, the output of most
variable bypass vaporizers is less than the dial setting.

Temperature. Because of improvements in design, the output of contemporary


temperature-compensated vaporizers is almost linear over a wide range of
temperatures. A bimetallic strip (Fig. 22-15) or an expansion element (Fig. 22-16) directs
a greater proportion of gas flow through the bypass chamber as temperature increases.

Intermittent Back Pressure. Intermittent back pressure associated with positive


pressure ventilation or with oxygen flushing can cause higher vaporizer output
concentration than the dialed setting. This phenomenon, known as the pumping
effect,41,46,49–51 is more pronounced at low flow rates, low dial settings, and low levels of
liquid anesthetic in the vaporizing chamber. Additionally, the pumping effect is increased
by rapid respiratory rates, high peak inspired pressures, and rapid drops in pressure
during expiration.

Carrier Gas Composition. Vaporizer output is influenced by the composition of the


carrier gas that flows through the vaporizer.47,48,53–60 When the carrier gas is quickly
switched from 100% oxygen to 100% nitrous oxide, there is a rapid transient decrease in
vaporizer output followed by a slow increase to a new steady-state value

197) Which of the following anesthetics are contra-indicated in someone who is


known anaphylactic to pancuronium:
a) Meperidine
b) Fentanyl
c) Morphine
d) Codeine
Answer C
I would go on morphine based on the following article, Refs\Anaphylaxis and anesthesia
,Dx approach.pdf

165
Therefore, historically, the recommendation has been to switch to a different subclass
with a distinct chemical structure (when a pt has an allergy to one particular opioid) in
order to avoid antibody recognition (127). However, the observation that codeine,
meperidine and methadone, representatives of the different opioid subclasses react
with morphine antibodies put into question this practice (87, 123). In the absence of
validated drug specific diagnostic tools, correct diagnosis of opiate allergy remains a
clinical challenge. Particularly, because SPT (skin prick test) have been demonstrated to
be not useful for this purpose (118, 128) and morphine sIgE, as addressed above,
does not per se indicate sensitization from this drug, but might rather mirror
sensitization from quaternary ammonium structures from NMBA. Placebocontrolled
Challenges may be required to diagnose opioid allergy.

All NMBA can elicit anaphylaxis and there is an agreement that the short-acting
depolarizing succinylcholine poses the greatest risk, despite its close structural
homology to acetylcholine (20, 31, 32). Neuromuscular blocking agents can induce two
types of reactions. One is driven by an immunological mechanism and is IgE-dependent
with the quaternary ammonium (NH4 +) structures as main antigenic epitope (33, 34),
while the second one, particularly described with benzylisoquinolinium-type NMBA such
as mivacurium, atracurium and d-tubocurarine, results from nonspecific stimulation of
mast cells (35–37). Cisatracurium, anintermediate acting nondepolarizing stereoisomer
of the benzylisoquinolinium agent atracurium and succinylcholine appear to demonstrate
the lowest potency of nonspecific mast cell and basophil activation.

Cross-reactivity between NMBA is said to be common because of ubiquitous ammonium


groups in these drugs. The estimated prevalence of cross-reactivity between
NMBA is about 65% by skin tests and 80% by radioimmuno assay (RIA) inhibition tests.
Cross-reactivity depends on the configuration of the paratope of the antibody, which
might either completely correspond to the ammonium epitope or extend to an adjacent
part of the NMBA molecule, to the structure of the NMBA (flexibility, inter-ammonium
distance) and to the relative affinities of the different NMBA for their sIgE antibodies

Opioids
Generalized reactions to opioids [e.g. morphine, codeine and synthetic opioids such as
meperidine (international nonproprietary name, INN: pethidine)] usually result from
nonspecific mast cell activation, rather than from IgE-mediated degranulation (117, 118).
Particularly, skin mast cells are sensitive to nonspecific activation by opioids.

Tryptase
An elevated total serum tryptase level (pro-b and pro-a and mature-btryptase) is
therefore highly indicative for mast cell degranulation, as seen in systemic anaphylaxis
[for review: (21)]. A rise in total tryptase can be quantified in serum (or plasma) as soon
as 30 min after onset of symptoms, but sampling is recommended 60–120 min after
onset-ofsymptoms (22). Tryptase‘s half-life is about 120 min, and levels gradually
decrease over time. To enable comparison with baseline levels, a new sample should be
collected >2 days after the reaction. _False-negative_ results have been attributed to a
mechanism where the reaction involves basophils rather than mast cells (22), whereas
false positive results have been reported in cases of extreme stress such as hypoxemia
and major trauma (27, 28). Note that serum tryptase does not differentiate between
immunological and nonimmunological mast cell activation, and does not contribute in the
identification of the causative compound(s). However, in nonimmunological reactions

166
rise of tryptase is less prevalent and usually less pronounced than in immunological
mast cell activation

Time elapsed between administration and onset-of-symptoms


Clinical signs and symptoms of anaesthetic anaphylaxis usually start within 5–10 min
after intravenous administration of the responsible agent but can occur within seconds.
In contrast, anaphylaxis from natural rubber latex (NRL) and antiseptics exhibit a more
delayed onset and generally occurs during maintenance anaesthesia or recovery, as a
result of later application or absorption through skin, mucosal surfaces and/or soft
tissues, or removal of a tourniquet (2, 17, 18). In patients allergic to latex, bronchospasm
may also be observed early following arrival of a sensitized patient in the operating
theatre. Anaphylaxis from colloids can occur immediately or demonstrate a more
delayed onset.

198) In regards to mild hypothermia, all of the following are true EXCEPT:
a) Increased wound infections
b) Longer post-op hospital stay
c) Impaired coagulation
d) Impaired NMB reversal
C

199) Of the following all are risk factors for post-op nausea and vomiting
EXCEPT:
a) Suffers motion sickness
b) Female
c) Cigarette smoking
d) Dehydration
C

200) Patient with a fixed airway obstruction breathing 70% Helium 30%
oxygen will decrease the resistance to airflow through the stenotic
region within the trachea because:
a) Helium decreases the viscosity of the gas mixture
b) Helium decreases the friction coefficient of the gas mixture
c) Helium decreases the density of the gas mixture
d) Helium decreases the Reynolds number of the gas mixture

C
Reference: Helium-oxygen mixtures in airway obstruction due to thyroid carcinoma.
Mark Rudow, et al. C A N A N A E S T H S O C J , 1 9 8 6 7 3 3 : 4 / p p 4 9 8 - 5 0
1 (CJA)

Under normal circumstances gas flow in the large airways is laminar and:

The laws relating the transition from laminar to turbulent flow were elucidated by
Reynolds, and Reynolds critical number is the factor relating critical velocity to pipe

167
diameter, density and viscosity. Flow through an orifice is always partly turbulent and
under these conditions flow rate will vary inversely with the square root of the density.
This contrasts with conditions of laminar flow when rate will vary inversely with viscosity.
The viscosities of helium and oxygen are very similar with the result that flow rates will
be similar under conditions of laminar flow, but their densities are very different, with the
result that helium will flow almost three times faster across an orifice than will oxygen
(Table III). The marked improvement in this patient on inspiring mixtures less dense than
air confirmed that laminar flow had been replaced by turbulent flow.

201) 50-year-old male alcoholic presents for inguinal hernia repair, what
is the most likely postoperative electrolyte abnormality?

a) HypoMg
b) HypoK
c) HypoNa
d) HyperK
A

202) Differences between LMWH and standard unfractionated heparin include all
EXCEPT:

a) No need for PTT to monitor anticoagulation


b) Less interaction with antiplatelet medications
c) Less formation of antiplatelet antibodies
d) Equal development of antibodies
B

203) Increased closing capacity with all EXCEPT:

a) Smoking
b) Decreased cardiac output
c) Obesity
d) Supine position
B
(Repeat 2001 Q-119)

Miller

168
The CV plus the residual volume is known as the CC and is expressed as a percentage
of total lung capacity. Smoking, obesity, aging, and the supine position increase the
CC. In healthy individuals at a mean age of 44 years, CC = FRC in the supine position,
and at a mean age of 66 years, CC = FRC in the upright position.

204) Which of the following would be expected in a patient with


tracheomalacia resulting after excision of a large thyroid goiter?

a) Decreased expiratory and inspiratory flow pattern on flow volume


curve
b) Decreased inspiratory and normal expiratory flow pattern on flow
volume curve
c) Decreased expiratory and normal inspiratory flow pattern on flow
volume curve
d) Expiratory/inspiratory ratio < 0.5
B

205) What is true about a neonate who presents to the OR for repair of a
myelomeningocele?

a) The risk of hydrocephalus is present from the time of birth


b) There is a risk of hyperkalemia with succinylcholine
c) There is the risk of brainstem compression (Arnold Chiari
malformation)
d) There should be concern about coexisting cardiac anomalies
C
Repeat 2004 Q115

206) Breakdown or metabolism of volatiles in the human body from highest to


lowest:

a) Halothane > Desflurane > sevoflurane > Isoflurane


b) Sevoflurane > halothane > Isoflurane > Desflurane
c) Halothane > sevoflurane > Desflurane > Isoflurane
(Hal>Sevo>Iso>Des@UWO version)
d) Sevoflurane > Isoflurane > halothane > Desflurane
C
Anesth Analg 2005;100:1020–33; Illustrations of Inhaled Anesthetic Uptake, Including
Intertissue Diffusion to and from Fat (http://www.anesthesia-
analgesia.org/cgi/reprint/100/4/1020.pdf)

Approximately 0.02% of what is taken up is lost from the liver by metabolism of


desflurane (metabolism is the third factor that governs the uptake and distributionof
potent inhaled anesthetics). A larger amount of isoflurane (0.2%) and sevoflurane (5%)
is lost in this manner.

Miller

Halothane ; Approximately 25% of administered halothane (CF3CHBrCl) is metabolized


to trifluoroacetic acid, chloride (Cl-), and bromide (Br-) .

169
Sevoflurane; undergoes approximately 5% metabolism.
The rate of sevoflurane defluorination in vitro is approximately the same as that of
methoxyflurane.However, serum F- concentrations after sevoflurane administration are
significantly less than those after methoxyflurane,presumably the result of large
differences in blood-gas partition coefficients of the two agents (0.69 for sevoflurane
versus 10.2 for methoxyflurane).
Enflurane ; Approximately 2.5% of absorbed enflurane is metabolized.
Isoflurane ; an isomer of enflurane, is metabolized even more slowly (≈0.2%) than
halothane or enflurane.

Desflurane ; is expected to be metabolized in a manner similar to isoflurane, because


these two molecules differ by only one atom at the α-carbon position; desflurane has a
fluorine atom, and isoflurane has a chlorine atom. However, the fluorine atom
substitution decreases the metabolism at the α-carbon position significantly so that the
amount of F- and nonvolatile organic fluorine compounds formed from the metabolism
of desflurane is considerably less compared with isoflurane.

207) Which statement BEST characterizes the addition of sodium bicarbonate to


Lidocaine 1.5%?

a) No precipitation will occur because the local anesthetic solution is


alkaline
b) The amount of ionized local anesthetic will increase
c) The pCO2 of the solution will increase
d) Addition of 1 mmol NaHCO3 to 10 mL of Lidocaine 1.5% will increase
the pH to > 10
C
Similar to 2002 Q-88.Addition of bicarbonate to local anesthetic results in:
a) May precipitate
b) Reduction in speed of onset
c) Results in increased ionized fraction
d) Decreases PCO2

CORRECT ANSWER: A

Ref: Miller &Barash

The presence of bicarbonate CO2 in a solution of local anesthetic applied to an isolated


nerve results in a more rapid onset and a decrease in the minimum concentration
required for conduction blockade. Although the effect of CO2 on local anesthetic activity
is easily demonstrable in isolated nerve, controversy exists concerning the clinical utility
of carbonated local anesthetic solutions. For example, some studies have failed to
demonstrate a significantly more rapid onset of action for lidocaine carbonate as
compared with lidocaine hydrochloride for epidural blockade, whereas others have
reported a significant reduction in onset time of epidural blockade with lidocaine
carbonate. Similar discrepancies existed when bupivacaine hydrochloride and
bupivacaine carbonate were evaluated clinically. Although the general effect of CO2 on
the latency of conduction blockade may be controversial, carbonated solutions appear to
improve the depth of sensory and motor blockade when they are administered into the

170
epidural space. In addition, these solutions may produce a more complete blockade of
the radial, median, and ulnar nerves when they are employed for brachial plexus
blockade. Addition of sodium bicarbonate to local anesthetic solutions has also been
reported to decrease the onset time of conduction blockade. An increase in the pH of
the local anesthetic solution increases the amount of drug in the uncharged base form
and thus should enhance the rate of diffusion across the nerve sheath and nerve
membrane, resulting in a more rapid onset of anesthesia. Alkalinization of solutions of
bupivacaine or lidocaine reportedly does significantly decrease the latency of onset of
brachial plexus and epidural blockade. On the other hand, at least one study failed to
demonstrate an improved onset of brachial plexus blockade when the pH of bupivacaine
solution was increased by addition of sodium bicarbonate. Clinically used local
anesthetics cannot be alkalinized beyond a pH of 6.05-8 before precipitation occurs, and
such pHs will only increase the neutral form from 3% to about 10%. Although a tripling
of the available local anesthetic (from 3% to 10%) will improve efficacy, recent studies
have proposed several other mechanisms for the effects of alkalinization (eg. by
activating vasoconstrictive effects of epinephrine when present in the local anesthetic
solution.

Studies on isolated, desheathed nerves using a variety of impulse blocking agents at


constant extracellular pH have shown that un-ionized anesthetics (e.g., benzocaine) and
ionizable tertiary amines are potentiated by bicarbonate carbon dioxide buffers. In
contrast, the potency of permanently cationic (quaternary) local anesthetics is unaffected
by bicarbonate carbon dioxide. Therefore, the mechanism of ion trapping, whereby the
ionized form of the drug is concentrated in the axoplasm by the internal acidification
wrought by membrane-permeant carbon dioxide molecules, does not fully account for
potentiation by bicarbonate carbon dioxide. Furthermore, this potentiation in isolated,
desheathed nerve is strongly dependent on bicarbonate concentration but is almost
independent of CO2 tension (PCO2). At present, there is no easy explanation for the
potentiation of local anesthetics by bicarbonate buffers, nor has a clear connection been
established between in vitro results and clinical phenomena.

208) Which of the following physiologic responses is most frequently observed


with infra-renal cross-clamping of the abdominal aorta in a pt w/ a normal LV
and coronary arteries:

a) Decreased or no change in blood pressure


b) Decreased CO
c) Increased CVP
d) Increased PCWP

C
Miller,Ch-52

171
Figure 52-13 Systemic hemodynamic response to aortic unclamping. AoX, aortic
cross-clamping; Cven, venous capacitance; R art, arterial resistance; Rpv, pulmonary
vascular resistance.

Barash

Table 33-8.EFFECT OF LEVEL OF AORTIC OCCLUSION ON CHANGESIN


CARDIOVASCULAR VARIABLES
% Change in Variable, by Level ofAortic Occlusion

Suprarenal–
Cardiovascular Variable Supraceliac infraceliac Infrarenal

Mean arterial blood pressure 54 5* 2*


Pulmonary capillary wedge 38 10* 0*
pressure
End-diastolic area 28 2* 9*
End-systolic area 69 10* 11*
Ejection fraction –38 –10* –3*
Abnormal motion of wall, % of 92 33* 0*
patients

172
New myocardial infarctions, % of 8 0* 0*
patients
*Statistically different (p <0.05) from group undergoing supraceliac aortic
occlusion.

209) Which of the following is true regarding phenol?

a) Hypobaric in glycine
b) Agent of choice for celiac plexus blocks
c) Higher affinity for vascular tissue than neural tissue
d) Causes irritation of meninges when injected intrathecally
C

210) Which amide local anesthetic is metabolized by liver and tissue


cholinesterases:

a) Lidocaine
b) Bupivicaine
c) Ropivicaine
d) Tetracaine
Hani ;Wrong stem(tetracaine is Ester), correct one from UWO version;

D
UWO Version
Which medication is metabolized by plasma and liver cholinesterase:
i. Bupivicaine
j. Lidocaine
k. Ropivicaine
l. Tetracaine
Answer Tetracaine

211) What herbal supplement interacts with platelets:

a) St. John's Wort


b) Kava
c) Echinacea
d) Gingko Biloba
D

212) What is the MOST sensitive sign for neuromuscular reversal:

a) Sustained head lift for 5 seconds


b) No fade with TOF
c) No fade with 5 seconds sustained tetanus at 50 Hz
d) No fade with double burst stimulation
D

173
Br J Anaesth. 1989 Mar;62(3):274-8;
‗Double burst stimulation (DBS): a new pattern of nerve stimulation to identify residual
neuromuscular block‘
Golden paragraph: The DBS with three impulses in each burst (DBS3,3) was considered
to be the most sensitive and the least painful and thus most suitable for clinical use.

CJA, Vol 42, 21-27;1995


Comparison of tactile and mechanomyographical assessment of response to double
burst and train-of-four stimulation during moderate and profound neuromuscular
blockade
‗It is concluded that tactile evaluation of responses ot DBS stimulation can estimate
deeper levels of blockade than tactile evaluation of responses to TOF.

Double Burst Stimulation: (Dorsch and Dorsch p. 854)


DBS consists of two short trains of 50 Hz tetanic stimuli (usually 3 0.2 second
impulses) separated by 750 milliseconds. Numerous studies show that fade is
more readily detected with DBS than TOF using visual or tactile monitoring.
Another possible use of DBS is during assessment of deep block, since the first twitch in
double burst can be detected at slightly deeper levels of block than the first twitch of
TOF.

Miller 6th ed Ch-39

Double-Burst Stimulation
DBS was developed with the specific aim of allowing manual (tactile) detection of small
amounts of residual neuromuscular blockade under clinical conditions. During recovery
from neuromuscular blockade, the degree of residual block can be evaluated from
recorded responses to TOF nerve stimulation. Without recording equipment,
however, it is not possible by visual or tactile means to evaluate the TOF response
with certainty to exclude shallow degrees of residual neuromuscular blockade.
With DBS (as opposed to TOF stimulation), it is easier to "feel" fade in the
response.
DBS consists of two short bursts of 50-Hz tetanic stimulation separated by 750
ms. The duration of each square wave impulse in the burst is 0.2 ms. Although the
number of impulses in each burst can vary, initial studies indicate that DBS with three
impulses in each of the two tetanic bursts (DBS3,3 ) is suitable for clinical use. Studies
are currently evaluating the suitability of other types of DBS.

In nonparalyzed muscle, the response to DBS3,3 is two short muscle contractions of


equal strength. In the partly paralyzed muscle, the second response is weaker than the
first (i.e., the response fades). Measured mechanically, the TOF ratio correlates
closely with the DBS3,3 ratio. During recovery and immediately after surgery, tactile
evaluation of the response to DBS3,3 is superior to tactile evaluation of the response to
TOF stimulation. However, as appears from, absence of fade in the manually evaluated
response to DBS3,3 does not exclude residual neuromuscular blockade.

During recovery of neuromuscular function, when all four responses to TOF


stimulation can be felt, an estimation of the TOF ratio may be attempted. However,
manual (tactile) evaluation of the response to TOF stimulation (see Fig. 39-8 ) is not
sensitive enough to exclude the possibility of residual neuromuscular
blockade.[29][104][110] Greater sensitivity is achieved with DBS3,3, but even absence of

174
manual fade in the DBS3,3 response does not exclude clinically significant residual
blockade.[33] Therefore, manual evaluation of responses to nerve stimulation should
always be considered in relation to reliable clinical signs and symptoms of residual
neuromuscular blockade.

213) Which of the following is a characteristic pattern seen in a fellow


anesthetist addicted to drugs:

a) Always requests a medical student to help with the case


b) Meticulous charting
c) Refuses to take over cases from colleagues
d) Insists on giving pts narcotics in PACU himself
D

214) Which of the following drugs is LEAST useful in treating post-herpetic


neuralgia:

a) Ibuprofen
b) Morphine
c) Amitryptyline
d) Epidural methylprednisolone
Answer A

215) What is the GCS for the following: opens eyes to command, confused
speaking, moves limbs to command:

a) 13
b) 12
c) 10
d) 14
Answer A
Table 28-18. MODIFIED GLASGOW COMA
SCALE
EYE OPENING
Spontaneously 4
To verbal command 3
To pain 2
None 1
BEST VERBAL RESPONSE
Oriented, conversing 5
Disoriented, conversing 4
Inappropriate words 3

175
Incomprehensible sounds 2
No verbal response 1
BEST MOTOR RESPONSE
Obeys verbal commands 6
Localizes to pain 5
Flexion/withdrawal 4
Abnormal flexion (decorticate) 3
Extension (decerebrate) 2
No response (flaccid) 1
Mild head injury = 13–15; moderate = 9–12;
severe = 8.

216) What forms the posterior boundary of the epidural space?

a) Posterior longitudinal ligament


b) Ligamentum flavum
c) Pedicles
d) Supraspinous ligament
Answer B

217) A 26 y o male is involved in a MVA and has a C6 fracture. Which time frame
is the MOST likely for developing a NEW neurological deficit:

a) Intubation
b) Positioning
c) Surgical correction
d) Emergence
Answer B
I even Checked it with Dr Duggal

218) What patient does this blood gas represent? 7.35/ PCO2 34/ PO2 75 98%

a) Neonate
b) Geriatric
c) Pregnant
d) High Altitude (Chronic)
Answer A
Miller
Acute exposure to altitudes in the range of 4000 to 5000 m causes a decrease in arterial
Pao2 to approximately 40 mm Hg and a decrease in arterial Hb O2 saturation to
approximately 75%
Geriatric

176
The combination of decreased elastic recoil and increased chest wall stiffness causes an
elevation in intrapleural pressure by 2 to 4 cm H2O. However, total lung capacity is
relatively unchanged. Residual volume increases by 5% to 10% per decade. Therefore,
vital capacity decreases. Closing capacity, the volume at which small dependent airways
start to close, increases with age. Change in the relationship between functional residual
capacity and closing capacity causes an increased ventilation-perfusion mismatch and
represents the most important mechanism for the increase in alveolar-arterial gradient
for oxygen observed in aging. Ventilatory responses to hypoxia, hypercapnia, and
mechanical stress are impaired secondary to reduced central nervous system activity.[15]
In addition, the respiratory depressant effects of benzodiazepines, opioids, and volatile
anesthetics are exaggerated.
Table 62-3 -- Normal values for arterial Po2
Age (yr) Mean and Range (mm Hg)
20–29 94 (84–104)
30–39 91 (81–101)
40–49 88 (78–98)
50–59 84 (74–94)
60–69 81 (71–91)
Increases in pulmonary vascular resistance and pulmonary artery pressure occur with
age and may be secondary to decreases in cross-sectional area of the pulmonary
capillary bed.[18] Hypoxic pulmonary vasoconstriction is blunted in the elderly and may
cause difficulty with one-lung ventilation.. At 44 years of age, closing capacity equals
functional residual capacity in the supine position and, at 66 years of age, equals
functional residual capacity in the upright position

Barash

Chestnut

177
219) For the acute management of intraoperative hypotension in a patient with
HOCM, which is the best initial choice?

a) Phenylephrine
b) Ephedrine
c) Metoprolol
d) Dopamine
Answer A

220) Which of the following will decrease the incidence of postoperative


neuropraxia associated with the use of an inflated tourniquet?

a) Deflated the tourniquet every 150 minutes.


b) Maintain a gradient between systolic BP and inflation pressure of no
more than 150 mmHg.
c) Avoid regional anesthesia to the limb.
d) Treat tourniquet induced hypertension with vasodilators
Answer B
Miller
Neurologic Consequences

Neurologic problems may occur when tourniquets are inflated for long periods (>2 hours)
or when excessive inflation pressures are used. A shear force is applied to nerve trunks
at the edges of the tourniquet.[229] Within 30 minutes of inflating a tourniquet, nerve
conduction ceases.[230][231] This may reflect axonal hypoxia or the direct result of extrinsic
pressure on the nerves beneath the tourniquet.[232] In clinical practice, tourniquets
should be deflated every 90 to 120 minutes to minimize the risk of postoperative
neurapraxias.[206][226] Alternatively, tourniquet pressure can be lowered to 250 mm
Hg while maintaining systolic pressures at 90 to 100 mm Hg. This provides a
pressure gradient of 150 mm Hg between the tourniquet and systolic pressure,
more than enough to maintain an exsanguinated limb. Anesthesiologists who use
regional anesthesia may be implicated when postoperative neurapraxias are caused by
tourniquet injury. Recognition of the adverse effects of tourniquets has led many to
perform surgery without a tourniquet[233][234][235] or to limit the duration of its use

Table 61-7 -- Physiologic changes caused by limb tourniquets

178
Neurologic Effects
Abolition of somatosensory evoked potentials and nerve conduction occurs within 30
minutes
Application for more than 60 minutes causes tourniquet pain and hypertension
Application for more than 2 hours may result in postoperative neurapraxia
Evidence of nerve injury may occur at a skin level underlying the edge of the tourniquet
Systemic Effects of Tourniquet Release
Transient fall in core temperature[214]
Transient metabolic acidosis
Transient fall in central venous oxygen tension (but systemic hypoxemia unusual)
Release of acid metabolites into central circulation (e.g., thromboxane)
Transient fall in pulmonary and systemic arterial pressures
Transient increase in end-tidal carbon dioxide
Increased oxygen consumption[286]

221) Compared to singleton pregnancy, twin gestations have which of the


following?

a) Increased incidence of pregnancy induced hypertension.


b) Decreased incidence of prolonged labour.
c) Increased maternal hematocrit.
d) Decreased incidence of post partum hemorrhage.
Answer A
Fetal complications associated with multiple gestation:
 Preterm delivery
 Congenital anomalies
 Polyhydramnios
 Cord entanglement
 Umbilical cord prolapse
 Intrauterine growth restriction
 Twin-twin transfusion
 Malpresentation
Exaggerated routine considerations of pregnancy:
1. Exaggerated physiologic changes of pregnancy (Increased CO & 500 ml more
bld volume, decreased FRC & TLC & increased metabolic rate, more anemic)
2. Exaggerated anatomic changes (more difficult intubation, greater risk of supine
hypotensive syndrome)
Specific maternal complications associated with multiple gestation:
 preterm premature rupture of membranes
 preterm labour
 prolonged labour
 preeclampsia/eclampsia
 placental abruption
 DIC (after intrauterine fetal death of one fetus)

179
 operative delivery (forceps and abdominal)
 uterine atony
 antepartum and/or postpartum hemorrhage

222) A patient is undergoing a left thoracotomy with a left-sided endobronchial


double lumen tube. During one-lung ventilation with the right lung ventilated,
there is an increase in the ventilating pressures and the tidal volume is only
100mL. What maneuver is most likely to improve this?

a) Pull back the endobronchial tube.


b) Apply suction to the non-dependent lung.
c) Deflate the endobronchial cuff.
d) Apply CPAP to the non-dependent lung.
Answer C
Both A and C are possible solutions depending on whether the tube is too far in or out.

223) Which of the following is NOT at risk for post-partum hemorrhage:

a) PIH
b) Macrosomnia
c) Prolonged Labour
d) Augmented Labour (oxytocin)
Answer A
Conditions associated with uterine atony
 Multiple gestation
 Macrosomia
 Polyhydramnios
 High parity
 Prolonged labour
 Chorioamnionitis
 Precipitous labour
 Augmented labour
 Tocolytic agents
 High concentrations of a volatile

224) Compared with moderately hydrophobic local anesthetics, highly


hydrophobic LA’s are:

a) Less potent
b) Longer duration of action
c) Faster onset of action
d) Less protein bound
Answer B
Barash CD 4ed

Lipid solubility is another important determinant of activity. Although increasing lipid


solubility may hasten penetration of neural membranes, increasing solubility may also
result in increased sequestration of local anesthetic in myelin and other lipid-soluble
compartments. Thus, increasing lipid solubility usually slows the rate of onset of

180
action.41 Similarly, duration of action is increased as absorption of local anesthetic
molecules into myelin and surrounding neural compartments creates a depot for
slow release of local anesthetics.41 Finally, increased lipid solubility increases
potency of the local anesthetic.

Degree of protein binding also affects activity of local anesthetics, as only the unbound
form is free for pharmacologic activity. In general, increasing protein binding is
associated with increased duration of action. Although the sodium channel is a
protein structure, it does not appear that degree of local anesthetic protein binding
correlates with binding to the local anesthetic receptor. Studies suggest that dissociation
of local anesthetic molecules from the sodium channel occurs in a matter of seconds
regardless of degree of protein binding of the local anesthetic.42 Thus, prolongation in
duration of action associated with an increased degree of protein binding must involve
other extracellular or membranous proteins.

Decreasing p Ka for a given environmental pH will increase the percentage of lipid-


soluble forms in existence, hastening penetration of neural membranes and onset of
action.

225) A patient with significant mitral regurgitation has a LVEDV of 500cc, a SV of


250cc and has a regurgitant volume of 175cc. What is his EF?

a) 15%
b) 25%
c) 35%
d) 50%
Answer D

226) What is the loading dose of rectal acetaminophen in the pediatric patient?

a) 15 mg/kg
b) 40 mg/kg
c) 60 mg/kg
d) 100mg/kg
Answer B

227) Which of the following is associated with metabolic acidosis with an anion
gap?
a) Renal tubular acidosis
b) Ureterosigmoidostomy
c) Ingestion of carbonic anhydrase inhibitors
d) Salicylate intoxication
D

228) Which of the following will be increased in a patient with renal failure?
a) Esmolol
b) Nifedipine
c) Digoxin
d) Pancuronium
C
Answer Dig, as w/ panc it is the metabolite is prolonged ( mainly anyway)

181
Both Digoxin and pancuronium are affected by renal failure, but I think the use of
digoxin in renal failure is more risky, since using pancuronium will only result in a
prolonged relaxation. Although the question is not asking about the bad affects.

Barash
The older nondepolarizing muscle relaxants (i.e., d-tubocurarine, metocurine,
pancuronium, and gallamine) all have prolonged elimination half-lives in renal
failure (see Table 36-4). A single small dose of these drugs should be well
tolerated, but larger doses or repeated small doses may result in the
accumulation of the parent drug and prolonged clinical effect.

Table 36-4. NONDEPOLARIZING MUSCLE RELAXANTS IN RENAL FAILURE60, 64, 65,


68–71, 78, 80

Renally
Excreted
% Renal Half-life (h) Active
Drug Excretion Normal/ESRD Metabolite Use in ESRD
d-Tubocurarine <60 1.4–2.2 – Avoid
Pancuronium <30 2.3/4–8 + Avoid
Gallamine >85 2.5/6–20 – Avoid
Vecuronium <30 0.9/1.4 + Normal single,
smaller repeat
doses; avoid
prolonged CI
Rocuronium <30 1.2–1.6/1.6–1.7 – Normal single,
repeat doses,
increased
variability
Atracurium/cis- <5 0.3/0.4 – Normal single,
atracurium repeat, CI doses
Mivacurium <7 2 min/2 min – Duration 1.5 ×
normal, lower CI
dose
CI = continuous infusion; ESRD = end-stage renal disease.

P&P

Clearance of digoxin from the plasma is primarily by the kidneys, with approximately
35% of the drug excreted daily. In the presence of renal dysfunction, the elimination half-
time of digoxin is depressed in proportion to the decrease in creatinine clearance. For
example, the elimination half-time of digoxin is 31 to 33 hours in the presence of normal
renal function and up to 4.4 days in the absence of renal function.

182
Nifidipine:
Protein binding approaches 90%. Hepatic metabolism is nearly complete, with
elimination of inactive metabolites principally in urine (about 80%) and, to a lesser
extent, in bile. The elimination half-time is 2 to 5 hours.

229) With regards to norepinephrine biosynthesis, which of the following is the


most immediate precursor?
a) Tyrosine
b) Dopamine
c) Epinephrine
d) Phenylalanine

230) Direct coronary artery vasoconstrictor:


a) PGE1
b) Hypoxia
c) Histamine
d) ACh

Wrong Question , see Q157, for this Q , both H1 and PGE2 cause constriction,
while the opposite is true for H2 and PGE1

231) What variables are required to measure SVR:


a) MAP, CVP, CO
b) Systolic BP, CVP, CO
c) MAP, PCWP, CO

183
d) Systolic BP, PCWP, CO
A

232) Celiac plexus block effective for cancer pain in all of the following EXCEPT:
a) Liver
b) Stomach
c) Kidney
d) Small intestine
C

Cousins
The celiac plexus contributes to the innervation of all the intra-abdominal viscera derived
from embryonic foregut, including much of the gastrointestinal tract (distal esophagus,
stomach, duodenum, small bowel, ascending and proximal transverse colon), pancreas,
adrenal glands, spleen, liver, and biliary system. Thus, celiac block has potential utility
for treating pain emanating from a variety of intra-abdominal structures, although its
most important indication is pain due to pancreatic cancer. Though efficacy is less than
for pancreatic pain,148,152 painful hepatic metastases may respond to celiac block, unless
referred right shoulder pain is prominent. Diffuse abdominal pain due to disseminated
peritoneal implants is unlikely to be relieved after celiac block.

233) What is true regarding traumatic aortic rupture:


a) Most common site is the ascending aorta
b) Blood pressure is monitored via the left radial art line
c) Blood pressure is managed with NTG and Beta-blocker
d) OLV is contraindicated in the presence of nitroprusside infusion
C

234) Epinephrine containing LA given epidurally will cause which of the


following compared to LA without epinephrine:
a) Increased HR
b) Increased BP
c) Increased contractility
d) Increased SVR

Answer A ( What about C)

Barash

The hemodynamic changes produced by epidural anesthesia are largely dependent on


whether or not epinephrine is added to the local anesthetic solution (Fig. 26-7).

a. Hemodynamic changes of high epidural anesthesia without epinephrine in the local


anesthetic solution resemble spinal anesthesia, although the magnitude is usually less
than that seen with comparable levels of spinal block.

b. When epinephrine is added to the local anesthetic solution the resulting B2-mediated
vasodilation leads to a greater decrease in blood pressure than occurs in the
absence of epinephrine.

184
Figure 26-7. The cardiovascular effects of spinal and epidural anesthesia in volunteers
with T5 sensory blocks. The effects of spinal anesthesia and epidural anesthesia without
epinephrine were generally comparable and are both qualitatively and quantitatively
different from the effects of epidural anesthesia with epinephrine added to the local
anesthetic solution.

Cousin‘s

Figure 8-18. Cardiovascular effects of epidural block to T5 level, with and without
epinephrine. A: Mean arterial pressure. B: Cardiac output (I could not download it ) C:
peripheral resistance.

185
Figure 8-19. Cardiovascular effects of epidural block. A, B: Blockade of T1-T4 alone
compared with control measurements (control). C: Blockade of T1-T4 alone (upper) is
compared with total blockade of T1-S5 (total) and control measurements (control). Note:
Block of T1-T4 alone results in reductions in blood pressure, heart rate, and cardiac
index, accompanied by a rise in central venous pressure. Extension of block to T1-S5
results in a further fall in CVP and cardiac index. (Parts A and B from Otton, P.E., and
Wilson, E.J.: The cardiocirculatory effects of upper thoracic epidural analgesia. Can.
Anaesth. Soc. J., 13:541, 1966; Part C from McLean, A.P.H., Mulligan, G.W., Otton, P.,
and MacLean, L.D.: Hemodynamic alterations associated with epidural anesthesia.
Surgery, 62:79, 1967.)

Cousin‘s

Epinephrine-Containing Solutions. The preceding discussion has focused on studies


using plain solutions of local anesthetic. Vascular absorption of added epinephrine does
result in systemic actions on -adrenergic receptors. It is now well established that the
cardiovascular effects of low-dose epinephrine are quite different from its traditional
picture of tachycardia, hypertension, and peripheral ischemia. Systemic effects of doses
of epinephrine in the range of 80 to 130 mcg, as used in epidural block, are a
moderate increase in heart rate, increased cardiac output, decreased peripheral
resistance, and decreased mean arterial pressure. Bonica and colleagues40 attribute
these effects solely to B-adrenergic stimulation.

The most likely explanations for the more pronounced cardiovascular effects of
epinephrine-containing local anesthetic solutions appear to be as follows:

1. Systemic absorption of epinephrine: B-adrenergic effects on the heart, resulting in


increased heart rate and cardiac output; peripheral vascular B-adrenergic effects,
resulting in further vasodilatation within the area of sympathetic block and antagonism of
compensatory vasoconstrictor responses outside the area of blockade. Thus, total
peripheral resistance falls, and mean arterial blood pressure is reduced to a degree
comparable to that seen with equivalent levels of subarachnoid blockade (Fig. 8-18).

186
235) All of the following increase the risk of pulmonary artery perforation
EXCEPT:
a) Age > 60 y o
b) Anticoagulated pt
c) Induced hypothermia
d) Heparin bonded PAC
D

236) All of the following decrease K+ concentration EXCEPT:


a) Epinephrine
b) Kayexalate
c) Calcium gluconate
d) Insulin
C

237) Which is true about CO2 lasers:


a) Causes corneal burns
b) ?
c) ?
d) ?
A

238) In regard to a properly functioning variable bypass vaporizer, what will


increase as the barometric pressure decreases
a) Vapor pressure
b) Vapor pressure as a proportion of output (conc. %)
c) Partial pressure of the vapor
d) Partial pressure within the vaporizer
C Repeat 177

239) Which least affects SSEP monitoring:


a) Fentanyl infusion
b) Induced hypotension with nitroprusside
c) N2O and bolus sufentanil
d) Propofol infusion
A
We had this question many many times. I revisited the answer and tried to make a
summary.
Barash:
 For SSEPs, a 50% reduction in amplitude from baseline in response to a specific
surgical maneuver is considered to be a significant change warranting action to
avert potential damage. For BAEPs, an increase in latency of more than 1
millisecond is considered clinically significant.
 Evoked potentials of cortical origin (i.e., the cortical component of the SSEP and
VEP) are more vulnerable to anesthetic influences than brain stem potentials (e.g.,
BAEP and the subcortical components of SSEP).

187
 In general, to obtain satisfactory intraoperative SEP recordings, it is important to
maintain constant anesthetic drug levels. Specifically, bolus administration of
intravenous agents and step changes in inspired inhalation agent concentration
must be avoided, especially at times when neurologic injury might occur.

When recording cortical evoked potentials (SSEPs or VEPs), one should employ
intravenous techniques. High concentrations of volatile agents essentially eliminate
cortical evoked potentials. However, end-tidal concentrations of 0.5 MAC of a volatile
agent are compatible with satisfactory recordings in patients who are neurologically
normal.

 In general, volatile agents cause a dose-dependent increase in latency and a decrease


in amplitude of the cortical SSEP or VEP.
 Nitrous oxide alone has been shown to produce significant decreases in amplitude
with minimal latency changes in the cortical SSEP, but it decreases amplitude and
increases latency in the VEP.178–180 When nitrous oxide is administered in combination
with a volatile anesthetic, it produces a profound depressant effect on SSEPs or VEPs.
 Studies on the effects of intravenous agents demonstrate that induction doses of
thiopental, etomidate, and fentanyl preserve SSEP recordings.
 The benzodiazepines produce minimal SSEP& VEP changes and no changes in
BAEPs.
 Propofol increases the latency and decreases the amplitude of cortical SSEPs.
Propofol (2 mg/kg iv followed by an infusion) increases the BAEP latency of I, III, and
V waves without changing the amplitudes. Propofol completely suppresses
middlelatency auditory potentials
 The opioids produce minimal changes in SEP waveforms. For example, fentanyl and
morphine cause minimal latency prolongation and amplitude depression of the SSEP
waveforms, and high-dose opioid administration has been shown to be compatible with
reproducible recordings of SSEPs. Opioids also produce minimal to no effect on BAEP
recordings. Furthermore, low-dose continuous infusions of opioids tend to depress
SEPs less than intermittent bolus injections.
 Because opioids preserve SEP recordings even in relatively high doses, they are
recommended for use as infusions during intraoperative monitoring. As with all
intravenous agents used, bolus administration should be avoided during critical times
when neurologic injury might occur.
 Physiologic factors such as temperature, systemic blood pressure, PaO2, and PaCO2
can alter SEPs and must be controlled during intraoperative recordings. Both
hypothermia and hyperthermia alter all SEPs.198–200 In addition, fluids used to irrigate
the brain or spinal cord can cause marked changes in recordings despite normal core
temperature measurement.
 Systemic hypotension below levels of cerebral autoregulation produces progressive
decreases in amplitude of cortical SSEPs until the waveform is lost.

Motor Evoked Potentials

Motor evoked potentials are extremely sensitive to depression by anesthetics. Table 28-
9 summarizes the known effects of intravenous and inhaled agents. Increasing N2O
concentration gradually attenuates and abolishes MEPs at concentrations between 60
and 70%. However, N2O concentration of 50% is reported to cause a minimal response

188
alteration and appears to be compatible with monitoring. The volatile agents are powerful
depressants of myogenic MEPs. During nitrous oxide–opioid anesthesia, even low
concentrations of isoflurane are reported to abolish MEP responses. Benzodiazepines,
barbiturates, and propofol also produce marked depressed of the myogenic MEP.
Fentanyl, etomidate, and ketamine have little or no effect on myogenic MEP and are
compatible with intraoperative recording. Muscle relaxants affect the recorded
electromyographic response by depressing myoneural transmission. By adjusting a
continuous infusion of muscle relaxant to maintain one or two twitches in a train of four,
reliable MEP responses have been recorded. It appears that agents with CNS inhibitory
action (e.g., thiopental and midazolam) produce profound changes in MEP responses
and should be avoided. As with SEPs, hypothermia, hypoxia, and hypotension will alter
MEPs under anesthesia.

Table 28-9. EFFECTS OF INTRAVENOUSAND INHALED AGENTS ON


MOTOREVOKED POTENTIALS
Latency Amplitude
INTRAVENOUS AGENTS
Thiopental 
Diazepam 
Midazolam 0 
Fentanyl 0 0
Etomidate 0 /0
Propofol 0 
Ketamine* 
INHALED AGENTS
Desflurane 
Enflurane 
Halothane 
Isoflurane 
Sevoflurane 
Nitrous oxide 

= increased;  = decreased; 0 = no change.

* Latency delay in doses ≥35–40 mg•kg–1, amplitude depression in doses ≥15–20


mg•kg–1.

189

You might also like